Anda di halaman 1dari 46

80 FUNDA QUESTIONS d.

Malpractice
14. Which of the following is an example of nursing malpractice?
1. The most appropriate nursing order for a patient who develops dyspnea and a. The nurse administers penicillin to a patient with a documented history of
shortness of breath would be… allergy to the drug. The patient experiences an allergic reaction and has cerebral
a.Maintain the patient on strict bed rest at all times damage resulting from anoxia.
b.Maintain the patient in an orthopneic position as needed b. The nurse applies a hot water bottle or a heating pad to the abdomen of a
c.Administer oxygen by Venturi mask at 24%, as needed patient with abdominal cramping.
d. Allow a 1 hour rest period between activities c. The nurse assists a patient out of bed with the bed locked in position; the
patient slips and fractures his right humerus.
2. The nurse observes that Mr. Adams begins to have increased difficulty d. The nurse administers the wrong medication to a patient and the patient
breathing. She elevates the head of the bed to the high Fowler position, which vomits. This information is documented and reported to the physician and the
decreases his respiratory distress. The nurse documents this breathing as: nursing supervisor.
a. Tachypnea 15. Which of the following signs and symptoms would the nurse expect to find
b. Eupnca when assessing an Asian patient for postoperative pain following abdominal
c. Orthopnea surgery?
d. Hyperventilation a. Decreased blood pressure and heart rate and shallow respirations
b. Quiet crying
3. The physician orders a platelet count to be performed on Mrs. Smith after c. Immobility, diaphoresis, and avoidance of deep breathing or coughing
breakfast. The nurse is responsible for: d. Changing position every 2 hours
a. Instructing the patient about this diagnostic test 16. A patient is admitted to the hospital with complaints of nausea, vomiting,
b. Writing the order for this test diarrhea, and severe abdominal pain. Which of the following would immediately
c. Giving the patient breakfast alert the nurse that the patient has bleeding from the GI tract?
d. All of the above a. Complete blood count
b. Guaiac test
4. Mrs. Mitchell has been given a copy of her diet. The nurse discusses the foods c. Vital signs
allowed on a 500-mg low sodium diet. These include: d. Abdominal girth
a. A ham and Swiss cheese sandwich on whole wheat bread 17. The correct sequence for assessing the abdomen is:
b. Mashed potatoes and broiled chicken a. Tympanic percussion, measurement of abdominal girth, and inspection
c. A tossed salad with oil and vinegar and olives b. Assessment for distention, tenderness, and discoloration around the
d. Chicken bouillon umbilicus.
c. Percussions, palpation, and auscultation
5. The physician orders a maintenance dose of 5,000 units of subcutaneous d. Auscultation, percussion, and palpation
heparin (an anticoagulant) daily. Nursing responsibilities for Mrs. Mitchell now 18. High-pitched gurgles head over the right lower quadrant are:
include: a. A sign of increased bowel motility
a. Reviewing daily activated partial thromboplastin time (APTT) and prothrombin b. A sign of decreased bowel motility
time. c. Normal bowel sounds
b. Reporting an APTT above 45 seconds to the physician d. A sign of abdominal cramping
c. Assessing the patient for signs and symptoms of frank and occult bleeding 19. A patient about to undergo abdominal inspection is best placed in which of
d. All of the above the following positions?
a. Prone
6. The four main concepts common to nursing that appear in each of the current b. Trendelenburg
conceptual models are: c. Supine
a. Person, nursing, environment, medicine d. Side-lying
b. Person, health, nursing, support systems 20. For a rectal examination, the patient can be directed to assume which of
c. Person, health, psychology, nursing the following positions?
d. Person, environment, health, nursing a. Genupecterol
b. Sims
7. In Maslow’s hierarchy of physiologic needs, the human need of greatest c. Horizontal recumbent
priority is: d. All of the above
a. Love 21. During a Romberg test, the nurse asks the patient to assume which
b. Elimination position?
c. Nutrition a. Sitting
d. Oxygen b. Standing
8. The family of an accident victim who has been declared brain-dead seems c. Genupectoral
amenable to organ donation. What should the nurse do? d. Trendelenburg
a. Discourage them from making a decision until their grief has eased 22. If a patient’s blood pressure is 150/96, his pulse pressure is:
b. Listen to their concerns and answer their questions honestly a. 54
c. Encourage them to sign the consent form right away b. 96
d. Tell them the body will not be available for a wake or funeral c. 150
9. A new head nurse on a unit is distressed about the poor staffing on the 11 d. 246
p.m. to 7 a.m. shift. What should she do? 23. A patient is kept off food and fluids for 10 hours before surgery. His oral
a. Complain to her fellow nurses temperature at 8 a.m. is 99.8 F (37.7 C) This temperature reading probably
b. Wait until she knows more about the unit indicates:
c. Discuss the problem with her supervisor a. Infection
d. Inform the staff that they must volunteer to rotate b. Hypothermia
10. Which of the following principles of primary nursing has proven the most c. Anxiety
satisfying to the patient and nurse? d. Dehydration
a. Continuity of patient care promotes efficient, cost-effective nursing care 24. Which of the following parameters should be checked when assessing
b. Autonomy and authority for planning are best delegated to a nurse who respirations?
knows the patient well a. Rate
c. Accountability is clearest when one nurse is responsible for the overall plan b. Rhythm
and its implementation. c. Symmetry
d. The holistic approach provides for a therapeutic relationship, continuity, d. All of the above
and efficient nursing care. 25. A 38-year old patient’s vital signs at 8 a.m. are axillary temperature 99.6 F
11. If nurse administers an injection to a patient who refuses that injection, she (37.6 C); pulse rate, 88; respiratory rate, 30. Which findings should be reported?
has committed: a. Respiratory rate only
a. Assault and battery b. Temperature only
b. Negligence c. Pulse rate and temperature
c. Malpractice d. Temperature and respiratory rate
d. None of the above 26. All of the following can cause tachycardia except:
12. If patient asks the nurse her opinion about a particular physicians and the a. Fever
nurse replies that the physician is incompetent, the nurse could be held liable for: b. Exercise
a. Slander c. Sympathetic nervous system stimulation
b. Libel d. Parasympathetic nervous system stimulation
c. Assault 27. Palpating the midclavicular line is the correct technique for assessing
d. Respondent superior a. Baseline vital signs
13. A registered nurse reaches to answer the telephone on a busy pediatric b. Systolic blood pressure
unit, momentarily turning away from a 3 month-old infant she has been weighing. c. Respiratory rate
The infant falls off the scale, suffering a skull fracture. The nurse could be d. Apical pulse
charged with: 28. The absence of which pulse may not be a significant finding when a patient
a. Defamation is admitted to the hospital?
b. Assault a. Apical
c. Battery b. Radial
c. Pedal b. Side rails should not be used
d. Femoral c. Side rails are a deterrent that prevent a patient from falling out of bed.
29. Which of the following patients is at greatest risk for developing pressure d. Side rails are a reminder to a patient not to get out of bed
ulcers? 43. Examples of patients suffering from impaired awareness include all of the
a. An alert, chronic arthritic patient treated with steroids and aspirin following except:
b. An 88-year old incontinent patient with gastric cancer who is confined to a. A semiconscious or over fatigued patient
his bed at home b. A disoriented or confused patient
c. An apathetic 63-year old COPD patient receiving nasal oxygen via cannula c. A patient who cannot care for himself at home
d. A confused 78-year old patient with congestive heart failure (CHF) who d. A patient demonstrating symptoms of drugs or alcohol withdrawal
requires assistance to get out of bed. 44. The most common injury among elderly persons is:
30. The physician orders the administration of high-humidity oxygen by face a. Atheroscleotic changes in the blood vessels
mask and placement of the patient in a high Fowler’s position. After assessing b. Increased incidence of gallbladder disease
Mrs. Paul, the nurse writes the following nursing diagnosis: Impaired gas c. Urinary Tract Infection
exchange related to increased secretions. Which of the following nursing d. Hip fracture
interventions has the greatest potential for improving this situation? 45. The most common psychogenic disorder among elderly person is:
a. Encourage the patient to increase her fluid intake to 200 ml every 2 hours a. Depression
b. Place a humidifier in the patient’s room. b. Sleep disturbances (such as bizarre dreams)
c. Continue administering oxygen by high humidity face mask c. Inability to concentrate
d. Perform chest physiotheraphy on a regular schedule d. Decreased appetite
31. The most common deficiency seen in alcoholics is: 46. Which of the following vascular system changes results from aging?
a. Thiamine a. Increased peripheral resistance of the blood vessels
b. Riboflavin b. Decreased blood flow
c. Pyridoxine c. Increased work load of the left ventricle
d. Pantothenic acid d. All of the above
32. Which of the following statement is incorrect about a patient with 47. Which of the following is the most common cause of dementia among
dysphagia? elderly persons?
a. The patient will find pureed or soft foods, such as custards, easier to a. Parkinson’s disease
swallow than water b. Multiple sclerosis
b. Fowler’s or semi Fowler’s position reduces the risk of aspiration during c. Amyotrophic lateral sclerosis (Lou Gerhig’s disease)
swallowing d. Alzheimer’s disease
c. The patient should always feed himself 48. The nurse’s most important legal responsibility after a patient’s death in a
d. The nurse should perform oral hygiene before assisting with feeding. hospital is:
33. To assess the kidney function of a patient with an indwelling urinary a. Obtaining a consent of an autopsy
(Foley) catheter, the nurse measures his hourly urine output. She should notify b. Notifying the coroner or medical examiner
the physician if the urine output is: c. Labeling the corpse appropriately
a. Less than 30 ml/hour d. Ensuring that the attending physician issues the death certification
b. 64 ml in 2 hours 49. Before rigor mortis occurs, the nurse is responsible for:
c. 90 ml in 3 hours a. Providing a complete bath and dressing change
d. 125 ml in 4 hours b. Placing one pillow under the body’s head and shoulders
34. Certain substances increase the amount of urine produced. These include: c. Removing the body’s clothing and wrapping the body in a shroud
a. Caffeine-containing drinks, such as coffee and cola. d. Allowing the body to relax normally
b. Beets 50. When a patient in the terminal stages of lung cancer begins to exhibit loss
c. Urinary analgesics of consciousness, a major nursing priority is to:
d. Kaolin with pectin (Kaopectate) a. Protect the patient from injury
35. A male patient who had surgery 2 days ago for head and neck cancer is b. Insert an airway
about to make his first attempt to ambulate outside his room. The nurse notes c. Elevate the head of the bed
that he is steady on his feet and that his vision was unaffected by the surgery. d. Withdraw all pain medications
Which of the following nursing interventions would be appropriate?
a. Encourage the patient to walk in the hall alone ______________________________________
b. Discourage the patient from walking in the hall for a few more days 1. Nurse Brenda is teaching a patient about a newly prescribed drug. What
c. Accompany the patient for his walk. could cause a geriatric patient to have difficulty retaining knowledge about
d. Consuit a physical therapist before allowing the patient to ambulate prescribed medications?
36. A patient has exacerbation of chronic obstructive pulmonary disease a. Decreased plasma drug levels
(COPD) manifested by shortness of breath; orthopnea: thick, tenacious b. Sensory deficits
secretions; and a dry hacking cough. An appropriate nursing diagnosis would be: c. Lack of family support
a. Ineffective airway clearance related to thick, tenacious secretions. d. History of Tourette syndrome
b. Ineffective airway clearance related to dry, hacking cough. 2. When examining a patient with abdominal pain the nurse in charge should
c. Ineffective individual coping to COPD. assess:
d. Pain related to immobilization of affected leg. a. Any quadrant first
37. Mrs. Lim begins to cry as the nurse discusses hair loss. The best response b. The symptomatic quadrant first
would be: c. The symptomatic quadrant last
a. “Don’t worry. It’s only temporary” d. The symptomatic quadrant either second or third
b. “Why are you crying? I didn’t get to the bad news yet” 3. The nurse is assessing a postoperative adult patient. Which of the following
c. “Your hair is really pretty” should the nurse document as subjective data?
d. “I know this will be difficult for you, but your hair will grow back after the a. Vital signs
completion of chemotheraphy” b. Laboratory test result
38. An additional Vitamin C is required during all of the following periods c. Patient’s description of pain
except: d. Electrocardiographic (ECG) waveforms
a. Infancy 4. A male patient has a soft wrist-safety device. Which assessment finding
b. Young adulthood should the nurse consider abnormal?
c. Childhood a. A palpable radial pulse
d. Pregnancy b. A palpable ulnar pulse
39. A prescribed amount of oxygen s needed for a patient with COPD to c. Cool, pale fingers
prevent: d. Pink nail beds
a. Cardiac arrest related to increased partial pressure of carbon dioxide in 5. Which of the following planes divides the body longitudinally into anterior and
arterial blood (PaCO2) posterior regions?
b. Circulatory overload due to hypervolemia a. Frontal plane
c. Respiratory excitement b. Sagittal plane
d. Inhibition of the respiratory hypoxic stimulus c. Midsagittal plane
40. After 1 week of hospitalization, Mr. Gray develops hypokalemia. Which of d. Transverse plane
the following is the most significant symptom of his disorder? 6. A female patient with a terminal illness is in denial. Indicators of denial
a. Lethargy include:
b. Increased pulse rate and blood pressure a. Shock dismay
c. Muscle weakness b. Numbness
d. Muscle irritability c. Stoicism
41. Which of the following nursing interventions promotes patient safety? d. Preparatory grief
a. Asses the patient’s ability to ambulate and transfer from a bed to a chair 7. The nurse in charge is transferring a patient from the bed to a chair. Which
b. Demonstrate the signal system to the patient action does the nurse take during this patient transfer?
c. Check to see that the patient is wearing his identification band a. Position the head of the bed flat
d. All of the above b. Helps the patient dangle the legs
42. Studies have shown that about 40% of patients fall out of bed despite the c. Stands behind the patient
use of side rails; this has led to which of the following conclusions? d. Places the chair facing away from the bed
a. Side rails are ineffective
8. A female patient who speaks a little English has emergency gallbladder a. Primary prevention
surgery, during discharge preparation, which nursing action would best help this b. Secondary prevention
patient understand wound care instruction? c. Tertiary prevention
a. Asking frequently if the patient understands the instruction d. Passive prevention
b. Asking an interpreter to replay the instructions to the patient. 22. What does the nurse in charge do when making a surgical bed?
c. Writing out the instructions and having a family member read them to the a. Leaves the bed in the high position when finished
patient b. Places the pillow at the head of the bed
d. Demonstrating the procedure and having the patient return the c. Rolls the patient to the far side of the bed
demonstration d. Tucks the top sheet and blanket under the bottom of the bed
9. Before administering the evening dose of a prescribed medication, the nurse 23. The physician prescribes 250 mg of a drug. The drug vial reads 500 mg/ml.
on the evening shift finds an unlabeled, filled syringe in the patient’s medication how much of the drug should the nurse give?
drawer. What should the nurse in charge do? a. 2 ml
a. Discard the syringe to avoid a medication error b. 1 ml
b. Obtain a label for the syringe from the pharmacy c. ½ ml
c. Use the syringe because it looks like it contains the same medication the d. ¼ ml
nurse was prepared to give 24. Nurse Mackey is monitoring a patient for adverse reactions during
d. Call the day nurse to verify the contents of the syringe barbiturate therapy. What is the major disadvantage of barbiturate use?
10. When administering drug therapy to a male geriatric patient, the nurse must a. Prolonged half-life
stay especially alert for adverse effects. Which factor makes geriatric patients to b. Poor absorption
adverse drug effects? c. Potential for drug dependence
a. Faster drug clearance d. Potential for hepatotoxicity
b. Aging-related physiological changes 25. Which nursing action is essential when providing continuous enteral
c. Increased amount of neurons feeding?
d. Enhanced blood flow to the GI tract a. Elevating the head of the bed
11. A female patient is being discharged after cataract surgery. After providing b. Positioning the patient on the left side
medication teaching, the nurse asks the patient to repeat the instructions. The c. Warming the formula before administering it
nurse is performing which professional role? d. Hanging a full day’s worth of formula at one time
a. Manager 26. When teaching a female patient how to take a sublingual tablet, the nurse
b. Educator should instruct the patient to place the table on the:
c. Caregiver a. Top of the tongue
d. Patient advocate b. Roof of the mouth
12. A female patient exhibits signs of heightened anxiety. Which response by c. Floor of the mouth
the nurse is most likely to reduce the patient’s anxiety? d. Inside of the cheek
a. “Everything will be fine. Don’t worry.” 27. Which action by the nurse in charge is essential when cleaning the area
b. “Read this manual and then ask me any questions you may have.” around a Jackson-Pratt wound drain?
c. “Why don’t you listen to the radio?” a. Cleaning from the center outward in a circular motion
d. “Let’s talk about what’s bothering you.” b. Removing the drain before cleaning the skin
13. A scrub nurse in the operating room has which responsibility? c. Cleaning briskly around the site with alcohol
a. Positioning the patient d. Wearing sterile gloves and a mask
b. Assisting with gowning and gloving 28. The doctor orders dextrose 5% in water, 1,000 ml to be infused over 8
c. Handling surgical instruments to the surgeon hours. The I.V. tubing delivers 15 drops per milliliter. The nurse in charge should
d. Applying surgical drapes run the I.V. infusion at a rate of:
14. A patient is in the bathroom when the nurse enters to give a prescribed a. 15 drop per minute
medication. What should the nurse in charge do? b. 21 drop per minute
a. Leave the medication at the patient’s bedside c. 32 drop per minute
b. Tell the patient to be sure to take the medication. And then leave it at the d. 125 drops per minute
bedside 29. A male patient undergoes a total abdominal hysterectomy. When assessing
c. Return shortly to the patient’s room and remain there until the patient takes the patient 10 hours later, the nurse identifies which finding as an early sign of
the medication shock?
d. Wait for the patient to return to bed, and then leave the medication at the a. Restlessness
bedside b. Pale, warm, dry skin
15. The physician orders heparin, 7,500 units, to be administered c. Heart rate of 110 beats/minute
subcutaneously every 6 hours. The vial reads 10,000 units per milliliter. The d. Urine output of 30 ml/hour
nurse should anticipate giving how much heparin for each dose? 30. Which pulse should the nurse palpate during rapid assessment of an
a. ¼ ml unconscious male adult?
b. ½ ml a. Radial
c. ¾ ml b. Brachial
d. 1 ¼ ml c. Femoral
16. The nurse in charge measures a patient’s temperature at 102 degrees F. d. Carotid
what is the equivalent Centigrade temperature? ______________________________________
a. 39 degrees C ANSWERS and RATIONALES for FUNDAMENTALS OF NURSING PRACTICE
b. 47 degrees C QUESTIONS
c. 38.9 degrees C
d. 40.1 degrees C
17. To evaluate a patient for hypoxia, the physician is most likely to order which
laboratory test? 1. B. When a patient develops dyspnea and shortness of breath, the
a. Red blood cell count orthopneic position encourages maximum chest expansion and keeps the
b. Sputum culture abdominal organs from pressing against the diaphragm, thus improving
c. Total hemoglobin ventilation. Bed rest and oxygen by Venturi mask at 24% would improve
d. Arterial blood gas (ABG) analysis oxygenation of the tissues and cells but must be ordered by a physician. Allowing
18. The nurse uses a stethoscope to auscultate a male patient’s chest. Which for rest periods decreases the possibility of hypoxia.
statement about a stethoscope with a bell and diaphragm is true? 2. C. Orthopnea is difficulty of breathing except in the upright position.
a. The bell detects high-pitched sounds best Tachypnea is rapid respiration characterized by quick, shallow breaths. Eupnea
b. The diaphragm detects high-pitched sounds best is normal respiration – quiet, rhythmic, and without effort.
c. The bell detects thrills best 3. C. A platelet count evaluates the number of platelets in the circulating
d. The diaphragm detects low-pitched sounds best blood volume. The nurse is responsible for giving the patient breakfast at the
19. A male patient is to be discharged with a prescription for an analgesic that scheduled time. The physician is responsible for instructing the patient about the
is a controlled substance. During discharge teaching, the nurse should explain test and for writing the order for the test.
that the patient must fill this prescription how soon after the date on which it was 4. B. Mashed potatoes and broiled chicken are low in natural sodium
written? chloride. Ham, olives, and chicken bouillon contain large amounts of sodium and
a. Within 1 month are contraindicated on a low sodium diet.
b. Within 3 months 5. D. All of the identified nursing responsibilities are pertinent when a patient
c. Within 6 months is receiving heparin. The normal activated partial thromboplastin time is 16 to 25
d. Within 12 months seconds and the normal prothrombin time is 12 to 15 seconds; these levels must
20. Which human element considered by the nurse in charge during remain within two to two and one half the normal levels. All patients receiving
assessment can affect drug administration? anticoagulant therapy must be observed for signs and symptoms of frank and
a. The patient’s ability to recover occult bleeding (including hemorrhage, hypotension, tachycardia, tachypnea,
b. The patient’s occupational hazards restlessness, pallor, cold and clammy skin, thirst and confusion); blood pressure
c. The patient’s socioeconomic status should be measured every 4 hours and the patient should be instructed to report
d. The patient’s cognitive abilities promptly any bleeding that occurs with tooth brushing, bowel movements,
21. An employer establishes a physical exercise area in the workplace and urination or heavy prolonged menstruation.
encourages all employees to use it. This is an example of which level of health 6. D. The focus concepts that have been accepted by all theorists as the
promotion? focus of nursing practice from the time of Florence Nightingale include the person
receiving nursing care, his environment, his health on the health illness temperature is approximately one degree lower and a rectal temperature, one
continuum, and the nursing actions necessary to meet his needs. degree higher. Thus, an axillary temperature of 99.6°F (37.6°C) would be
7. D. Maslow, who defined a need as a satisfaction whose absence causes considered abnormal. The resting pulse rate in an adult ranges from 60 to 100
illness, considered oxygen to be the most important physiologic need; without it, beats/minute, so a rate of 88 is normal.
human life could not exist. According to this theory, other physiologic needs 26. D. Parasympathetic nervous system stimulation of the heart decreases the
(including food, water, elimination, shelter, rest and sleep, activity and heart rate as well as the force of contraction, rate of impulse conduction and
temperature regulation) must be met before proceeding to the next hierarchical blood flow through the coronary vessels. Fever, exercise, and sympathetic
levels on psychosocial needs. stimulation all increase the heart rate.
8. B. The brain-dead patient’s family needs support and reassurance in 27. D. The apical pulse (the pulse at the apex of the heart) is located on the
making a decision about organ donation. Because transplants are done within midclavicular line at the fourth, fifth, or sixth intercostal space. Base line vital
hours of death, decisions about organ donation must be made as soon as signs include pulse rate, temperature, respiratory rate, and blood pressure. Blood
possible. However, the family’s concerns must be addressed before members pressure is typically assessed at the antecubital fossa, and respiratory rate is
are asked to sign a consent form. The body of an organ donor is available for assessed best by observing chest movement with each inspiration and
burial. expiration.
9. C. Although a new head nurse should initially spend time observing the 28. C. Because the pedal pulse cannot be detected in 10% to 20% of the
unit for its strengths and weakness, she should take action if a problem threatens population, its absence is not necessarily a significant finding. However, the
patient safety. In this case, the supervisor is the resource person to approach. presence or absence of the pedal pulse should be documented upon admission
10. D. Studies have shown that patients and nurses both respond well to so that changes can be identified during the hospital stay. Absence of the apical,
primary nursing care units. Patients feel less anxious and isolated and more radial, or femoral pulse is abnormal and should be investigated.
secure because they are allowed to participate in planning their own care. 29. B. Pressure ulcers are most likely to develop in patients with impaired
Nurses feel personal satisfaction, much of it related to positive feedback from the mental status, mobility, activity level, nutrition, circulation and bladder or bowel
patients. They also seem to gain a greater sense of achievement and esprit de control. Age is also a factor. Thus, the 88-year old incontinent patient who has
corps. impaired nutrition (from gastric cancer) and is confined to bed is at greater risk.
11. A. Assault is the unjustifiable attempt or threat to touch or injure another 30. A. Adequate hydration thins and loosens pulmonary secretions and also
person. Battery is the unlawful touching of another person or the carrying out of helps to replace fluids lost from elevated temperature, diaphoresis, dehydration
threatened physical harm. Thus, any act that a nurse performs on the patient and dyspnea. High- humidity air and chest physiotherapy help liquefy and
against his will is considered assault and battery. mobilize secretions.
12. A. Oral communication that injures an individual’s reputation is considered 31. A. Chronic alcoholism commonly results in thiamine deficiency and other
slander. Written communication that does the same is considered libel. symptoms of malnutrition.
13. D. Malpractice is defined as injurious or unprofessional actions that harm 32. C. A patient with dysphagia (difficulty swallowing) requires assistance with
another. It involves professional misconduct, such as omission or commission of feeding. Feeding himself is a long-range expected outcome. Soft foods, Fowler’s
an act that a reasonable and prudent nurse would or would not do. In this or semi-Fowler’s position, and oral hygiene before eating should be part of the
example, the standard of care was breached; a 3-month-old infant should never feeding regimen.
be left unattended on a scale. 33. A. A urine output of less than 30ml/hour indicates hypovolemia or oliguria,
14. A. The three elements necessary to establish a nursing malpractice are which is related to kidney function and inadequate fluid intake.
nursing error (administering penicillin to a patient with a documented allergy to 34. A. Fluids containing caffeine have a diuretic effect. Beets and urinary
the drug), injury (cerebral damage), and proximal cause (administering the analgesics, such as pyridium, can color urine red. Kaopectate is an anti diarrheal
penicillin caused the cerebral damage). Applying a hot water bottle or heating medication.
pad to a patient without a physician’s order does not include the three required 35. C. A hospitalized surgical patient leaving his room for the first time fears
components. Assisting a patient out of bed with the bed locked in position is the rejection and others staring at him, so he should not walk alone. Accompanying
correct nursing practice; therefore, the fracture was not the result of malpractice. him will offer moral support, enabling him to face the rest of the world. Patients
Administering an incorrect medication is a nursing error; however, if such action should begin ambulation as soon as possible after surgery to decrease
resulted in a serious illness or chronic problem, the nurse could be sued for complications and to regain strength and confidence. Waiting to consult a
malpractice. physical therapist is unnecessary.
15. C. An Asian patient is likely to hide his pain. Consequently, the nurse must 36. A. Thick, tenacious secretions, a dry, hacking cough, orthopnea, and
observe for objective signs. In an abdominal surgery patient, these might include shortness of breath are signs of ineffective airway clearance. Ineffective airway
immobility, diaphoresis, and avoidance of deep breathing or coughing, as well as clearance related to dry, hacking cough is incorrect because the cough is not the
increased heart rate, shallow respirations (stemming from pain upon moving the reason for the ineffective airway clearance. Ineffective individual coping related to
diaphragm and respiratory muscles), and guarding or rigidity of the abdominal COPD is wrong because the etiology for a nursing diagnosis should not be a
wall. Such a patient is unlikely to display emotion, such as crying. medical diagnosis (COPD) and because no data indicate that the patient is
16. B. To assess for GI tract bleeding when frank blood is absent, the nurse coping ineffectively. Pain related to immobilization of affected leg would be an
has two options: She can test for occult blood in vomitus, if present, or in stool – appropriate nursing diagnosis for a patient with a leg fracture.
through guaiac (Hemoccult) test. A complete blood count does not provide 37. D. “I know this will be difficult” acknowledges the problem and suggests a
immediate results and does not always immediately reflect blood loss. Changes resolution to it. “Don’t worry..” offers some relief but doesn’t recognize the
in vital signs may be cause by factors other than blood loss. Abdominal girth is patient’s feelings. “..I didn’t get to the bad news yet” would be inappropriate at
unrelated to blood loss. any time. “Your hair is really pretty” offers no consolation or alternatives to the
17. D. Because percussion and palpation can affect bowel motility and thus patient.
bowel sounds, they should follow auscultation in abdominal assessment. 38. B. Additional Vitamin C is needed in growth periods, such as infancy and
Tympanic percussion, measurement of abdominal girth, and inspection are childhood, and during pregnancy to supply demands for fetal growth and
methods of assessing the abdomen. Assessing for distention, tenderness and maternal tissues. Other conditions requiring extra vitamin C include wound
discoloration around the umbilicus can indicate various bowel-related conditions, healing, fever, infection and stress.
such as cholecystitis, appendicitis and peritonitis. 39. D. Delivery of more than 2 liters of oxygen per minute to a patient with
18. C. Hyperactive sounds indicate increased bowel motility; two or three chronic obstructive pulmonary disease (COPD), who is usually in a state of
sounds per minute indicate decreased bowel motility. Abdominal cramping with compensated respiratory acidosis (retaining carbon dioxide (CO2)), can inhibit
hyperactive, high pitched tinkling bowel sounds can indicate a bowel obstruction. the hypoxic stimulus for respiration. An increased partial pressure of carbon
19. C. The supine position (also called the dorsal position), in which the patient dioxide in arterial blood (PACO2) would not initially result in cardiac arrest.
lies on his back with his face upward, allows for easy access to the abdomen. In Circulatory overload and respiratory excitement have no relevance to the
the prone position, the patient lies on his abdomen with his face turned to the question.
side. In the Trendelenburg position, the head of the bed is tilted downward to 30 40. C. Presenting symptoms of hypokalemia ( a serum potassium level below
to 40 degrees so that the upper body is lower than the legs. In the lateral 3.5 mEq/liter) include muscle weakness, chronic fatigue, and cardiac
position, the patient lies on his side. dysrhythmias. The combined effects of inadequate food intake and prolonged
20. D. All of these positions are appropriate for a rectal examination. In the diarrhea can deplete the potassium stores of a patient with GI problems.
genupectoral (knee-chest) position, the patient kneels and rests his chest on the 41. D. Assisting a patient with ambulation and transfer from a bed to a chair
table, forming a 90 degree angle between the torso and upper legs. In Sims’ allows the nurse to evaluate the patient’s ability to carry out these functions
position, the patient lies on his left side with the left arm behind the body and his safely. Demonstrating the signal system and providing an opportunity for a return
right leg flexed. In the horizontal recumbent position, the patient lies on his back demonstration ensures that the patient knows how to operate the equipment and
with legs extended and hips rotated outward. encourages him to call for assistance when needed. Checking the patient’s
21. B. During a Romberg test, which evaluates for sensory or cerebellar identification band verifies the patient’s identity and prevents identification
ataxia, the patient must stand with feet together and arms resting at the sides— mistakes in drug administration.
first with eyes open, then with eyes closed. The need to move the feet apart to 42. D. Since about 40% of patients fall out of bed despite the use of side rails,
maintain this stance is an abnormal finding. side rails cannot be said to prevent falls; however, they do serve as a reminder
22. A. The pulse pressure is the difference between the systolic and diastolic that the patient should not get out of bed. The other answers are incorrect
blood pressure readings – in this case, 54. interpretations of the statistical data.
23. D. A slightly elevated temperature in the immediate preoperative or post 43. C. A patient who cannot care for himself at home does not necessarily
operative period may result from the lack of fluids before surgery rather than from have impaired awareness; he may simply have some degree of immobility.
infection. Anxiety will not cause an elevated temperature. Hypothermia is an 44. D. Hip fracture, the most common injury among elderly persons, usually
abnormally low body temperature. results from osteoporosis. The other answers are diseases that can occur in the
24. D. The quality and efficiency of the respiratory process can be determined elderly from physiologic changes.
by appraising the rate, rhythm, depth, ease, sound, and symmetry of respirations. 45. A. Sleep disturbances, inability to concentrate and decreased appetite are
25. D. Under normal conditions, a healthy adult breathes in a smooth symptoms of depression, the most common psychogenic disorder among elderly
uninterrupted pattern 12 to 20 times a minute. Thus, a respiratory rate of 30 persons. Other symptoms include diminished memory, apathy, disinterest in
would be abnormal. A normal adult body temperature, as measured on an oral appearance, withdrawal, and irritability. Depression typically begins before the
thermometer, ranges between 97° and 100°F (36.1° and 37.8°C); an axillary
onset of old age and usually is caused by psychosocial, genetic, or biochemical nurse should never leave medication at the patient’s bedside unless specifically
factors requested to do so.
46. D. Aging decreases elasticity of the blood vessels, which leads to Answer C. The nurse solves the problem as follows:
increased peripheral resistance and decreased blood flow. These changes, in 10,000 units/7,500 units = 1 ml/X
turn, increase the work load of the left ventricle. 10,000 X = 7,500
47. D. Alzheimer;s disease, sometimes known as senile dementia of the X= 7,500/10,000 or ¾ ml
Alzheimer’s type or primary degenerative dementia, is an insidious; progressive, Answer C. To convert Fahrenheit degrees to centigrade, use this formula:
irreversible, and degenerative disease of the brain whose etiology is still C degrees = (F degrees – 32) x 5/9
unknown. Parkinson’s disease is a neurologic disorder caused by lesions in the C degrees = (102 – 32) 5/9
extrapyramidial system and manifested by tremors, muscle rigidity, hypokinesis, + 70 x 5/9
dysphagia, and dysphonia. Multiple sclerosis, a progressive, degenerative 38.9 degrees C
disease involving demyelination of the nerve fibers, usually begins in young Answer D. All of these test help evaluate a patient with respiratory problems.
adulthood and is marked by periods of remission and exacerbation. Amyotrophic However, ABG analysis is the only test evaluates gas exchange in the lungs,
lateral sclerosis, a disease marked by progressive degeneration of the neurons, providing information about patient’s oxygenation status.
eventually results in atrophy of all the muscles; including those necessary for Answer B. The diaphragm of a stethoscope detects high-pitched sound best; the
respiration. bell detects low pitched sounds best. Palpation detects thrills best.
48. C. The nurse is legally responsible for labeling the corpse when death Answer C. In most cases, an outpatient must fill a prescription for a controlled
occurs in the hospital. She may be involved in obtaining consent for an autopsy substance within 6 months of the date on which the prescription was written.
or notifying the coroner or medical examiner of a patient’s death; however, she is Answer D. The nurse must consider the patient’s cognitive abilities to understand
not legally responsible for performing these functions. The attending physician drug instructions. If not, the nurse must find a family member or significant other
may need information from the nurse to complete the death certificate, but he is to take on the responsibility of administering medications in the home setting.
responsible for issuing it. The patient’s ability to recover, occupational hazards, and socioeconomic status
49. B. The nurse must place a pillow under the decreased person’s head and do not affect drug administration.
shoulders to prevent blood from settling in the face and discoloring it. She is Answer A. Primary prevention precedes disease and applies to health patients.
required to bathe only soiled areas of the body since the mortician will wash the Secondary prevention focuses on patients who have health problems and are at
entire body. Before wrapping the body in a shroud, the nurse places a clean risk for developing complications. Tertiary prevention enables patients to gain
gown on the body and closes the eyes and mouth. health from others’ activities without doing anything themselves.
50. A. Ensuring the patient’s safety is the most essential action at this time. Answer A. When making a surgical bed, the nurse leaves the bed in the high
The other nursing actions may be necessary but are not a major priority. position when finished. After placing the top linens on the bed without pouching
them, the nurse fanfolds these linens to the side opposite from where the patient
will enter and places the pillow on the bedside chair. All these actions promote
transfer of the postoperative patient from the stretcher to the bed. When making
Answer B. Sensory deficits could cause a geriatric patient to have difficulty an occupied bed or unoccupied bed, the nurse places the pillow at the head of
retaining knowledge about prescribed medications. Decreased plasma drug the bed and tucks the top sheet and blanket under the bottom of the bed. When
levels do not alter the patient’s knowledge about the drug. A lack of family making an occupied bed, the nurse rolls the patient to the far side of the bed.
support may affect compliance, not knowledge retention. Toilette syndrome is Answer C. The nurse should give ½ ml of the drug. The dosage is calculated as
unrelated to knowledge retention. follows:
Answer C. The nurse should systematically assess all areas of the abdomen, if 250 mg/X=500 mg/1 ml
time and the patient’s condition permit, concluding with the symptomatic area. 500x=250
Otherwise, the nurse may elicit pain in the symptomatic area, causing the X=1/2 ml
muscles in other areas to tighten. This would interfere with further assessment. Answer C. Patients can become dependent on barbiturates, especially with
Answer C. Subjective data come directly from the patient and usually are prolonged use. Because of the rapid distribution of some barbiturates, no
recorded as direct quotations that reflect the patient’s opinions or feelings about correlation exists between duration of action and half-life. Barbiturates are
a situation. Vital signs, laboratory test result, and ECG waveforms are examples absorbed well and do not cause hepatotoxicity, although existing hepatic damage
of objective data. does require cautions use of the drug because barbiturates are metabolized in
Answer C. A safety device on the wrist may impair circulation and restrict blood the liver.
supply to body tissues. Therefore, the nurse should assess the patient for signs Answer A. Elevating the head of the bed during enteral feeding minimizes the
of impaired circulation, such as cool, pale fingers. A palpable radial or lunar pulse risk of aspiration and allows the formula to flow in the patient’s intestines. When
and pink nail beds are normal findings. such elevation is contraindicated, the patient should be positioned on the right
Answer A. Frontal or coronal plane runs longitudinally at a right angle to a sagittal side. The nurse should give enteral feeding at room temperature to minimize GI
plane dividing the body in anterior and posterior regions. A sagittal plane runs distress. To limit microbial growth, the nurse should hang only the amount of
longitudinally dividing the body into right and left regions; if exactly midline, it is formula that can be infused in 3 hours.
called a midsagittal plane. A transverse plane runs horizontally at a right angle to Answer C. The nurse should instruct the patient to touch the tip of the tongue to
the vertical axis, dividing the structure into superior and inferior regions. the roof of the mouth and then place the sublingual tablet on the floor of the
Answer A. Shock and dismay are early signs of denial-the first stage of grief. The mouth. Sublingual medications are absorbed directly into the bloodstream form
other options are associated with depression—a later stage of grief. the oral mucosa, bypassing the GI and hepatic systems. No drug is administered
Answer B. After placing the patient in high Fowler’s position and moving the on top of the tongue or on the roof of the mouth. With the buccal route, the tablet
patient to the side of the bed, the nurse helps the patient sit on the edge of the is placed between the gum and the cheek.
bed and dangle the legs; the nurse then faces the patient and places the chair Answer A. The nurse always should clean around a wound drain, moving from
next to and facing the head of the bed. center outward in ever-larger circles, because the skin near the drain site is more
Answer D. Demonstrating by the nurse with a return demonstration by the patient contaminated than the site itself. The nurse should never remove the drain
ensures that the patient can perform wound care correctly. Patients may claim to before cleaning the skin. Alcohol should never be used to clean around a drain; it
understand discharge instruction when they do not. An interpreter of family may irritate the skin and has no lasting effect on bacteria because it evaporates.
member may communicate verbal or written instructions inaccurately. The nurse should wear sterile gloves to prevent contamination, but a mask is not
Answer A. As a safety precaution, the nurse should discard an unlabeled syringe necessary.
that contains medication. The other options are considered unsafe because they Answer C. Giving 1,000 ml over 8 hours is the same as giving 125 ml over 1 hour
promote error. (60 minutes) to find the number of milliliters per minute:
Answer B. Aging-related physiological changes account for the increased 125/60 min = X/1 minute
frequency of adverse drug reactions in geriatric patients. Renal and hepatic 60X = 125X = 2.1 ml/minute
changes cause drugs to clear more slowly in these patients. With increasing age, To find the number of drops/minute:
neurons are lost and blood flow to the GI tract decreases. 2.1 ml/X gtts = 1 ml/15 gtts
Answer B. When teaching a patient about medications before discharge, the X = 32 gtts/minute, or 32 drops/minute
nurse is acting as an educator. The nurse acts as a manager when performing Answer A. Early in shock, hyperactivity of the sympathetic nervous system
such activities as scheduling and making patient care assignments. The nurse causes increased epinephrine secretion, which typically makes the patient
performs the care giving role when providing direct care, including bathing restless, anxious, nervous, and irritable. It also decreases tissue perfusion to the
patients and administering medications and prescribed treatments. The nurse skin, causing pale, cool clammy skin. An above-normal heart rate is a late sign of
acts as a patient advocate when making the patient’s wishes known to the shock. A urine output of 30 ml/hour is within normal limits.
doctor. Answer D. During a rapid assessment, the nurse’s first priority is to check the
Answer D. Anxiety may result from feeling of helplessness, isolation, or patient’s vital functions by assessing his airway, breathing, and circulation. To
insecurity. This response helps reduce anxiety by encouraging the patient to check a patient’s circulation, the nurse must assess his heart and vascular
express feelings. The nurse should be supportive and develop goals together network function. This is done by checking his skin color, temperature, mental
with the patient to give the patient some control over an anxiety-inducing status and, most importantly, his pulse. The nurse should use the carotid artery
situation. Because the other options ignore the patient’s feeling and block to check a patient’s circulation. In a patient with a circulatory problems or a
communication, they would not reduce anxiety. history of compromised circulation, the radial pulse may not be palpable. The
Answer C. The scrub nurse assist the surgeon by providing appropriate surgical brachial pulse is palpated during rapid assessment of an infant.
instruments and supplies, maintaining strict surgical asepsis and, with the
circulating nurse, accounting for all gauze, sponges, needles, and instruments.
The circulating nurse assists the surgeon and scrub nurse, positions the patient,
applies appropriate equipment and surgical drapes, assists with gowning and
gloving, and provides the surgeon and scrub nurse with supplies.
Answer C. The nurse should return shortly to the patient’s room and remain there
until the patient takes the medication to verify that it was taken as directed. The
NP1- FOUNDATION OF PROFESSIONAL NURSING PRACTICE b. One-to-one nurse patient ratio.
c. Emphasize the use of group collaboration.
1. The nurse In-charge in labor and delivery unit administered a dose of d. Concentrates on tasks and activities.
terbutaline to a client without checking the client’s pulse. The standard that would
be used to determine if the nurse was negligent is: 13.Which type of medication order might read "Vitamin K 10 mg I.M. daily × 3
days?"
a. The physician’s orders.
b. The action of a clinical nurse specialist who is recognized expert in the field. a. Single order b. Standard written order c. Standing order d. Stat order
c. The statement in the drug literature about administration of terbutaline.
d. The actions of a reasonably prudent nurse with similar education and 14.A female client with a fecal impaction frequently exhibits which clinical
experience. manifestation?

2. Nurse Trish is caring for a female client with a history of GI bleeding, sickle cell a. Increased appetite b. Loss of urge to defecate
disease, and a platelet count of 22,000/μl. The female client is dehydrated and c. Hard, brown, formed stools d. Liquid or semi-liquid stools
receiving dextrose 5% in half-normal saline solution at 150 ml/hr. The client
complains of severe bone pain and is scheduled to receive a dose of morphine 15.Nurse Linda prepares to perform an otoscopic examination on a female client.
sulfate. In administering the medication, Nurse Trish should avoid which route? For proper visualization, the nurse should position the client's ear by:
a. I.V b. I.M c. Oral d. S.C
a. Pulling the lobule down and back b. Pulling the helix up and forward
3. Dr. Garcia writes the following order for the client who has been recently c. Pulling the helix up and back d. Pulling the lobule down and forward
admitted “Digoxin .125 mg P.O. once daily.” To prevent a dosage error, how
should the nurse document this order onto the medication administration record? 16. Which instruction should nurse Tom give to a male client who is having
external radiation therapy:
a. “Digoxin .1250 mg P.O. once daily” b. “Digoxin 0.1250 mg P.O. once daily”
c. “Digoxin 0.125 mg P.O. once daily” d. “Digoxin .125 mg P.O. once daily” a. Protect the irritated skin from sunlight.
b. Eat 3 to 4 hours before treatment.
4. A newly admitted female client was diagnosed with deep vein thrombosis. c. Wash the skin over regularly.
Which nursing diagnosis should receive the highest priority? d. Apply lotion or oil to the radiated area when it is red or sore.

a. Ineffective peripheral tissue perfusion related to venous congestion. 17.In assisting a female client for immediate surgery, the nurse In-charge is
b. Risk for injury related to edema. aware that she should:
c. Excess fluid volume related to peripheral vascular disease.
d. Impaired gas exchange related to increased blood flow. a. Encourage the client to void following preoperative medication.
b. Explore the client’s fears and anxieties about the surgery.
5. Nurse Betty is assigned to the following clients. The client that the nurse would c. Assist the client in removing dentures and nail polish.
see first after endorsement? d. Encourage the client to drink water prior to surgery.

a. A 34 year-old post operative appendectomy client of five hours who is 18. A male client is admitted and diagnosed with acute pancreatitis after a
complaining of pain. holiday celebration of excessive food and alcohol. Which assessment finding
b. A 44 year-old myocardial infarction (MI) client who is complaining of nausea. reflects this diagnosis?
c. A 26 year-old client admitted for dehydration whose intravenous (IV) has
infiltrated. a. Blood pressure above normal range.
d. A 63 year-old post operative’s abdominal hysterectomy client of three days b. Presence of crackles in both lung fields.
whose incisional dressing is saturated with serosanguinous fluid. c. Hyperactive bowel sounds
d. Sudden onset of continuous epigastric and back pain.
6. Nurse Gail places a client in a four-point restraint following orders from the
physician. The client care plan should include: 19. Which dietary guidelines are important for nurse Oliver to implement in caring
for the client with burns?
a. Assess temperature frequently. b. Provide diversional activities.
c. Check circulation every 15-30 minutes. d. Socialize with other patients once a a. Provide high-fiber, high-fat diet b. Provide high-protein, high-carbohydrate diet.
shift. c. Monitor intake to prevent weight gain. d. Provide ice chips or water intake.

7. A male client who has severe burns is receiving H2 receptor antagonist 20.Nurse Hazel will administer a unit of whole blood, which priority information
therapy. The nurse In-charge knows the purpose of this therapy is to: should the nurse have about the client?

a. Prevent stress ulcer b. Block prostaglandin synthesis a. Blood pressure and pulse rate. b. Height and weight.
c. Facilitate protein synthesis. d. Enhance gas exchange c. Calcium and potassium levels d. Hgb and Hct levels.

8. The doctor orders hourly urine output measurement for a postoperative male 21. Nurse Michelle witnesses a female client sustain a fall and suspects that the
client. The nurse Trish records the following amounts of output for 2 consecutive leg may be broken. The nurse takes which priority action?
hours: 8 a.m.: 50 ml; 9 a.m.: 60 ml. Based on these amounts, which action
should the nurse take? a. Takes a set of vital signs.
b. Call the radiology department for X-ray.
a. Increase the I.V. fluid infusion rate c. Reassure the client that everything will be alright.
b. Irrigate the indwelling urinary catheter d. Immobilize the leg before moving the client.
c. Notify the physician
d. Continue to monitor and record hourly urine output 22.A male client is being transferred to the nursing unit for admission after
receiving a radium implant for bladder cancer. The nurse in-charge would take
9. Tony, a basketball player twist his right ankle while playing on the court and which priority action in the care of this client?
seeks care for ankle pain and swelling. After the nurse applies ice to the ankle for
30 minutes, which statement by Tony suggests that ice application has been a. Place client on reverse isolation.
effective? b. Admit the client into a private room.
c. Encourage the client to take frequent rest periods.
a. “My ankle looks less swollen now”. b. “My ankle feels warm”. d. Encourage family and friends to visit.
c. “My ankle appears redder now”. d. “I need something stronger for pain relief”
23.A newly admitted female client was diagnosed with agranulocytosis. The
10.The physician prescribes a loop diuretic for a client. When administering this nurse formulates which priority nursing diagnosis?
drug, the nurse anticipates that the client may develop which electrolyte
imbalance? a. Constipation b. Diarrhea c. Risk for infection d. Deficient knowledge

a. Hypernatremia b. Hyperkalemia c. Hypokalemia d. Hypervolemia 24.A male client is receiving total parenteral nutrition suddenly demonstrates
signs and symptoms of an air embolism. What is the priority action by the nurse?
11.She finds out that some managers have benevolent-authoritative style of
management. Which of the following behaviors will she exhibit most likely? a. Notify the physician.
b. Place the client on the left side in the Trendelenburg position.
a. Have condescending trust and confidence in their subordinates. c. Place the client in high-Fowlers position.
b. Gives economic and ego awards. d. Stop the total parenteral nutrition.
c. Communicates downward to staffs.
d. Allows decision making among subordinates. 25.Nurse May attends an educational conference on leadership styles. The nurse
is sitting with a nurse employed at a large trauma center who states that the
12. Nurse Amy is aware that the following is true about functional nursing leadership style at the trauma center is task-oriented and directive. The nurse
determines that the leadership style used at the trauma center is:
a. Provides continuous, coordinated and comprehensive nursing services.
a. Autocratic. b. Laissez-faire. c. Democratic. d. Situational c. It’s the smallest measurement in the apothecary system.
d. It’s a measure of effect, not a standard measure of weight or quantity.
26.The physician orders DS 500 cc with KCl 10 mEq/liter at 30 cc/hr. The nurse
in-charge is going to hang a 500 cc bag. KCl is supplied 20 mEq/10 cc. How 39.Nurse Oliver measures a client’s temperature at 102° F. What is the
many cc’s of KCl will be added to the IV solution? equivalent Centigrade temperature?

a. .5 cc b. 5 cc c. 1.5 cc d. 2.5 cc a. 40.1 °C b. 38.9 °C c. 48 °C d. 38 °C

27.A child of 10 years old is to receive 400 cc of IV fluid in an 8 hour shift. The IV 40.The nurse is assessing a 48-year-old client who has come to the physician’s
drip factor is 60. The IV rate that will deliver this amount is: office for his annual physical exam. One of the first physical
signs of aging is:
a. 50 cc/ hour b. 55 cc/ hour c. 24 cc/ hour d. 66 cc/ hour
a. Accepting limitations while developing assets.
28.The nurse is aware that the most important nursing action when a client b. Increasing loss of muscle tone.
returns from surgery is: c. Failing eyesight, especially close vision.
d. Having more frequent aches and pains.
a. Assess the IV for type of fluid and rate of flow.
b. Assess the client for presence of pain. 41.The physician inserts a chest tube into a female client to treat a
c. Assess the Foley catheter for patency and urine output pneumothorax. The tube is connected to water-seal drainage. The nurse in-
d. Assess the dressing for drainage. charge can prevent chest tube air leaks by:

29. Which of the following vital sign assessments that may indicate cardiogenic a. Checking and taping all connections.
shock after myocardial infarction? b. Checking patency of the chest tube.
c. Keeping the head of the bed slightly elevated.
a. BP – 80/60, Pulse – 110 irregular d. Keeping the chest drainage system below the level of the chest.
b. BP – 90/50, Pulse – 50 regular
c. BP – 130/80, Pulse – 100 regular 42.Nurse Trish must verify the client’s identity before administering medication.
d. BP – 180/100, Pulse – 90 irregular She is aware that the safest way to verify identity is to:

30.Which is the most appropriate nursing action in obtaining a blood pressure a. Check the client’s identification band.
measurement? b. Ask the client to state his name.
c. State the client’s name out loud and wait a client to repeat it.
a. Take the proper equipment, place the client in a comfortable position, and d. Check the room number and the client’s name on the bed.
record the appropriate information in the client’s chart.
b. Measure the client’s arm, if you are not sure of the size of cuff to use. 43.The physician orders dextrose 5 % in water, 1,000 ml to be infused over 8
c. Have the client recline or sit comfortably in a chair with the forearm at the level hours. The I.V. tubing delivers 15 drops/ml. Nurse John should run the I.V.
of the heart. infusion at a rate of:
d. Document the measurement, which extremity was used, and the position that
the client was in during the measurement. a. 30 drops/minute b. 32 drops/minute c. 20 drops/minute d. 18 drops/minute
44.If a central venous catheter becomes disconnected accidentally, what should
31.Asking the questions to determine if the person understands the health the nurse in-charge do immediately?
teaching provided by the nurse would be included during which step of the
nursing process? a. Clamp the catheter b. Call another nurse
c. Call the physician d. Apply a dry sterile dressing to the site.
a. Assessment b. Evaluation c. Implementation d. Planning and goals
45.A female client was recently admitted. She has fever, weight loss, and watery
32.Which of the following item is considered the single most important factor in diarrhea is being admitted to the facility. While assessing the client, Nurse Hazel
assisting the health professional in arriving at a diagnosis or determining the inspects the client’s abdomen and notice that it is slightly concave. Additional
person’s needs? assessment should proceed in which order:

a. Diagnostic test results b. Biographical date a. Palpation, auscultation, and percussion.


c. History of present illness d. Physical examination b. Percussion, palpation, and auscultation.
c. Palpation, percussion, and auscultation.
33.In preventing the development of an external rotation deformity of the hip in a d. Auscultation, percussion, and palpation.
client who must remain in bed for any period of time, the most appropriate
nursing action would be to use: 46. Nurse Betty is assessing tactile fremitus in a client with pneumonia. For this
examination, nurse Betty should use the:
a. Trochanter roll extending from the crest of the ileum to the midthigh.
b. Pillows under the lower legs. a. Fingertips b. Finger pads
c. Footboard c. Dorsal surface of the hand d. Ulnar surface of the hand
d. Hip-abductor pillow
47. Which type of evaluation occurs continuously throughout the teaching and
34.Which stage of pressure ulcer development does the ulcer extend into the learning process?
subcutaneous tissue?
a. Summative b. Informative c. Formative d. Retrospective
a. Stage I b. Stage II c. Stage III d. Stage IV
48.A 45 year old client, has no family history of breast cancer or other risk factors
35.When the method of wound healing is one in which wound edges are not for this disease. Nurse John should instruct her to have
surgically approximated and integumentary continuity is restored by granulations, mammogram how often?
the wound healing is termed
a. Twice per year b. Once per year
a. Second intention healing b. Primary intention healing c. Every 2 years d. Once, to establish baseline
c. Third intention healing d. First intention healing
49.A male client has the following arterial blood gas values: pH 7.30; Pao2 89
36.An 80-year-old male client is admitted to the hospital with a diagnosis of mmHg; Paco2 50 mmHg; and HCO3 26mEq/L. Based on these values, Nurse
pneumonia. Nurse Oliver learns that the client lives alone and hasn’t been eating Patricia should expect which condition?
or drinking. When assessing him for dehydration, nurse Oliver would expect to
find: a. Respiratory acidosis b. Respiratory alkalosis
c. Metabolic acidosis d. Metabolic alkalosis
a. Hypothermia b. Hypertension c. Distended neck veins d. Tachycardia
50.Nurse Len refers a female client with terminal cancer to a local hospice. What
37.The physician prescribes meperidine (Demerol), 75 mg I.M. every 4 hours as is the goal of this referral?
needed, to control a client’s postoperative pain. The package insert is
“Meperidine, 100 mg/ml.” How many milliliters of meperidine should the a. To help the client find appropriate treatment options.
client receive? b. To provide support for the client and family in coping with terminal illness.
c. To ensure that the client gets counseling regarding health care costs.
a. 0.75 b. 0.6 c. 0.5 d. 0.25 d. To teach the client and family about cancer and its treatment.

38. A male client with diabetes mellitus is receiving insulin. Which statement 51.When caring for a male client with a 3-cm stage I pressure ulcer on the
correctly describes an insulin unit? coccyx, which of the following actions can the nurse institute
independently?
a. It’s a common measurement in the metric system.
b. It’s the basis for solids in the avoirdupois system. a. Massaging the area with an astringent every 2 hours.
b. Applying an antibiotic cream to the area three times per day. d. Covers up the incorrect information completely using a black pen and writes in
c. Using normal saline solution to clean the ulcer and applying a protective the correct information
dressing as necessary.
d. Using a povidone-iodine wash on the ulceration three times per day. 63.Nurse Ron is assisting with transferring a client from the operating room table
to a stretcher. To provide safety to the client, the nurse should:
52.Nurse Oliver must apply an elastic bandage to a client’s ankle and calf. He
should apply the bandage beginning at the client’s: a. Moves the client rapidly from the table to the stretcher.
b. Uncovers the client completely before transferring to the stretcher.
a. Knee b. Ankle c. Lower thigh d. Foot c. Secures the client safety belts after transferring to the stretcher.
d. Instructs the client to move self from the table to the stretcher.
53.A 10 year old child with type 1 diabetes develops diabetic ketoacidosis and
receives a continuous insulin infusion. Which condition represents the greatest 64.Nurse Myrna is providing instructions to a nursing assistant assigned to give a
risk to this child? bed bath to a client who is on contact precautions. Nurse Myrna instructs the
nursing assistant to use which of the following protective items when giving bed
a. Hypernatremia b. Hypokalemia c. Hyperphosphatemia d. Hypercalcemia bath?

54.Nurse Len is administering sublingual nitrglycerin (Nitrostat) to the newly a. Gown and goggles b. Gown and gloves
admitted client. Immediately afterward, the client may experience: c. Gloves and shoe protectors d. Gloves and goggles

a. Throbbing headache or dizziness b. Nervousness or paresthesia. 65. Nurse Oliver is caring for a client with impaired mobility that occurred as a
c. Drowsiness or blurred vision. d. Tinnitus or diplopia. result of a stroke. The client has right sided arm and leg weakness. The nurse
would suggest that the client use which of the following assistive devices that
55.Nurse Michelle hears the alarm sound on the telemetry monitor. The nurse would provide the best stability for ambulating?
quickly looks at the monitor and notes that a client is in a ventricular tachycardia.
The nurse rushes to the client’s room. Upon reaching the client’s bedside, the a. Crutches b. Single straight-legged cane
nurse would take which action first? c. Quad cane d. Walker

a. Prepare for cardioversion b. Prepare to defibrillate the client 66.A male client with a right pleural effusion noted on a chest X-ray is being
c. Call a code d. Check the client’s level of consciousness prepared for thoracentesis. The client experiences severe dizziness when sitting
upright. To provide a safe environment, the nurse assists the client to which
56.Nurse Hazel is preparing to ambulate a female client. The best and the safest position for the procedure?
position for the nurse in assisting the client is to stand:
a. Prone with head turned toward the side supported by a pillow.
a. On the unaffected side of the client. b. On the affected side of the client. b. Sims’ position with the head of the bed flat.
c. In front of the client. d. Behind the client. c. Right side-lying with the head of the bed elevated 45 degrees.
d. Left side-lying with the head of the bed elevated 45 degrees.
57.Nurse Janah is monitoring the ongoing care given to the potential organ donor
who has been diagnosed with brain death. The nurse determines that the 67.Nurse John develops methods for data gathering. Which of the following
standard of care had been maintained if which of the following data is observed? criteria of a good instrument refers to the ability of the instrument to yield the
same results upon its repeated administration?
a. Urine output: 45 ml/hr b. Capillary refill: 5 seconds
c. Serum pH: 7.32 d. Blood pressure: 90/48 mmHg a. Validity b. Specificity c. Sensitivity d. Reliability

58. Nurse Amy has an order to obtain a urinalysis from a male client with an 68.Harry knows that he has to protect the rights of human research subjects.
indwelling urinary catheter. The nurse avoids which of the following, which Which of the following actions of Harry ensures anonymity?
contaminate the specimen?
a. Keep the identities of the subject secret
a. Wiping the port with an alcohol swab before inserting the syringe. b. Obtain informed consent
b. Aspirating a sample from the port on the drainage bag. c. Provide equal treatment to all the subjects of the study.
c. Clamping the tubing of the drainage bag. d. Release findings only to the participants of the study
d. Obtaining the specimen from the urinary drainage bag.
69.Patient’s refusal to divulge information is a limitation because it is beyond the
59.Nurse Meredith is in the process of giving a client a bed bath. In the middle of control of Tifanny”. What type of research is appropriate for this study?
the procedure, the unit secretary calls the nurse on the intercom to tell the nurse
that there is an emergency phone call. The appropriate nursing action is to: a. Descriptive- correlational b. Experiment
c. Quasi-experiment d. Historical
a. Immediately walk out of the client’s room and answer the phone call.
b. Cover the client, place the call light within reach, and answer the phone call. 70.Nurse Ronald is aware that the best tool for data gathering is?
c. Finish the bed bath before answering the phone call.
d. Leave the client’s door open so the client can be monitored and the nurse can a. Interview schedule b. Questionnaire
answer the phone call. c. Use of laboratory data d. Observation

60. Nurse Janah is collecting a sputum specimen for culture and sensitivity 71.Monica is aware that there are times when only manipulation of study
testing from a client who has a productive cough. Nurse Janah plans to variables is possible and the elements of control or randomization are not
implement which intervention to obtain the specimen? attendant. Which type of research is referred to this?

a. Ask the client to expectorate a small amount of sputum into the emesis basin. a. Field study b. Quasi-experiment
b. Ask the client to obtain the specimen after breakfast. c. Solomon-Four group design d. Post-test only design
c. Use a sterile plastic container for obtaining the specimen.
d. Provide tissues for expectoration and obtaining the specimen. 72.Cherry notes down ideas that were derived from the description of an
investigation written by the person who conducted it. Which type of reference
61. Nurse Ron is observing a male client using a walker. The nurse determines source refers to this?
that the client is using the walker correctly if the client:
a. Footnote b. Bibliography c. Primary source d. Endnotes
a. Puts all the four points of the walker flat on the floor, puts weight on the hand
pieces, and then walks into it. 73.When Nurse Trish is providing care to his patient, she must remember that
b. Puts weight on the hand pieces, moves the walker forward, and then walks her duty is bound not to do doing any action that will cause the patient harm. This
into it. is the meaning of the bioethical principle:
c. Puts weight on the hand pieces, slides the walker forward, and then walks into
it. a. Non-maleficence b. Beneficence c. Justice d. Solidarity
d. Walks into the walker, puts weight on the hand pieces, and then puts all four
points of the walker flat on the floor. 74.When a nurse in-charge causes an injury to a female patient and the injury
caused becomes the proof of the negligent act, the presence of the injury is said
62.Nurse Amy has documented an entry regarding client care in the client’s to exemplify the principle of:
medical record. When checking the entry, the nurse realizes that incorrect
information was documented. How does the nurse correct this error? a. Force majeure b. Respondeat superior
c. Res ipsa loquitor d. Holdover doctrine
a. Erases the error and writes in the correct information.
b. Uses correction fluid to cover up the incorrect information and writes in the 75.Nurse Myrna is aware that the Board of Nursing has quasi-judicial power. An
correct information. example of this power is:
c. Draws one line to cross out the incorrect information and then initials the
change. a. The Board can issue rules and regulations that will govern the practice of
nursing
b. The Board can investigate violations of the nursing law and code of ethics a. Arrange for typing and cross matching of the client’s blood.
c. The Board can visit a school applying for a permit in collaboration with CHED b. Compare the client’s identification wristband with the tag on the unit of blood.
d. The Board prepares the board examinations c. Start an I.V. infusion of normal saline solution.
d. Measure the client’s vital signs.
76. When the license of nurse Krina is revoked, it means that she:
90.A 65 years old male client requests his medication at 9 p.m. instead of 10
a. Is no longer allowed to practice the profession for the rest of her life p.m. so that he can go to sleep earlier. Which type of nursing intervention is
b. Will never have her/his license re-issued since it has been revoked required?
c. May apply for re-issuance of his/her license based on certain conditions
stipulated in RA 9173 a. Independent b. Dependent c. Interdependent d. Intradependent
d. Will remain unable to practice professional nursing
91.A female client is to be discharged from an acute care facility after treatment
77.Ronald plans to conduct a research on the use of a new method of pain for right leg thrombophlebitis. The Nurse Betty notes that the client's leg is pain-
assessment scale. Which of the following is the second step in the free, without redness or edema. The nurse's actions reflect which step of the
conceptualizing phase of the research process? nursing process?

a. Formulating the research hypothesis a. Assessment b. Diagnosis c. Implementation d. Evaluation


b. Review related literature
c. Formulating and delimiting the research problem 92.Nursing care for a female client includes removing elastic stockings once per
d. Design the theoretical and conceptual framework day. The Nurse Betty is aware that the rationale for this intervention?

78. The leader of the study knows that certain patients who are in a specialized a. To increase blood flow to the heart
research setting tend to respond psychologically to the conditions of the study. b. To observe the lower extremities
This referred to as : c. To allow the leg muscles to stretch and relax
d. To permit veins in the legs to fill with blood.
a. Cause and effect b. Hawthorne effect
c. Halo effect d. Horns effect 93.Which nursing intervention takes highest priority when caring for a newly
admitted client who's receiving a blood transfusion?
79.Mary finally decides to use judgment sampling on her research. Which of the
following actions of is correct? a. Instructing the client to report any itching, swelling, or dyspnea.
b. Informing the client that the transfusion usually take 1 ½ to 2 hours.
a. Plans to include whoever is there during his study. c. Documenting blood administration in the client care record.
b. Determines the different nationality of patients frequently admitted and decides d. Assessing the client’s vital signs when the transfusion ends.
to get representations samples from each.
c. Assigns numbers for each of the patients, place these in a fishbowl and draw 94.A male client complains of abdominal discomfort and nausea while receiving
10 from it. tube feedings. Which intervention is most appropriate for this problem?
d. Decides to get 20 samples from the admitted patients
a. Give the feedings at room temperature.
80. The nursing theorist who developed transcultural nursing theory is: b. Decrease the rate of feedings and the concentration of the formula.
c. Place the client in semi-Fowler's position while feeding.
a. Florence Nightingale b. Madeleine Leininger d. Change the feeding container every 12 hours.
c. Albert Moore d. Sr. Callista Roy
95.Nurse Patricia is reconstituting a powdered medication in a vial. After adding
81.Marion is aware that the sampling method that gives equal chance to all units the solution to the powder, she nurse should:
in the population to get picked is:
a. Do nothing.
a. Random b. Accidental c. Quota d. Judgment b. Invert the vial and let it stand for 3 to 5 minutes.
c. Shake the vial vigorously.
82.John plans to use a Likert Scale to his study to determine the: d. Roll the vial gently between the palms.

a. Degree of agreement and disagreement 96.Which intervention should the nurse Trish use when administering oxygen by
b. Compliance to expected standards face mask to a female client?
c. Level of satisfaction
d. Degree of acceptance a. Secure the elastic band tightly around the client's head.
b. Assist the client to the semi-Fowler position if possible.
83.Which of the following theory addresses the four modes of adaptation? c. Apply the face mask from the client's chin up over the nose.
d. Loosen the connectors between the oxygen equipment and humidifier.
a. Madeleine Leininger b. Sr. Callista Roy
c. Florence Nightingale d. Jean Watson 97.The maximum transfusion time for a unit of packed red blood cells (RBCs) is:

84.Ms. Garcia is responsible to the number of personnel reporting to her. This a. 6 hours b. 4 hours c. 3 hours d. 2 hours
principle refers to:
98.Nurse Monique is monitoring the effectiveness of a client's drug therapy.
a. Span of control b. Unity of command When should the nurse Monique obtain a blood sample to measure the trough
c. Downward communication d. Leader drug level?

85.Ensuring that there is an informed consent on the part of the patient before a a. 1 hour before administering the next dose.
surgery is done, illustrates the bioethical principle of: b. Immediately before administering the next dose.
c. Immediately after administering the next dose.
a. Beneficence b. Autonomy c. Veracity d. Non-maleficence d. 30 minutes after administering the next dose.

86.Nurse Reese is teaching a female client with peripheral vascular disease 99.Nurse May is aware that the main advantage of using a floor stock system is:
about foot care; Nurse Reese should include which instruction?
a. The nurse can implement medication orders quickly.
a. Avoid wearing cotton socks. b. The nurse receives input from the pharmacist.
b. Avoid using a nail clipper to cut toenails. c. The system minimizes transcription errors.
c. Avoid wearing canvas shoes. d. The system reinforces accurate calculations.
d. Avoid using cornstarch on feet.
100. Nurse Oliver is assessing a client's abdomen. Which finding should the
87.A client is admitted with multiple pressure ulcers. When developing the client's nurse report as abnormal?
diet plan, the nurse should include:
a. Dullness over the liver.
a. Fresh orange slices b. Steamed broccoli b. Bowel sounds occurring every 10 seconds.
c. Ice cream d. Ground beef patties c. Shifting dullness over the abdomen.
d. Vascular sounds heard over the renal arteries
88.The nurse prepares to administer a cleansing enema. What is the most RATIO 
common client position used for this procedure? 1. Answer: (D) The actions of a reasonably prudent nurse with similar education
and experience.
a. Lithotomy b. Supine c. Prone d. Sims’ left lateral Rationale: The standard of care is determined by the average degree of skill,
care, and diligence by nurses in similar circumstances.
89.Nurse Marian is preparing to administer a blood transfusion. Which action
should the nurse take first? 2. Answer: (B) I.M
Rationale: With a platelet count of 22,000/μl, the clients tends to bleed easily. Rationale: The autodigestion of tissue by the pancreatic enzymes results in pain
Therefore, the nurse should avoid using the I.M. route because the area is a from inflammation, edema, and possible hemorrhage. Continuous, unrelieved
highly vascular and can bleed readily when penetrated by a needle. The bleeding epigastric or back pain reflects the inflammatory process in the pancreas.
can be difficult to stop.
19. Answer: (B) Provide high-protein, high-carbohydrate diet.
3. Answer: (C) “Digoxin 0.125 mg P.O. once daily” Rationale: A positive nitrogen balance is important for meeting metabolic needs,
Rationale: The nurse should always place a zero before a decimal point so that tissue repair, and resistance to infection. Caloric goals may be as high as 5000
no one misreads the figure, which could result in a dosage error. The nurse calories per day.
should never insert a zero at the end of a dosage that includes a decimal point
because this could be misread, possibly leading to a tenfold increase in the 20. Answer: (A) Blood pressure and pulse rate.
dosage. Rationale: The baseline must be established to recognize the signs of an
anaphylactic or hemolytic reaction to the transfusion.
4. Answer: (A) Ineffective peripheral tissue perfusion related to venous
congestion. 21. Answer: (D) Immobilize the leg before moving the client.
Rationale: Ineffective peripheral tissue perfusion related to venous congestion Rationale: If the nurse suspects a fracture, splinting the area before moving the
takes the highest priority because venous inflammation and clot formation client is imperative. The nurse should call for emergency help if the client is not
impede blood flow in a client with deep vein thrombosis. hospitalized and call for a physician for the hospitalized client.

5. Answer: (B) A 44 year-old myocardial infarction (MI) client who is complaining 22. Answer: (B) Admit the client into a private room.
of nausea. Rationale: The client who has a radiation implant is placed in a private room and
Rationale: Nausea is a symptom of impending myocardial infarction (MI) and has a limited number of visitors. This reduces the exposure of others to the
should be assessed immediately so that treatment can be instituted and further radiation.
damage to the heart is avoided.
23. Answer: (C) Risk for infection
6. Answer: (C) Check circulation every 15-30 minutes. Rationale: Agranulocytosis is characterized by a reduced number of leukocytes
Rationale: Restraints encircle the limbs, which place the client at risk for (leucopenia) and neutrophils (neutropenia) in the blood. The client is at high risk
circulation being restricted to the distal areas of the extremities. Checking the for infection because of the decreased body defenses against microorganisms.
client’s circulation every 15-30 minutes will allow the nurse to adjust the restraints Deficient knowledge related to the nature of the disorder may be appropriate
before injury from decreased blood flow occurs. diagnosis but is not the priority.

7. Answer: (A) Prevent stress ulcer 24. Answer: (B) Place the client on the left side in the Trendelenburg position.
Rationale: Curling’s ulcer occurs as a generalized stress response in burn Rationale: Lying on the left side may prevent air from flowing into the pulmonary
patients. This results in a decreased production of mucus and increased veins. The Trendelenburg position increases intrathoracic pressure, which
secretion of gastric acid. The best treatment for this prophylactic use of antacids decreases the amount of blood pulled into the vena cava during aspiration.
and H2 receptor blockers.
25. Answer: (A) Autocratic.
8. Answer: (D) Continue to monitor and record hourly urine output Rationale: The autocratic style of leadership is a task-oriented and directive.
Rationale: Normal urine output for an adult is approximately 1 ml/minute (60
ml/hour). Therefore, this client's output is normal. Beyond continued evaluation, 26. Answer: (D) 2.5 cc
no nursing action is warranted. Rationale: 2.5 cc is to be added, because only a 500 cc bag of solution is being
medicated instead of a 1 liter.
9. Answer: (B) “My ankle feels warm”.
Rationale: Ice application decreases pain and swelling. Continued or increased 27. Answer: (A) 50 cc/ hour
pain, redness, and increased warmth are signs of inflammation that shouldn't Rationale: A rate of 50 cc/hr. The child is to receive 400 cc over a period of 8
occur after ice application hours = 50 cc/hr.

10. Answer: (B) Hyperkalemia 28. Answer: (B) Assess the client for presence of pain.
Rationale: A loop diuretic removes water and, along with it, sodium and Rationale: Assessing the client for pain is a very important measure.
potassium. This may result in hypokalemia, hypovolemia, and hyponatremia. Postoperative pain is an indication of complication. The nurse should also assess
the client for pain to provide for the client’s comfort.
11. Answer:(A) Have condescending trust and confidence in their subordinates
Rationale: Benevolent-authoritative managers pretentiously show their trust and 29. Answer: (A) BP – 80/60, Pulse – 110 irregular
confidence to their followers. Rationale: The classic signs of cardiogenic shock are low blood pressure, rapid
and weak irregular pulse, cold, clammy skin, decreased urinary output, and
12. Answer: (A) Provides continuous, coordinated and comprehensive nursing cerebral hypoxia.
services.
Rationale: Functional nursing is focused on tasks and activities and not on the 30. Answer: (A) Take the proper equipment, place the client in a comfortable
care of the patients. position, and record the appropriate information in the client’s chart.
Rationale: It is a general or comprehensive statement about the correct
13. Answer: (B) Standard written order procedure, and it includes the basic ideas which are found in the other options
Rationale: This is a standard written order. Prescribers write a single order for
medications given only once. A stat order is written for 31. Answer: (B) Evaluation
medications given immediately for an urgent client problem. A standing order, Rationale: Evaluation includes observing the person, asking questions, and
also known as a protocol, establishes guidelines for treating a comparing the patient’s behavioral responses with the expected outcomes.
particular disease or set of symptoms in special care areas such as the coronary
care unit. Facilities also may institute medication protocols that specifically 32. Answer: (C) History of present illness
designate drugs that a nurse may not give. Rationale: The history of present illness is the single most important factor in
assisting the health professional in arriving at a diagnosis or determining the
14. Answer: (D) Liquid or semi-liquid stools person’s needs.
Rationale: Passage of liquid or semi-liquid stools results from seepage of
unformed bowel contents around the impacted stool in the rectum. Clients 33. Answer: (A) Trochanter roll extending from the crest of the ileum to the mid-
with fecal impaction don't pass hard, brown, formed stools because the feces thigh.
can't move past the impaction. These clients typically report the urge Rationale: A trochanter roll, properly placed, provides resistance to the external
to defecate (although they can't pass stool) and a decreased appetite. rotation of the hip.

15. Answer: (C) Pulling the helix up and back 34. Answer: (C) Stage III
Rationale: To perform an otoscopic examination on an adult, the nurse grasps Rationale: Clinically, a deep crater or without undermining of adjacent tissue is
the helix of the ear and pulls it up and back to straighten the ear canal. For a noted.
child, the nurse grasps the helix and pulls it down to straighten the ear canal.
Pulling the lobule in any direction wouldn't straighten the ear canal for 35. Answer: (A) Second intention healing
visualization. Rationale: When wounds dehisce, they will allowed to heal by secondary
intention
16. Answer: (A) Protect the irritated skin from sunlight.
Rationale: Irradiated skin is very sensitive and must be protected with clothing or 36. Answer: (D) Tachycardia
sunblock. The priority approach is the avoidance of strong sunlight. Rationale: With an extracellular fluid or plasma volume deficit, compensatory
mechanisms stimulate the heart, causing an increase in heart rate.
17. Answer: (C) Assist the client in removing dentures and nail polish.
Rationale: Dentures, hairpins, and combs must be removed. Nail polish must be 37. Answer: (A) 0.75
removed so that cyanosis can be easily monitored by observing the nail beds. Rationale: To determine the number of milliliters the client should receive, the
nurse uses the fraction method in the following equation.
18. Answer: (D) Sudden onset of continuous epigastric and back pain. 75 mg/X ml = 100 mg/1 ml
To solve for X, cross-multiply:
75 mg x 1 ml = X ml x 100 mg
75 = 100X 51. Answer: (C) Using normal saline solution to clean the ulcer and applying a
75/100 = X protective dressing as necessary.
0.75 ml (or ¾ ml) = X Rationale: Washing the area with normal saline solution and applying a
protective dressing are within the nurse’s realm of interventions and will protect
38. Answer: (D) It’s a measure of effect, not a standard measure of weight or the area. Using a povidone-iodine wash and an antibiotic cream require a
quantity. physician’s order. Massaging with an astringent can further damage the skin.
Rationale: An insulin unit is a measure of effect, not a standard measure of
weight or quantity. Different drugs measured in units may have no relationship to 52. Answer: (D) Foot
one another in quality or quantity. Rationale: An elastic bandage should be applied form the distal area to the
proximal area. This method promotes venous return. In this case, the nurse
39. Answer: (B) 38.9 °C should begin applying the bandage at the client’s foot. Beginning at the ankle,
Rationale: To convert Fahrenheit degreed to Centigrade, use this formula lower thigh, or knee does not promote venous return.
°C = (°F – 32) ÷ 1.8
°C = (102 – 32) ÷ 1.8 53. Answer: (B) Hypokalemia
°C = 70 ÷ 1.8 Rationale: Insulin administration causes glucose and potassium to move into the
°C = 38.9 cells, causing hypokalemia.

40. Answer: (C) Failing eyesight, especially close vision. 54. Answer: (A) Throbbing headache or dizziness
Rationale: Failing eyesight, especially close vision, is one of the first signs of Rationale: Headache and dizziness often occur when nitroglycerin is taken at the
aging in middle life (ages 46 to 64). More frequent aches and pains begin in the beginning of therapy. However, the client usually develops tolerance
early late years (ages 65 to 79). Increase in loss of muscle tone occurs in later
years (age 80 and older). 55. Answer: (D) Check the client’s level of consciousness
Rationale: Determining unresponsiveness is the first step assessment action to
41. Answer: (A) Checking and taping all connections take. When a client is in ventricular tachycardia, there is a significant decrease in
Rationale: Air leaks commonly occur if the system isn’t secure. Checking all cardiac output. However, checking the unresponsiveness ensures whether the
connections and taping them will prevent air leaks. The chest drainage system is client is affected by the decreased cardiac output.
kept lower to promote drainage – not to prevent leaks.
56. Answer: (B) On the affected side of the client.
42. Answer: (A) Check the client’s identification band. Rationale: When walking with clients, the nurse should stand on the affected side
Rationale: Checking the client’s identification band is the safest way to verify a and grasp the security belt in the midspine area of the small of the back. The
client’s identity because the band is assigned on admission and isn’t be removed nurse should position the free hand at the shoulder area so that the client can be
at any time. (If it is removed, it must be replaced). Asking the client’s name or pulled toward the nurse in the event that there is a forward fall. The client is
having the client repeated his name would be appropriate only for a client who’s instructed to look up and outward rather than at his or her feet.
alert, oriented, and able to understand what is being said, but isn’t the safe
standard of practice. Names on bed aren’t always reliable 57. Answer: (A) Urine output: 45 ml/hr
Rationale: Adequate perfusion must be maintained to all vital organs in order for
43. Answer: (B) 32 drops/minute the client to remain visible as an organ donor. A urine output of 45 ml per hour
Rationale: Giving 1,000 ml over 8 hours is the same as giving 125 ml over 1 hour indicates adequate renal perfusion. Low blood pressure and delayed capillary
(60 minutes). Find the number of milliliters per minute as follows: refill time are circulatory system indicators of inadequate perfusion. A serum pH
125/60 minutes = X/1 minute of 7.32 is acidotic, which adversely affects all body tissues.
60X = 125 = 2.1 ml/minute
To find the number of drops per minute: 58. Answer: (D ) Obtaining the specimen from the urinary drainage bag.
2.1 ml/X gtt = 1 ml/ 15 gtt Rationale: A urine specimen is not taken from the urinary drainage bag. Urine
X = 32 gtt/minute, or 32 drops/minute undergoes chemical changes while sitting in the bag and does not necessarily
reflect the current client status. In addition, it may become contaminated with
44. Answer: (A) Clamp the catheter bacteria from opening the system.
Rationale: If a central venous catheter becomes disconnected, the nurse should
immediately apply a catheter clamp, if available. If a clamp isn’t available, the 59. Answer: (B) Cover the client, place the call light within reach, and answer the
nurse can place a sterile syringe or catheter plug in the catheter hub. After phone call.
cleaning the hub with alcohol or povidone-iodine solution, the nurse must replace Rationale: Because telephone call is an emergency, the nurse may need to
the I.V. extension and restart the infusion. answer it. The other appropriate action is to ask another nurse to accept the call.
However, is not one of the options. To maintain privacy and safety, the nurse
45. Answer: (D) Auscultation, percussion, and palpation. covers the client and places the call light within the client’s reach. Additionally,
Rationale: The correct order of assessment for examining the abdomen is the client’s door should be closed or the room curtains pulled around the bathing
inspection, auscultation, percussion, and palpation. The reason for this approach area.
is that the less intrusive techniques should be performed before the more
intrusive techniques. Percussion and palpation can alter natural findings during 60. Answer: (C) Use a sterile plastic container for obtaining the specimen.
auscultation. Rationale: Sputum specimens for culture and sensitivity testing need to be
obtained using sterile techniques because the test is done to determine the
46. Answer: (D) Ulnar surface of the hand presence of organisms. If the procedure for obtaining the specimen is not sterile,
Rationale: The nurse uses the ulnar surface, or ball, of the hand to asses tactile then the specimen is not sterile, then the specimen would be contaminated and
fremitus, thrills, and vocal vibrations through the chest wall. The the results of the test would be invalid.
fingertips and finger pads best distinguish texture and shape. The dorsal surface
best feels warmth. 61. Answer: (A) Puts all the four points of the walker flat on the floor, puts weight
on the hand pieces, and then walks into it.
47. Answer: (C) Formative Rationale: When the client uses a walker, the nurse stands adjacent to the
Rationale: Formative (or concurrent) evaluation occurs continuously throughout affected side. The client is instructed to put all four points of the walker 2 feet
the teaching and learning process. One benefit is that the nurse can adjust forward flat on the floor before putting weight on hand pieces. This will ensure
teaching strategies as necessary to enhance learning. Summative, or client safety and prevent stress cracks in the walker. The client is then instructed
retrospective, evaluation occurs at the conclusion of the teaching and learning to move the walker forward and walk into it.
session. Informative is not a type of evaluation.
62. Answer: (C) Draws one line to cross out the incorrect information and then
48. Answer: (B) Once per year initials the change.
Rationale: Yearly mammograms should begin at age 40 and continue for Rationale: To correct an error documented in a medical record, the nurse draws
as long as the woman is in good health. If health risks, such as family one line through the incorrect information and then initials the error. An error is
history, genetic tendency, or past breast cancer, exist, more frequent never erased and correction fluid is never used in the medical record.
examinations may be necessary.
63. Answer: (C) Secures the client safety belts after transferring to the stretcher.
49. Answer: (A) Respiratory acidosis Rationale: During the transfer of the client after the surgical procedure is
Rationale: The client has a below-normal (acidic) blood pH value and an above- complete, the nurse should avoid exposure of the client because of the risk for
normal partial pressure of arterial carbon dioxide (Paco2) value, indicating potential heat loss. Hurried movements and rapid changes in the position should
respiratory acidosis. In respiratory alkalosis, the pH value is above normal and in be avoided because these predispose the client to hypotension. At the time of
the Paco2 value is below normal. In metabolic acidosis, the pH and bicarbonate the transfer from the surgery table to the stretcher, the client is still affected by
(Hco3) values are below normal. In metabolic alkalosis, the pH and Hco3 values the effects of the anesthesia; therefore, the client should not move self. Safety
are above normal. belts can prevent the client from falling off the stretcher.

50. Answer: (B) To provide support for the client and family in coping with 64. Answer: (B) Gown and gloves
terminal illness. Rationale: Contact precautions require the use of gloves and a gown if direct
Rationale: Hospices provide supportive care for terminally ill clients and their client contact is anticipated. Goggles are not necessary unless the
families. Hospice care doesn’t focus on counseling regarding health care costs. nurse anticipates the splashes of blood, body fluids, secretions, or excretions
Most client referred to hospices have been treated for their disease without may occur. Shoe protectors are not necessary.
success and will receive only palliative care in the hospice.
65. Answer: (C) Quad cane
Rationale: Crutches and a walker can be difficult to maneuver for a client with Rationale: Sr. Callista Roy developed the Adaptation Model which involves the
weakness on one side. A cane is better suited for client with weakness of the arm physiologic mode, self-concept mode, role function mode and dependence
and leg on one side. However, the quad cane would provide the most stability mode.
because of the structure of the cane and because a quad cane has four legs.
84. Answer: (A) Span of control
66. Answer: (D) Left side-lying with the head of the bed elevated 45 degrees. Rationale: Span of control refers to the number of workers who report directly to
Rationale: To facilitate removal of fluid from the chest wall, the client is positioned a manager.
sitting at the edge of the bed leaning over the bedside table with the feet
supported on a stool. If the client is unable to sit up, the client is positioned lying 85. Answer: (B) Autonomy
in bed on the unaffected side with the head of the bed elevated 30 to 45 degrees. Rationale: Informed consent means that the patient fully understands about the
surgery, including the risks involved and the alternative solutions. In giving
67. Answer: (D) Reliability consent it is done with full knowledge and is given freely. The action of allowing
Rationale: Reliability is consistency of the research instrument. It refers to the patient to decide whether a surgery is to be done or not exemplifies the
the repeatability of the instrument in extracting the same responses upon bioethical principle of autonomy.
its repeated administration.
86. Answer: (C) Avoid wearing canvas shoes.
68. Answer: (A) Keep the identities of the subject secret Rationale: The client should be instructed to avoid wearing canvas shoes.
Rationale: Keeping the identities of the research subject secret will ensure Canvas shoes cause the feet to perspire, which may, in turn, cause skin
anonymity because this will hinder providing link between the information given irritation and breakdown. Both cotton and cornstarch absorb perspiration. The
to whoever is its source. client should be instructed to cut toenails straight across with nail
clippers.
69. Answer: (A) Descriptive- correlational
Rationale: Descriptive- correlational study is the most appropriate for this study 87. Answer: (D) Ground beef patties
because it studies the variables that could be the antecedents of the increased Rationale: Meat is an excellent source of complete protein, which this client
incidence of nosocomial infection. needs to repair the tissue breakdown caused by pressure ulcers.
Oranges and broccoli supply vitamin C but not protein. Ice cream supplies only
70. Answer: (C) Use of laboratory data some incomplete protein, making it less helpful in tissue repair.
Rationale: Incidence of nosocomial infection is best collected through the use of
biophysiologic measures, particularly in vitro measurements, hence laboratory 88. Answer: (D) Sims’ left lateral
data is essential. Rationale: The Sims' left lateral position is the most common position used to
administer a cleansing enema because it allows gravity to aid the flow of fluid
71. Answer: (B) Quasi-experiment along the curve of the sigmoid colon. If the client can't assume this position nor
Rationale: Quasi-experiment is done when randomization and control of the has poor sphincter control, the dorsal recumbent or right lateral position may be
variables are not possible. used. The supine and prone positions are inappropriate and uncomfortable for
the client.
72. Answer: (C) Primary source
Rationale: This refers to a primary source which is a direct account of the 89. Answer: (A) Arrange for typing and cross matching of the client’s blood.
investigation done by the investigator. In contrast to this is a secondary source, Rationale: The nurse first arranges for typing and cross matching of the client's
which is written by someone other than the original researcher. blood to ensure compatibility with donor blood. The other options,
although appropriate when preparing to administer a blood transfusion, come
73. Answer: (A) Non-maleficence later.
Rationale: Non-maleficence means do not cause harm or do any action that will
cause any harm to the patient/client. To do good is referred as beneficence. 90. Answer: (A) Independent
Rationale: Nursing interventions are classified as independent, interdependent,
74. Answer: (C) Res ipsa loquitor or dependent. Altering the drug schedule to coincide with the client's daily routine
Rationale: Res ipsa loquitor literally means the thing speaks for itself. This means represents an independent intervention, whereas consulting with the physician
in operational terms that the injury caused is the proof that there was a negligent and pharmacist to change a client's medication because of adverse reactions
act. represents an interdependent intervention. Administering an already-prescribed
drug on time is a dependent intervention. An intradependent nursing intervention
75. Answer: (B) The Board can investigate violations of the nursing law and code doesn't exist.
of ethics
Rationale: Quasi-judicial power means that the Board of Nursing has the 91. Answer: (D) Evaluation
authority to investigate violations of the nursing law and can issue summons, Rationale: The nursing actions described constitute evaluation of the expected
subpoena or subpoena duces tecum as needed. outcomes. The findings show that the expected outcomes have been achieved.
Assessment consists of the client's history, physical examination, and laboratory
76. Answer: (C) May apply for re-issuance of his/her license based on certain studies. Analysis consists of considering assessment information to derive the
conditions stipulated in RA 9173 appropriate nursing diagnosis. Implementation is the phase of the nursing
Rationale: RA 9173 sec. 24 states that for equity and justice, a revoked license process where the nurse puts the plan of care into action.
maybe re-issued provided that the following conditions are met: a)
the cause for revocation of license has already been corrected or removed; and, 92. Answer: (B) To observe the lower extremities
b) at least four years has elapsed since the license has been revoked. Rationale: Elastic stockings are used to promote venous return. The nurse needs
to remove them once per day to observe the condition of the skin underneath the
77. Answer: (B) Review related literature stockings. Applying the stockings increases blood flow to the heart. When the
Rationale: After formulating and delimiting the research problem, the researcher stockings are in place, the leg muscles can still stretch and relax, and the veins
conducts a review of related literature to determine the extent of what has been can fill with blood.
done on the study by previous researchers.
93. Answer:(A) Instructing the client to report any itching, swelling, or dyspnea.
78. Answer: (B) Hawthorne effect Rationale: Because administration of blood or blood products may cause serious
Rationale: Hawthorne effect is based on the study of Elton Mayo and company adverse effects such as allergic reactions, the nurse must monitor the client for
about the effect of an intervention done to improve the working conditions of the these effects. Signs and symptoms of life-threatening allergic reactions include
workers on their productivity. It resulted to an increased productivity but not due itching, swelling, and dyspnea. Although the nurse should inform the client of the
to the intervention but due to the psychological effects of being observed. They duration of the transfusion and should document its administration, these actions
performed differently because they were under observation. are less critical to the client's immediate health. The nurse should assess vital
signs at least hourly during the transfusion.
79. Answer: (B) Determines the different nationality of patients frequently
admitted and decides to get representations samples from each. 94. Answer: (B) Decrease the rate of feedings and the concentration of the
Rationale: Judgment sampling involves including samples according to the formula.
knowledge of the investigator about the participants in the study. Rationale: Complaints of abdominal discomfort and nausea are common in
clients receiving tube feedings. Decreasing the rate of the feeding and the
80. Answer: (B) Madeleine Leininger concentration of the formula should decrease the client's discomfort. Feedings
Rationale: Madeleine Leininger developed the theory on transcultural theory are normally given at room temperature to minimize abdominal cramping. To
based on her observations on the behavior of selected people within a culture. prevent aspiration during feeding, the head of the client's bed should be elevated
at least 30 degrees. Also, to prevent bacterial growth, feeding containers should
81. Answer: (A) Random be routinely changed every 8 to 12 hours.
Rationale: Random sampling gives equal chance for all the elements in the
population to be picked as part of the sample. 95. Answer: (D) Roll the vial gently between the palms.
Rationale: Rolling the vial gently between the palms produces heat, which helps
82. Answer: (A) Degree of agreement and disagreement dissolve the medication. Doing nothing or inverting the vial wouldn't help dissolve
Rationale: Likert scale is a 5-point summated scale used to determine the degree the medication. Shaking the vial vigorously could cause the medication to break
of agreement or disagreement of the respondents to a statement in a study down, altering its action.

83. Answer: (B) Sr. Callista Roy


96. Answer: (B) Assist the client to the semi-Fowler position if possible. a. Contractions every 1 ½ minutes lasting 70-80 seconds.
Rationale: By assisting the client to the semi-Fowler position, the nurse promotes b. Maternal temperature 101.2
easier chest expansion, breathing, and oxygen intake. The nurse should secure c. Early decelerations in the fetal heart rate.
the elastic band so that the face mask fits comfortably and snugly rather than d. Fetal heart rate baseline 140-160 bpm.
tightly, which could lead to irritation. The nurse should apply the face mask from
the client's nose down to the chin — not vice versa. The nurse should check the 9. Calcium gluconate is being administered to a client with pregnancy induced
connectors between the oxygen equipment and humidifier to ensure that they're hypertension (PIH). A nursing action that must be initiated as the plan of care
airtight; loosened connectors can cause loss of oxygen. throughout injection of the drug is:

97. Answer: (B) 4 hours a. Ventilator assistance b. CVP readings c. EKG tracings d. Continuous CPR
Rationale: A unit of packed RBCs may be given over a period of between 1 and 4
hours. It shouldn't infuse for longer than 4 hours because the risk of 10. A trial for vaginal delivery after an earlier caesareans, would likely to be given
contamination and sepsis increases after that time. Discard or return to the blood to a gravida, who had:
bank any blood not given within this time, according to facility policy.
a. First low transverse cesarean was for active herpes type 2 infections; vaginal
98. Answer: (B) Immediately before administering the next dose. culture at 39 weeks pregnancy was positive.
Rationale: Measuring the blood drug concentration helps determine whether the b. First and second caesareans were for cephalopelvic disproportion.
dosing has achieved the therapeutic goal. For measurement of the trough, or c. First caesarean through a classic incision as a result of severe fetal distress.
lowest, blood level of a drug, the nurse draws a blood sample immediately before d. First low transverse caesarean was for breech position. Fetus in this
administering the next dose. Depending on the drug's duration of action and half- pregnancy is in a vertex presentation.
life, peak blood drug levels typically are drawn after administering the next dose.
11.Nurse Ryan is aware that the best initial approach when trying to take a crying
99. Answer: (A) The nurse can implement medication orders quickly. toddler’s temperature is:
Rationale: A floor stock system enables the nurse to implement medication
orders quickly. It doesn't allow for pharmacist input, nor does it minimize a. Talk to the mother first and then to the toddler.
transcription errors or reinforce accurate calculations. b. Bring extra help so it can be done quickly.
c. Encourage the mother to hold the child.
100. Answer: (C) Shifting dullness over the abdomen. d. Ignore the crying and screaming.
Rationale: Shifting dullness over the abdomen indicates ascites, an abnormal
finding. The other options are normal abdominal findings. 12.Baby Tina a 3 month old infant just had a cleft lip and palate repair. What
should the nurse do to prevent trauma to operative site?
NP2- MCN AND CHN
a. Avoid touching the suture line, even when cleaning.
1. May arrives at the health care clinic and tells the nurse that her last menstrual b. Place the baby in prone position.
period was 9 weeks ago. She also tells the nurse that a home pregnancy test c. Give the baby a pacifier.
was positive but she began to have mild cramps and is now having moderate d. Place the infant’s arms in soft elbow restraints.
vaginal bleeding. During the physical examination of the client, the nurse notes
that May has a dilated cervix. The nurse determines that May is experiencing 13. Which action should nurse Marian include in the care plan for a 2 month old
which type of abortion? with heart failure?

a. Inevitable b. Incomplete c. Threatened d. Septic a. Feed the infant when he cries.


b. Allow the infant to rest before feeding.
2. Nurse Reese is reviewing the record of a pregnant client for her first prenatal c. Bathe the infant and administer medications before feeding.
visit. Which of the following data, if noted on the client’s record, would alert the d. Weigh and bathe the infant before feeding.
nurse that the client is at risk for a spontaneous abortion?
14.Nurse Hazel is teaching a mother who plans to discontinue breast feeding
a. Age 36 years b. History of syphilis after 5 months. The nurse should advise her to include which foods in her infant’s
c. History of genital herpes d. History of diabetes mellitus diet?

3. Nurse Hazel is preparing to care for a client who is newly admitted to the a. Skim milk and baby food. b. Whole milk and baby food.
hospital with a possible diagnosis of ectopic pregnancy. Nurse Hazel develops a c. Iron-rich formula only. d. Iron-rich formula and baby food.
plan of care for the client and determines that which of the following nursing
actions is the priority? 15.Mommy Linda is playing with her infant, who is sitting securely alone on the
floor of the clinic. The mother hides a toy behind her back and the infant looks for
a. Monitoring weight b. Assessing for edema it. The nurse is aware that estimated age of the infant
c. Monitoring apical pulse d. Monitoring temperature would be:

4. Nurse Oliver is teaching a diabetic pregnant client about nutrition and insulin a. 6 months b. 4 months c. 8 months d. 10 months
needs during pregnancy. The nurse determines that the client understands
dietary and insulin needs if the client states that the second half of pregnancy 16.Which of the following is the most prominent feature of public health nursing?
require:
a. It involves providing home care to sick people who are not confined in the
a. Decreased caloric intake b. Increased caloric intake hospital.
c. Decreased Insulin d. Increase Insulin b. Services are provided free of charge to people within the catchments area.
c. The public health nurse functions as part of a team providing a public health
5. Nurse Michelle is assessing a 24 year old client with a diagnosis of nursing services.
hydatidiform mole. She is aware that one of the following is unassociated with d. Public health nursing focuses on preventive, not curative, services.
this condition?
17.When the nurse determines whether resources were maximized in
a. Excessive fetal activity. implementing Ligtas Tigdas, she is evaluating
b. Larger than normal uterus for gestational age.
c. Vaginal bleeding a. Effectiveness b. Efficiency c. Adequacy d. Appropriateness
d. Elevated levels of human chorionic gonadotropin.
18.Vangie is a new B.S.N. graduate. She wants to become a Public Health
6. A pregnant client is receiving magnesium sulfate for severe pregnancy Nurse. Where should she apply?
induced hypertension (PIH). The clinical findings that would warrant use of the
antidote , calcium gluconate is: a. Department of Health b. Provincial Health Office
c. Regional Health Office d. Rural Health Unit
a. Urinary output 90 cc in 2 hours. b. Absent patellar reflexes.
c. Rapid respiratory rate above 40/min. d. Rapid rise in blood pressure. 19.Tony is aware the Chairman of the Municipal Health Board is:

7. During vaginal examination of Janah who is in labor, the presenting part is at a. Mayor b. Municipal Health Officer
station plus two. Nurse, correctly interprets it as: c. Public Health Nurse d. Any qualified physician

a. Presenting part is 2 cm above the plane of the ischial spines. 20.Myra is the public health nurse in a municipality with a total population of
b. Biparietal diameter is at the level of the ischial spines. about 20,000. There are 3 rural health midwives among the RHU personnel. How
c. Presenting part in 2 cm below the plane of the ischial spines. many more midwife items will the RHU need?
d. Biparietal diameter is 2 cm above the ischial spines.
a. 1 b. 2 c. 3 d. The RHU does not need any more midwife item.
8. A pregnant client is receiving oxytocin (Pitocin) for induction of labor. A
condition that warrant the nurse in-charge to discontinue I.V. infusion of Pitocin 21.According to Freeman and Heinrich, community health nursing is a
is: developmental service. Which of the following best illustrates this statement?
a. The community health nurse continuously develops himself personally and a. Placenta previa b. Abruptio placentae
professionally. c. Premature labor d. Sexually transmitted disease
b. Health education and community organizing are necessary in providing
community health services. 36.A young child named Richard is suspected of having pinworms. The
c. Community health nursing is intended primarily for health promotion and community nurse collects a stool specimen to confirm the diagnosis. The nurse
prevention and treatment of disease. should schedule the collection of this specimen for:
d. The goal of community health nursing is to provide nursing services to people
in their own places of residence. a. Just before bedtime b. After the child has been bathe
c. Any time during the day d. Early in the morning
22.Nurse Tina is aware that the disease declared through Presidential
Proclamation No. 4 as a target for eradication in the Philippines is? 37.In doing a child’s admission assessment, Nurse Betty should be alert to note
which signs or symptoms of chronic lead poisoning?
a. Poliomyelitis b. Measles c. Rabies d. Neonatal tetanus
a. Irritability and seizures b. Dehydration and diarrhea
23.May knows that the step in community organizing that involves training of c. Bradycardia and hypotension d. Petechiae and hematuria
potential leaders in the community is:
38.To evaluate a woman’s understanding about the use of diaphragm for family
a. Integration b. Community organization planning, Nurse Trish asks her to explain how she will use the appliance. Which
c. Community study d. Core group formation response indicates a need for further health teaching?

24.Beth a public health nurse takes an active role in community participation. a. “I should check the diaphragm carefully for holes every time I use it”
What is the primary goal of community organizing? b. “I may need a different size of diaphragm if I gain or lose weight more than 20
pounds”
a. To educate the people regarding community health problems c. “The diaphragm must be left in place for atleast 6 hours after intercourse”
b. To mobilize the people to resolve community health problems d. “I really need to use the diaphragm and jelly most during the middle of my
c. To maximize the community’s resources in dealing with health problems. menstrual cycle”.
d. To maximize the community’s resources in dealing with health problems.
39.Hypoxia is a common complication of laryngotracheobronchitis. Nurse Oliver
25.Tertiary prevention is needed in which stage of the natural history of disease? should frequently assess a child with laryngotracheobronchitis for:

a. Pre-pathogenesis b. Pathogenesis c. Prodromal d. Terminal a. Drooling b. Muffled voice c. Restlessness d. Low-grade fever

26.The nurse is caring for a primigravid client in the labor and delivery area. 40.How should Nurse Michelle guide a child who is blind to walk to the
Which condition would place the client at risk for disseminated playroom?
intravascular coagulation (DIC)?
a. Without touching the child, talk continuously as the child walks down the hall.
a. Intrauterine fetal death. b. Placenta accreta. b. Walk one step ahead, with the child’s hand on the nurse’s elbow.
c. Dysfunctional labor. d. Premature rupture of the membranes. c. Walk slightly behind, gently guiding the child forward.
d. Walk next to the child, holding the child’s hand.
27.A fullterm client is in labor. Nurse Betty is aware that the fetal heart rate would
be: 41.When assessing a newborn diagnosed with ductus arteriosus, Nurse Olivia
should expect that the child most likely would have an:
a. 80 to 100 beats/minute b. 100 to 120 beats/minute
c. 120 to 160 beats/minute d. 160 to 180 beats/minute a. Loud, machinery-like murmur.
b. Bluish color to the lips.
28.The skin in the diaper area of a 7 month old infant is excoriated and red. c. Decreased BP reading in the upper extremities
Nurse Hazel should instruct the mother to: d. Increased BP reading in the upper extremities.

a. Change the diaper more often. 42.The reason nurse May keeps the neonate in a neutral thermal environment is
b. Apply talc powder with diaper changes. that when a newborn becomes too cool, the neonate requires:
c. Wash the area vigorously with each diaper change.
d. Decrease the infant’s fluid intake to decrease saturating diapers. a. Less oxygen, and the newborn’s metabolic rate increases.
b. More oxygen, and the newborn’s metabolic rate decreases.
29.Nurse Carla knows that the common cardiac anomalies in children with Down c. More oxygen, and the newborn’s metabolic rate increases.
Syndrome (tri-somy 21) is: d. Less oxygen, and the newborn’s metabolic rate decreases.

a. Atrial septal defect b. Pulmonic stenosis 43.Before adding potassium to an infant’s I.V. line, Nurse Ron must be sure to
c. Ventricular septal defect d. Endocardial cushion defect assess whether this infant has:

30.Malou was diagnosed with severe preeclampsia is now receiving I.V. a. Stable blood pressure b. Patant fontanelles
magnesium sulfate. The adverse effects associated with magnesium sulfate is: c. Moro’s reflex d. Voided

a. Anemia b. Decreased urine output 44.Nurse Carla should know that the most common causative factor of dermatitis
c. Hyperreflexia d. Increased respiratory rate in infants and younger children is:

31.A 23 year old client is having her menstrual period every 2 weeks that last for a. Baby oil b. Baby lotion c. Laundry detergent d. Powder with cornstarch
1 week. This type of menstrual pattern is bets defined by:
45.During tube feeding, how far above an infant’s stomach should the nurse hold
a. Menorrhagia b. Metrorrhagia c. Dyspareunia d. Amenorrhea the syringe with formula?

32.Jannah is admitted to the labor and delivery unit. The critical laboratory result a. 6 inches b. 12 inches c. 18 inches d. 24 inches
for this client would be:
46. In a mothers’ class, Nurse Lhynnete discussed childhood diseases such as
a. Oxygen saturation b. Iron binding capacity c. Blood typing d. Serum Calcium chicken pox. Which of the following statements about chicken pox is correct?

33.Nurse Gina is aware that the most common condition found during the a. The older one gets, the more susceptible he becomes to the complications of
second-trimester of pregnancy is: chicken pox.
b. A single attack of chicken pox will prevent future episodes, including conditions
a. Metabolic alkalosis b. Respiratory acidosis c. Mastitis d. Physiologic anemia such as shingles.
c. To prevent an outbreak in the community, quarantine may be imposed by
34.Nurse Lynette is working in the triage area of an emergency department. She health authorities.
sees that several pediatric clients arrive simultaneously. The client who needs to d. Chicken pox vaccine is best given when there is an impending outbreak in the
be treated first is: community.

a. A crying 5 year old child with a laceration on his scalp. 47.Barangay Pinoy had an outbreak of German measles. To prevent congenital
b. A 4 year old child with a barking coughs and flushed appearance. rubella, what is the BEST advice that you can give to women in the first trimester
c. A 3 year old child with Down syndrome who is pale and asleep in of pregnancy in the barangay Pinoy?
his mother’s arms.
d. A 2 year old infant with stridorous breath sounds, sitting up in his a. Advice them on the signs of German measles.
mother’s arms and drooling. b. Avoid crowded places, such as markets and movie houses.
c. Consult at the health center where rubella vaccine may be given.
35.Maureen in her third trimester arrives at the emergency room with painless d. Consult a physician who may give them rubella immunoglobulin.
vaginal bleeding. Which of the following conditions is suspected?
a. Inability to drink b. High grade fever
48.Myrna a public health nurse knows that to determine possible sources of c. Signs of severe dehydration d. Cough for more than 30 days
sexually transmitted infections, the BEST method that may be undertaken is:
62.Jimmy a 2-year old child revealed “baggy pants”. As a nurse, using the IMCI
a. Contact tracing b. Community survey guidelines, how will you manage Jimmy?
c. Mass screening tests d. Interview of suspects
a. Refer the child urgently to a hospital for confinement.
49.A 33-year old female client came for consultation at the health center with the b. Coordinate with the social worker to enroll the child in a feeding program.
chief complaint of fever for a week. Accompanying symptoms were muscle pains c. Make a teaching plan for the mother, focusing on menu planning for her child.
and body malaise. A week after the start of fever, the client noted yellowish d. Assess and treat the child for health problems like infections and intestinal
discoloration of his sclera. History showed that he waded in flood waters about 2 parasitism.
weeks before the onset of symptoms. Based on her history, which disease
condition will you suspect? 63.Gina is using Oresol in the management of diarrhea of her 3-year old child.
She asked you what to do if her child vomits. As a nurse you will tell her to:
a. Hepatitis A b. Hepatitis B c. Tetanus d. Leptospirosis
a. Bring the child to the nearest hospital for further assessment.
50.Mickey a 3-year old client was brought to the health center with the chief b. Bring the child to the health center for intravenous fluid therapy.
complaint of severe diarrhea and the passage of “rice water” stools. The client is c. Bring the child to the health center for assessment by the physician.
most probably suffering from which condition? d. Let the child rest for 10 minutes then continue giving Oresol more slowly.

a. Giardiasis b. Cholera c. Amebiasis d. Dysentery 64.Nikki a 5-month old infant was brought by his mother to the health center
because of diarrhea for 4 to 5 times a day. Her skin goes back slowly after a skin
51.The most prevalent form of meningitis among children aged 2 months to 3 pinch and her eyes are sunken. Using the IMCI guidelines, you will classify this
years is caused by which microorganism? infant in which category?

a. Hemophilus influenza b. Morbillivirus a. No signs of dehydration b. Some dehydration


c. Steptococcus pneumonia d. Neisseria meningitidis c. Severe dehydration d. The data is insufficient.

52.The student nurse is aware that the pathognomonic sign of measles is 65.Chris a 4-month old infant was brought by her mother to the health center
Koplik’s spot and you may see Koplik’s spot by inspecting the: because of cough. His respiratory rate is 42/minute. Using the Integrated
Management of Child Illness (IMCI) guidelines of assessment, his breathing is
a. Nasal mucosa b. Buccal mucosa considered as:
c. Skin on the abdomen d. Skin on neck
a. Fast b. Slow c. Normal d. Insignificant
53.Angel was diagnosed as having Dengue fever. You will say that there is slow
capillary refill when the color of the nailbed that you pressed does not return 66.Maylene had just received her 4th dose of tetanus toxoid. She is aware that
within how many seconds? her baby will have protection against tetanus for

a. 3 seconds b. 6 seconds c. 9 seconds d. 10 seconds a. 1 year b. 3 years c. 5 years d. Lifetime

54.In Integrated Management of Childhood Illness, the nurse is aware that the 67.Nurse Ron is aware that unused BCG should be discarded after how many
severe conditions generally require urgent referral to a hospital. Which of the hours of reconstitution?
following severe conditions DOES NOT always require urgent referral to a
hospital? a. 2 hours b. 4 hours c. 8 hours d. At the end of the day

a. Mastoiditis b. Severe dehydration 68.The nurse explains to a breastfeeding mother that breast milk is sufficient for
c. Severe pneumonia d. Severe febrile disease all of the baby’s nutrient needs only up to:

55.Myrna a public health nurse will conduct outreach immunization in a barangay a. 5 months b. 6 months c. 1 year d. 2 years
Masay with a population of about 1500. The estimated number of infants in the
barangay would be: 69.Nurse Ron is aware that the gestational age of a conceptus that is considered
viable (able to live outside the womb) is:
a. 45 infants b. 50 infants c. 55 infants d. 65 infants
a. 8 weeks b. 12 weeks c. 24 weeks d. 32 weeks
56.The community nurse is aware that the biological used in Expanded Program
on Immunization (EPI) should NOT be stored in the freezer? 70.When teaching parents of a neonate the proper position for the neonate’s
sleep, the nurse Patricia stresses the importance of placing the neonate on his
a. DPT b. Oral polio vaccine c. Measles vaccine d. MMR back to reduce the risk of which of the following?

57.It is the most effective way of controlling schistosomiasis in an endemic area? a. Aspiration
b. Sudden infant death syndrome (SIDS)
a. Use of molluscicides c. Suffocation
b. Building of foot bridges d. Gastroesophageal reflux (GER)
c. Proper use of sanitary toilets
d. Use of protective footwear, such as rubber boots 71.Which finding might be seen in baby James a neonate suspected of having an
infection?
58.Several clients is newly admitted and diagnosed with leprosy. Which of the
following clients should be classified as a case of multibacillary leprosy? a. Flushed cheeks b. Increased temperature
c. Decreased temperature d. Increased activity level
a. 3 skin lesions, negative slit skin smear
b. 3 skin lesions, positive slit skin smear 72.Baby Jenny who is small-for-gestation is at increased risk during the
c. 5 skin lesions, negative slit skin smear transitional period for which complication?
d. 5 skin lesions, positive slit skin smear
a. Anemia probably due to chronic fetal hyposia
59.Nurses are aware that diagnosis of leprosy is highly dependent on recognition b. Hyperthermia due to decreased glycogen stores
of symptoms. Which of the following is an early sign of c. Hyperglycemia due to decreased glycogen stores
leprosy? d. Polycythemia probably due to chronic fetal hypoxia

a. Macular lesions b. Inability to close eyelids 73.Marjorie has just given birth at 42 weeks’ gestation. When the nurse
c. Thickened painful nerves d. Sinking of the nosebridge assessing the neonate, which physical finding is expected?

60.Marie brought her 10 month old infant for consultation because of fever, a. A sleepy, lethargic baby
started 4 days prior to consultation. In determining malaria risk, what will you do? b. Lanugo covering the body
c. Desquamation of the epidermis
a. Perform a tourniquet test. d. Vernix caseosa covering the body
b. Ask where the family resides.
c. Get a specimen for blood smear. 74.After reviewing the Myrna’s maternal history of magnesium sulfate during
d. Ask if the fever is present everyday. labor, which condition would nurse Richard anticipate as a potential problem in
the neonate?
61.Susie brought her 4 years old daughter to the RHU because of cough and
colds. Following the IMCI assessment guide, which of the following is a danger a. Hypoglycemia b. Jitteriness
sign that indicates the need for urgent referral to a hospital? c. Respiratory depression d. Tachycardia
75.Which symptom would indicate the Baby Alexandra was adapting 86. Nurse John is knowledgeable that usually individual twins will grow
appropriately to extra-uterine life without difficulty? appropriately and at the same rate as singletons until how many weeks?

a. Nasal flaring a. 16 to 18 weeks b. 18 to 22 weeks


b. Light audible grunting c. 30 to 32 weeks d. 38 to 40 weeks
c. Respiratory rate 40 to 60 breaths/minute
d. Respiratory rate 60 to 80 breaths/minute 87. Which of the following classifications applies to monozygotic twins for whom
the cleavage of the fertilized ovum occurs more than 13 days after fertilization?
76. When teaching umbilical cord care for Jennifer a new mother, the nurse
Jenny would include which information? a. conjoined twins
b. diamniotic dichorionic twins
a. Apply peroxide to the cord with each diaper change c. diamniotic monochorionic twin
b. Cover the cord with petroleum jelly after bathing d. monoamniotic monochorionic twins
c. Keep the cord dry and open to air
d. Wash the cord with soap and water each day during a tub bath. 88. Tyra experienced painless vaginal bleeding has just been diagnosed as
having a placenta previa. Which of the following procedures is usually performed
77.Nurse John is performing an assessment on a neonate. Which of the following to diagnose placenta previa?
findings is considered common in the healthy neonate?
a. Amniocentesis
a. Simian crease b. Conjunctival hemorrhage b. Digital or speculum examination
c. Cystic hygroma d. Bulging fontanelle c. External fetal monitoring
d. Ultrasound
78.Dr. Esteves decides to artificially rupture the membranes of a mother who is
on labor. Following this procedure, the nurse Hazel checks the fetal heart tones 89. Nurse Arnold knows that the following changes in respiratory functioning
for which the following reasons? during pregnancy is considered normal:

a. To determine fetal well-being. a. Increased tidal volume


b. To assess for prolapsed cord b. Increased expiratory volume
c. To assess fetal position c. Decreased inspiratory capacity
d. To prepare for an imminent delivery. d. Decreased oxygen consumption

79.Which of the following would be least likely to indicate anticipated bonding 90. Emily has gestational diabetes and it is usually managed by which of the
behaviors by new parents? following therapy?

a. The parents’ willingness to touch and hold the new born. a. Diet
b. The parent’s expression of interest about the size of the new born. b. Long-acting insulin
c. The parents’ indication that they want to see the newborn. c. Oral hypoglycemic
d. The parents’ interactions with each other. d. Oral hypoglycemic drug and insulin

80.Following a precipitous delivery, examination of the client's vagina reveals 91. Magnesium sulfate is given to Jemma with preeclampsia to prevent which of
the following condition?
a fourth-degree laceration. Which of the following would be contraindicated when
caring for this client? a. Hemorrhage b. Hypertension c. Hypomagnesemia d. Seizure
a. Applying cold to limit edema during the first 12 to 24 hours.
b. Instructing the client to use two or more peripads to cushion the area. 92. Cammile with sickle cell anemia has an increased risk for having a sickle cell
c. Instructing the client on the use of sitz baths if ordered. crisis during pregnancy. Aggressive management of a sickle cell crisis includes
d. Instructing the client about the importance of perineal (kegel) exercises. which of the following measures?

81. A pregnant woman accompanied by her husband, seeks admission to the a. Antihypertensive agents b. Diuretic agents
labor and delivery area. She states that she's in labor and says she attended the c. I.V. fluids d. Acetaminophen (Tylenol) for pain
facility clinic for prenatal care. Which question should the nurse Oliver ask her
first? 93. Which of the following drugs is the antidote for magnesium toxicity?

a. “Do you have any chronic illnesses?” a. Calcium gluconate (Kalcinate)


b. “Do you have any allergies?” b. Hydralazine (Apresoline)
c. “What is your expected due date?” c. Naloxone (Narcan)
d. “Who will be with you during labor?” d. Rho (D) immune globulin (RhoGAM)

82.A neonate begins to gag and turns a dusky color. What should the nurse do 94. Marlyn is screened for tuberculosis during her first prenatal visit. An
first? intradermal injection of purified protein derivative (PPD) of the tuberculin bacilli is
given. She is considered to have a positive test for which of the following results?
a. Calm the neonate.
b. Notify the physician. a. An indurated wheal under 10 mm in diameter appears in 6 to 12 hours.
c. Provide oxygen via face mask as ordered b. An indurated wheal over 10 mm in diameter appears in 48 to 72 hours.
d. Aspirate the neonate’s nose and mouth with a bulb syringe. c. A flat circumcised area under 10 mm in diameter appears in 6 to 12 hours.
d. A flat circumcised area over 10 mm in diameter appears in 48 to 72 hours.
83. When a client states that her "water broke," which of the following actions
would be inappropriate for the nurse to do? 95. Dianne, 24 year-old is 27 weeks’ pregnant arrives at her physician’s office
with complaints of fever, nausea, vomiting, malaise, unilateral flank pain, and
a. Observing the pooling of straw-colored fluid. costovertebral angle tenderness. Which of the following diagnoses is most likely?
b. Checking vaginal discharge with nitrazine paper.
c. Conducting a bedside ultrasound for an amniotic fluid index. a. Asymptomatic bacteriuria b. Bacterial vaginosis
d. Observing for flakes of vernix in the vaginal discharge. c. Pyelonephritis d. Urinary tract infection (UTI)

84. A baby girl is born 8 weeks premature. At birth, she has no spontaneous 96. Rh isoimmunization in a pregnant client develops during which of the
respirations but is successfully resuscitated. Within several hours she develops following conditions?
respiratory grunting, cyanosis, tachypnea, nasal flaring, and retractions. She's
diagnosed with respiratory distress syndrome, intubated, and placed on a a. Rh-positive maternal blood crosses into fetal blood, stimulating fetal
ventilator. Which nursing action should be included in the baby's plan of care to antibodies.
prevent retinopathy of prematurity? b. Rh-positive fetal blood crosses into maternal blood, stimulating
maternal antibodies.
a. Cover his eyes while receiving oxygen. c. Rh-negative fetal blood crosses into maternal blood, stimulating
b. Keep her body temperature low. maternal antibodies.
c. Monitor partial pressure of oxygen (Pao2) levels. d. Rh-negative maternal blood crosses into fetal blood, stimulating fetal
d. Humidify the oxygen. antibodies.

85. Which of the following is normal newborn calorie intake? 97. To promote comfort during labor, the nurse John advises a client to assume
certain positions and avoid others. Which position may cause maternal
a. 110 to 130 calories per kg. hypotension and fetal hypoxia?
b. 30 to 40 calories per lb of body weight.
c. At least 2 ml per feeding a. Lateral position b. Squatting position
d. 90 to 100 calories per kg c. Supine position d. Standing position
98. Celeste who used heroin during her pregnancy delivers a neonate. When Rationale: Because feeding requires so much energy, an infant with heart failure
assessing the neonate, the nurse Lhynnette expects to find: should rest before feeding.

a. Lethargy 2 days after birth. 14. Answer: (C) Iron-rich formula only.
b. Irritability and poor sucking. Rationale: The infants at age 5 months should receive iron-rich formula and that
c. A flattened nose, small eyes, and thin lips. they shouldn’t receive solid food, even baby food until age 6 months.
d. Congenital defects such as limb anomalies.
15. Answer: (D) 10 months
99. The uterus returns to the pelvic cavity in which of the following time frames? Rationale: A 10 month old infant can sit alone and understands object
permanence, so he would look for the hidden toy. At age 4 to 6 months, infants
a. 7th to 9th day postpartum. can’t sit securely alone. At age 8 months, infants can sit securely alone but
b. 2 weeks postpartum. cannot understand the permanence of objects.
c. End of 6th week postpartum.
d. When the lochia changes to alba. 16. Answer: (D) Public health nursing focuses on preventive, not curative,
services.
100. Maureen, a primigravida client, age 20, has just completed a difficult, Rationale: The catchments area in PHN consists of a residential community,
forceps-assisted delivery of twins. Her labor was unusually long and required many of whom are well individuals who have greater need for
oxytocin (Pitocin) augmentation. The nurse who's caring for her should stay alert preventive rather than curative services.
for:
17. Answer: (B) Efficiency
a. Uterine inversion b. Uterine atony Rationale: Efficiency is determining whether the goals were attained at the least
c. Uterine involution d. Uterine discomfort possible cost.
RATIO 2 
18. Answer: (D) Rural Health Unit
1. Answer: (A) Inevitable Rationale: R.A. 7160 devolved basic health services to local government units
Rationale: An inevitable abortion is termination of pregnancy that cannot be (LGU’s ). The public health nurse is an employee of the LGU.
prevented. Moderate to severe bleeding with mild cramping and cervical dilation
would be noted in this type of abortion. 19. Answer: (A) Mayor
Rationale: The local executive serves as the chairman of the Municipal Health
2. Answer: (B) History of syphilis Board.
Rationale: Maternal infections such as syphilis, toxoplasmosis, and rubella are
causes of spontaneous abortion. 20. Answer: (A) 1
Rationale: Each rural health midwife is given a population assignment of about
3. Answer: (C) Monitoring apical pulse 5,000.
Rationale: Nursing care for the client with a possible ectopic pregnancy is
focused on preventing or identifying hypovolemic shock and controlling pain. An 21. Answer: (B) Health education and community organizing are necessary in
elevated pulse rate is an indicator of shock. providing community health services.
Rationale: The community health nurse develops the health capability of people
4. Answer: (B) Increased caloric intake through health education and community organizing activities.
Rationale: Glucose crosses the placenta, but insulin does not. High fetal
demands for glucose, combined with the insulin resistance caused by hormonal 22. Answer: (B) Measles
changes in the last half of pregnancy can result in elevation of maternal blood Rationale: Presidential Proclamation No. 4 is on the Ligtas Tigdas Program.
glucose levels. This increases the mother’s demand for insulin and is referred to
as the diabetogenic effect of pregnancy. 23. Answer: (D) Core group formation
Rationale: In core group formation, the nurse is able to transfer the technology of
5. Answer: (A) Excessive fetal activity. community organizing to the potential or informal community leaders through a
Rationale: The most common signs and symptoms of hydatidiform mole includes training program.
elevated levels of human chorionic gonadotropin, vaginal bleeding, larger than
normal uterus for gestational age, failure to detect fetal heart activity even with 24. Answer: (D) To maximize the community’s resources in dealing with health
sensitive instruments, excessive nausea and vomiting, and early development of problems.
pregnancy-induced hypertension. Fetal activity would not be noted. Rationale: Community organizing is a developmental service, with the goal of
developing the people’s self-reliance in dealing with community
6. Answer: (B) Absent patellar reflexes health problems. A, B and C are objectives of contributory objectives to this goal.
Rationale: Absence of patellar reflexes is an indicator of hypermagnesemia,
which requires administration of calcium gluconate. 25. Answer: (D) Terminal
Rationale: Tertiary prevention involves rehabilitation, prevention of permanent
7. Answer: (C) Presenting part in 2 cm below the plane of the ischial spines. disability and disability limitation appropriate for convalescents, the disabled,
Rationale: Fetus at station plus two indicates that the presenting part is 2 cm complicated cases and the terminally ill (those in the terminal stage of a disease).
below the plane of the ischial spines.
26. Answer: (A) Intrauterine fetal death.
8. Answer: (A) Contractions every 1 ½ minutes lasting 70-80 seconds. Rationale: Intrauterine fetal death, abruptio placentae, septic shock, and amniotic
Rationale: Contractions every 1 ½ minutes lasting 70-80 seconds, is indicative of fluid embolism may trigger normal clotting mechanisms; if clotting factors are
hyperstimulation of the uterus, which could result in injury to the mother and the depleted, DIC may occur. Placenta accreta, dysfunctional labor, and premature
fetus if Pitocin is not discontinued. rupture of the membranes aren't associated with DIC.

9. Answer: (C) EKG tracings 27. Answer: (C) 120 to 160 beats/minute
Rationale: A potential side effect of calcium gluconate administration is cardiac Rationale: A rate of 120 to 160 beats/minute in the fetal heart appropriate for
arrest. Continuous monitoring of cardiac activity (EKG) throught administration of filling the heart with blood and pumping it out to the system.
calcium gluconate is an essential part of care.
28. Answer: (A) Change the diaper more often.
10. Answer: (D) First low transverse caesarean was for breech position. Fetus in Rationale: Decreasing the amount of time the skin comes contact with wet soiled
this pregnancy is in a vertex presentation. diapers will help heal the irritation.
Rationale: This type of client has no obstetrical indication for a caesarean section
as she did with her first caesarean delivery. 29. Answer: (D) Endocardial cushion defect
Rationale: Endocardial cushion defects are seen most in children with Down
11. Answer: (A) Talk to the mother first and then to the toddler. syndrome, asplenia, or polysplenia.
Rationale: When dealing with a crying toddler, the best approach is to talk to the
mother and ignore the toddler first. This approach helps the toddler get used to 30. Answer: (B) Decreased urine output
the nurse before she attempts any procedures. It also gives the toddler an Rationale: Decreased urine output may occur in clients receiving I.V. magnesium
opportunity to see that the mother trusts the nurse. and should be monitored closely to keep urine output at
greater than 30 ml/hour, because magnesium is excreted through the kidneys
12. Answer: (D) Place the infant’s arms in soft elbow restraints. and can easily accumulate to toxic levels.
Rationale: Soft restraints from the upper arm to the wrist prevent the infant from
touching her lip but allow him to hold a favorite item such as a blanket. Because 31. Answer: (A) Menorrhagia
they could damage the operative site, such as objects as pacifiers, suction Rationale: Menorrhagia is an excessive menstrual period.
catheters, and small spoons shouldn’t be placed in a baby’s mouth after cleft
repair. A baby in a prone position may rub her face on the sheets and traumatize 32. Answer: (C) Blood typing
the operative site. The suture line should be cleaned gently to prevent infection, Rationale: Blood type would be a critical value to have because the risk of blood
which could interfere with healing and damage the cosmetic appearance of the loss is always a potential complication during the labor and delivery process.
repair. Approximately 40% of a woman’s cardiac output is delivered to the uterus,
therefore, blood loss can occur quite rapidly in the event of uncontrolled bleeding.
13. Answer: (B) Allow the infant to rest before feeding.
33. Answer: (D) Physiologic anemia
Rationale: Hemoglobin values and hematocrit decrease during pregnancy as the Rationale: Hemophilus meningitis is unusual over the age of 5 years. In
increase in plasma volume exceeds the increase in red blood cell production. developing countries, the peak incidence is in children less than 6 months of age.
Morbillivirus is the etiology of measles. Streptococcus pneumoniae and Neisseria
34. Answer: (D) A 2 year old infant with stridorous breath sounds, sitting up in his meningitidis may cause meningitis, but age distribution is not specific in young
mother’s arms and drooling. children.
Rationale: The infant with the airway emergency should be treated first, because
of the risk of epiglottitis. 52. Answer: (B) Buccal mucosa
Rationale: Koplik’s spot may be seen on the mucosa of the mouth or the throat.
35. Answer: (A) Placenta previa
Rationale: Placenta previa with painless vaginal bleeding. 53. Answer: (A) 3 seconds
Rationale: Adequate blood supply to the area allows the return of the color of the
36. Answer: (D) Early in the morning nailbed within 3 seconds.
Rationale: Based on the nurse’s knowledge of microbiology, the specimen should
be collected early in the morning. The rationale for this 54. Answer: (B) Severe dehydration
timing is that, because the female worm lays eggs at night around the perineal Rationale: The order of priority in the management of severe dehydration is as
area, the first bowel movement of the day will yield the best follows: intravenous fluid therapy, referral to a facility where IV fluids can be
results. The specific type of stool specimen used in the diagnosis of pinworms is initiated within 30 minutes, Oresol or nasogastric tube. When the foregoing
called the tape test. measures are not possible or effective, then urgent referral to the hospital is
done.
37. Answer: (A) Irritability and seizures
Rationale: Lead poisoning primarily affects the CNS, causing increased 55. Answer: (A) 45 infants
intracranial pressure. This condition results in irritability and changes in level of Rationale: To estimate the number of infants, multiply total population by 3%.
consciousness, as well as seizure disorders, hyperactivity, and learning
disabilities. 56. Answer: (A) DPT
Rationale: DPT is sensitive to freezing. The appropriate storage temperature of
38. Answer: (D) “I really need to use the diaphragm and jelly most during the DPT is 2 to 8° C only. OPV and measles vaccine are highly
middle of my menstrual cycle”. sensitive to heat and require freezing. MMR is not an immunization in the
Rationale: The woman must understand that, although the “fertile” period is Expanded Program on Immunization.
approximately mid-cycle, hormonal variations do occur and can result in early or
late ovulation. To be effective, the diaphragm should be inserted before every 57. Answer: (C) Proper use of sanitary toilets
intercourse. Rationale: The ova of the parasite get out of the human body together with feces.
Cutting the cycle at this stage is the most effective way of preventing the spread
39. Answer: (C) Restlessness of the disease to susceptible hosts.
Rationale: In a child, restlessness is the earliest sign of hypoxia. Late signs of
hypoxia in a child are associated with a change in color, such as pallor or 58. Answer: (D) 5 skin lesions, positive slit skin smear
cyanosis. Rationale: A multibacillary leprosy case is one who has a positive slit skin smear
and at least 5 skin lesions.
40. Answer: (B) Walk one step ahead, with the child’s hand on the nurse’s elbow.
Rationale: This procedure is generally recommended to follow in guiding a 59. Answer: (C) Thickened painful nerves
person who is blind. Rationale: The lesion of leprosy is not macular. It is characterized by a change in
skin color (either reddish or whitish) and loss of sensation, sweating and hair
41. Answer: (A) Loud, machinery-like murmur. growth over the lesion. Inability to close the eyelids (lagophthalmos) and sinking
Rationale: A loud, machinery-like murmur is a characteristic finding associated of the nosebridge are late symptoms.
with patent ductus arteriosus.
60. Answer: (B) Ask where the family resides.
42. Answer: (C) More oxygen, and the newborn’s metabolic rate increases. Rationale: Because malaria is endemic, the first question to determine malaria
Rationale: When cold, the infant requires more oxygen and there is an increase risk is where the client’s family resides. If the area of residence is not a known
in metabolic rate. Non-shievering thermogenesis is a complex process that endemic area, ask if the child had traveled within the past 6 months, where she
increases the metabolic rate and rate of oxygen consumption, therefore, the was brought and whether she stayed overnight in that area.
newborn increase heat production.
61. Answer: (A) Inability to drink
43. Answer: (D) Voided Rationale: A sick child aged 2 months to 5 years must be referred urgently to a
Rationale: Before administering potassium I.V. to any client, the nurse must first hospital if he/she has one or more of the following signs: not able to feed or drink,
check that the client’s kidneys are functioning and that the client is voiding. If the vomits everything, convulsions, abnormally sleepy or difficult to awaken.
client is not voiding, the nurse should withhold the potassium and notify the
physician. 62. Answer: (A) Refer the child urgently to a hospital for confinement.
Rationale: “Baggy pants” is a sign of severe marasmus. The best management is
44. Answer: (c) Laundry detergent urgent referral to a hospital.
Rationale: Eczema or dermatitis is an allergic skin reaction caused by an
offending allergen. The topical allergen that is the most common causative factor 63. Answer: (D) Let the child rest for 10 minutes then continue giving Oresol
is laundry detergent. more slowly.
Rationale: If the child vomits persistently, that is, he vomits everything that he
45. Answer: (A) 6 inches takes in, he has to be referred urgently to a hospital. Otherwise, vomiting is
Rationale: This distance allows for easy flow of the formula by gravity, but the managed by letting the child rest for 10 minutes and then continuing with Oresol
flow will be slow enough not to overload the stomach too rapidly. administration. Teach the mother to give Oresol more slowly.

46. Answer: (A) The older one gets, the more susceptible he becomes to the 64. Answer: (B) Some dehydration
complications of chicken pox. Rationale: Using the assessment guidelines of IMCI, a child (2 months to 5 years
Rationale: Chicken pox is usually more severe in adults than in children. old) with diarrhea is classified as having SOME DEHYDRATION if he shows 2 or
Complications, such as pneumonia, are higher in incidence in adults. more of the following signs: restless or irritable, sunken eyes, the skin goes back
slow after a skin pinch.
47. Answer: (D) Consult a physician who may give them rubella immunoglobulin.
Rationale: Rubella vaccine is made up of attenuated German measles viruses. 65. Answer: (C) Normal
This is contraindicated in pregnancy. Immune globulin, a specific prophylactic Rationale: In IMCI, a respiratory rate of 50/minute or more is fast breathing for an
against German measles, may be given to pregnant women. infant aged 2 to 12 months.

48. Answer: (A) Contact tracing 66. Answer: (A) 1 year


Rationale: Contact tracing is the most practical and reliable method of finding Rationale: The baby will have passive natural immunity by placental transfer of
possible sources of person-to-person transmitted infections, such as sexually antibodies. The mother will have active artificial immunity
transmitted diseases. lasting for about 10 years. 5 doses will give the mother lifetime protection.

49. Answer: (D) Leptospirosis 67. Answer: (B) 4 hours


Rationale: Leptospirosis is transmitted through contact with the skin or mucous Rationale: While the unused portion of other biologicals in EPI may be given until
membrane with water or moist soil contaminated with urine of infected animals, the end of the day, only BCG is discarded 4 hours after
like rats. reconstitution. This is why BCG immunization is scheduled only in the morning.

50. Answer: (B) Cholera 68. Answer: (B) 6 months


Rationale: Passage of profuse watery stools is the major symptom of cholera. Rationale: After 6 months, the baby’s nutrient needs, especially the baby’s iron
Both amebic and bacillary dysentery are characterized by the requirement, can no longer be provided by mother’s milk
presence of blood and/or mucus in the stools. Giardiasis is characterized by fat alone.
malabsorption and, therefore, steatorrhea.
69. Answer: (C) 24 weeks
51. Answer: (A) Hemophilus influenzae
Rationale: At approximately 23 to 24 weeks’ gestation, the lungs are developed 84. Answer: (C) Monitor partial pressure of oxygen (Pao2) levels.
enough to sometimes maintain extrauterine life. The lungs are the most immature Rationale: Monitoring PaO2 levels and reducing the oxygen concentration to
system during the gestation period. Medical care for premature labor begins keep PaO2 within normal limits reduces the risk of retinopathy of prematurity in a
much earlier (aggressively at 21 weeks’ gestation) premature infant receiving oxygen. Covering the infant's eyes and humidifying
the oxygen don't reduce the risk of retinopathy of prematurity. Because cooling
70. Answer: (B) Sudden infant death syndrome (SIDS) increases the risk of acidosis, the infant should be kept warm so that his
Rationale: Supine positioning is recommended to reduce the risk of SIDS in respiratory distress
infancy. The risk of aspiration is slightly increased with the supine position. isn't aggravated.
Suffocation would be less likely with an infant supine than prone and the position
for GER requires the head of the bed to be elevated. 85. Answer: (A) 110 to 130 calories per kg.
Rationale: Calories per kg is the accepted way of determined appropriate
71. Answer: (C) Decreased temperature nutritional intake for a newborn. The recommended calorie requirement is 110 to
Rationale: Temperature instability, especially when it results in a low temperature 130 calories per kg of newborn body weight. This level will maintain a consistent
in the neonate, may be a sign of infection. The neonate’s blood glucose level and provide enough calories for continued growth and
color often changes with an infection process but generally becomes ashen or development.
mottled. The neonate with an infection will usually show a
decrease in activity level or lethargy. 86. Answer: (C) 30 to 32 weeks
Rationale: Individual twins usually grow at the same rate as singletons until 30 to
72. Answer: (D) Polycythemia probably due to chronic fetal hypoxia 32 weeks’ gestation, then twins don’t’ gain weight as rapidly as singletons of the
Rationale: The small-for-gestation neonate is at risk for developing polycythemia same gestational age. The placenta can no longer keep pace with the nutritional
during the transitional period in an attempt to decrease requirements of both fetuses after 32 weeks, so there’s some growth retardation
hypoxia. The neonates are also at increased risk for developing hypoglycemia in twins if they remain in utero at 38 to 40 weeks.
and hypothermia due to decreased glycogen stores.
87. Answer: (A) conjoined twins
73. Answer: (C) Desquamation of the epidermis Rationale: The type of placenta that develops in monozygotic twins depends on
Rationale: Postdate fetuses lose the vernix caseosa, and the epidermis may the time at which cleavage of the ovum occurs. Cleavage in conjoined twins
become desquamated. These neonates are usually very alert. Lanugo is missing occurs more than 13 days after fertilization. Cleavage that occurs less than 3 day
in the postdate neonate. after fertilization results in diamniotic dicchorionic twins. Cleavage that occurs
between days 3 and 8 results in diamniotic monochorionic twins. Cleavage that
74. Answer: (C) Respiratory depression occurs between days 8 to 13 result in monoamniotic monochorionic twins.
Rationale: Magnesium sulfate crosses the placenta and adverse neonatal effects
are respiratory depression, hypotonia, and bradycardia. The serum blood sugar 88. Answer: (D) Ultrasound
isn’t affected by magnesium sulfate. The neonate would be floppy, not jittery. Rationale: Once the mother and the fetus are stabilized, ultrasound evaluation of
the placenta should be done to determine the cause of the bleeding.
75. Answer: (C) Respiratory rate 40 to 60 breaths/minute Amniocentesis is contraindicated in placenta previa. A digital or speculum
Rationale: A respiratory rate 40 to 60 breaths/minute is normal for a neonate examination shouldn’t be done as this may lead to severe bleeding or
during the transitional period. Nasal flaring, respiratory rate more than 60 hemorrhage. External fetal monitoring won’t detect a placenta previa, although it
breaths/minute, and audible grunting are signs of respiratory distress. will detect fetal distress, which may result from blood loss or placenta separation.

76. Answer: (C) Keep the cord dry and open to air 89. Answer: (A) Increased tidal volume
Rationale: Keeping the cord dry and open to air helps reduce infection and Rationale: A pregnant client breathes deeper, which increases the tidal volume of
hastens drying. Infants aren’t given tub bath but are sponged off until the cord gas moved in and out of the respiratory tract with each breath. The expiratory
falls off. Petroleum jelly prevents the cord from drying and encourages infection. volume and residual volume decrease as the pregnancy progresses. The
Peroxide could be painful and isn’t recommended. inspiratory capacity increases during pregnancy. The increased oxygen
consumption in the pregnant client is 15% to 20% greater than in the
77. Answer: (B) Conjunctival hemorrhage nonpregnant state.
Rationale: Conjunctival hemorrhages are commonly seen in neonates secondary
to the cranial pressure applied during the birth process. Bulging fontanelles are a 90. Answer: (A) Diet
sign of intracranial pressure. Simian creases are present in 40% of the neonates Rationale: Clients with gestational diabetes are usually managed by diet alone to
with trisomy 21. Cystic hygroma is a neck mass that can affect the airway. control their glucose intolerance. Oral hypoglycemic drugs are contraindicated in
pregnancy. Long-acting insulin usually isn’t needed for blood glucose control in
78. Answer: (B) To assess for prolapsed cord the client with gestational diabetes.
Rationale: After a client has an amniotomy, the nurse should assure that the cord
isn't prolapsed and that the baby tolerated the procedure well. The most effective 91. Answer: (D) Seizure
way to do this is to check the fetal heart rate. Fetal well-being is assessed via a Rationale: The anticonvulsant mechanism of magnesium is believes to depress
nonstress test. Fetal position is determined by vaginal examination. Artificial seizure foci in the brain and peripheral neuromuscular blockade.
rupture of membranes doesn't indicate an imminent delivery. Hypomagnesemia isn’t a complication of preeclampsia. Antihypertensive drug
other than magnesium are preferred for sustained hypertension. Magnesium
79. Answer: (D) The parents’ interactions with each other. doesn’t help prevent hemorrhage in preeclamptic clients.
Rationale: Parental interaction will provide the nurse with a good assessment of
the stability of the family's home life but it has no indication for parental bonding. 92. Answer: (C) I.V. fluids
Willingness to touch and hold the newborn, expressing interest about the Rationale: A sickle cell crisis during pregnancy is usually managed by exchange
newborn's size, and indicating a desire to see the newborn are behaviors transfusion oxygen, and L.V. Fluids. The client usually needs a stronger
indicating parental bonding. analgesic than acetaminophen to control the pain of a crisis. Antihypertensive
drugs usually aren’t necessary. Diuretic wouldn’t be used unless fluid overload
80. Answer: (B) Instructing the client to use two or more peripads to cushion the resulted.
area
Rationale: Using two or more peripads would do little to reduce the pain or 93. Answer: (A) Calcium gluconate (Kalcinate)
promote perineal healing. Cold applications, sitz baths, and Kegel Rationale: Calcium gluconate is the antidote for magnesium toxicity. Ten
exercises are important measures when the client has a fourth-degree laceration. milliliters of 10% calcium gluconate is given L.V. push over 3 to 5 minutes.
Hydralazine is given for sustained elevated blood pressure in preeclamptic
81. Answer: (C) “What is your expected due date?” clients. Rho (D) immune globulin is given to women with Rh-negative blood to
Rationale: When obtaining the history of a client who may be in labor, the nurse's prevent antibody formation from RH-positive conceptions. Naloxone is used to
highest priority is to determine her current status, particularly her due date, correct narcotic toxicity.
gravidity, and parity. Gravidity and parity affect the duration of labor and the
potential for labor complications. Later, the nurse should ask about chronic 94. Answer: (B) An indurated wheal over 10 mm in diameter appears in 48 to 72
illnesses, allergies, and support persons. hours.
Rationale: A positive PPD result would be an indurated wheal over 10 mm in
82. Answer: (D) Aspirate the neonate’s nose and mouth with a bulb syringe. diameter that appears in 48 to 72 hours. The area must be a raised wheal, not a
Rationale: The nurse's first action should be to clear the neonate's airway with a flat circumcised area to be considered positive.
bulb syringe. After the airway is clear and the neonate's color improves, the
nurse should comfort and calm the neonate. If the problem recurs or the 95. Answer: (C) Pyelonephritis
neonate's color doesn't improve readily, the nurse should notify the physician. Rational: The symptoms indicate acute pyelonephritis, a serious condition in a
Administering oxygen when the airway isn't clear would be ineffective. pregnant client. UTI symptoms include dysuria, urgency, frequency, and
suprapubic tenderness. Asymptomatic bacteriuria doesn’t cause symptoms.
83. Answer: (C) Conducting a bedside ultrasound for an amniotic fluid index. Bacterial vaginosis causes milky white vaginal discharge but no systemic
Rationale: It isn't within a nurse's scope of practice to perform and interpret a symptoms.
bedside ultrasound under these conditions and without
specialized training. Observing for pooling of straw-colored fluid, checking 96. Answer: (B) Rh-positive fetal blood crosses into maternal blood, stimulating
vaginal discharge with nitrazine paper, and observing for flakes of vernix are maternal antibodies.
appropriate assessments for determining whether a client has ruptured Rationale: Rh isoimmunization occurs when Rh-positive fetal blood cells cross
membranes. into the maternal circulation and stimulate maternal antibody
production. In subsequent pregnancies with Rh-positive fetuses, maternal 8. A client undergone ileostomy, when should the drainage appliance be applied
antibodies may cross back into the fetal circulation and destroy the fetal blood to the stoma?
cells.
a. 24 hours later, when edema has subsided.
97. Answer: (C) Supine position b. In the operating room.
Rationale: The supine position causes compression of the client's aorta and c. After the ileostomy begin to function.
inferior vena cava by the fetus. This, in turn, inhibits maternal d. When the client is able to begin self-care procedures.
circulation, leading to maternal hypotension and, ultimately, fetal hypoxia. The
other positions promote comfort and aid labor progress. For instance, the lateral, 9. A client undergone spinal anesthetic, it will be important that the nurse
or side-lying, position improves maternal and fetal circulation, enhances comfort, immediately position the client in:
increases maternal relaxation, reduces muscle tension, and eliminates pressure
points. The squatting position promotes comfort by taking advantage of gravity. a. On the side, to prevent obstruction of airway by tongue.
The standing position also takes advantage of gravity and aligns the fetus with b. Flat on back.
the pelvic angle. c. On the back, with knees flexed 15 degrees.
d. Flat on the stomach, with the head turned to the side.
98. Answer: (B) Irritability and poor sucking.
Rationale: Neonates of heroin-addicted mothers are physically dependent on the 10.While monitoring a male client several hours after a motor vehicle accident,
drug and experience withdrawal when the drug is no which assessment data suggest increasing intracranial pressure?
longer supplied. Signs of heroin withdrawal include irritability, poor sucking, and
restlessness. Lethargy isn't associated with neonatal heroin a. Blood pressure is decreased from 160/90 to 110/70.
addiction. A flattened nose, small eyes, and thin lips are seen in infants with fetal b. Pulse is increased from 87 to 95, with an occasional skipped beat.
alcohol syndrome. Heroin use during pregnancy hasn't been linked to specific c. The client is oriented when aroused from sleep, and goes back to sleep
congenital anomalies. immediately.
d. The client refuses dinner because of anorexia.
99. Answer: (A) 7th to 9th day postpartum
Rationale: The normal involutional process returns the uterus to the pelvic cavity 11.Mrs. Cruz, 80 years old is diagnosed with pneumonia. Which of the following
in 7 to 9 days. A significant involutional complication is the failure of the uterus to symptoms may appear first?
return to the pelvic cavity within the prescribed time period. This is known as
subinvolution. a. Altered mental status and dehydration b. Fever and chills
c. Hemoptysis and Dyspnea d. Pleuritic chest pain and cough
100. Answer: (B) Uterine atony
Rationale: Multiple fetuses, extended labor stimulation with oxytocin, and 12. A male client has active tuberculosis (TB). Which of the following symptoms
traumatic delivery commonly are associated with uterine atony, which may lead will be exhibit?
to postpartum hemorrhage. Uterine inversion may precede or follow delivery and
commonly results from apparent excessive traction on the umbilical cord and a. Chest and lower back pain
attempts to deliver the placenta manually. Uterine involution and some uterine b. Chills, fever, night sweats, and hemoptysis
discomfort are normal after delivery. c. Fever of more than 104°F (40°C) and nausea
NP3- MS/PSYCH d. Headache and photophobia
1. Nurse Michelle should know that the drainage is normal 4 days after a sigmoid
colostomy when the stool is: 13. Mark, a 7-year-old client is brought to the emergency department. He’s
tachypneic and afebrile and has a respiratory rate of 36 breaths/minute and has
a. Green liquid b. Solid formed c. Loose, bloody d. Semiformed a nonproductive cough. He recently had a cold. Form this history; the client may
have which of the following conditions?
2. Where would nurse Kristine place the call light for a male client with a right-
sided brain attack and left homonymous hemianopsia? a. Acute asthma b. Bronchial pneumonia
c. Chronic obstructive pulmonary disease (COPD) d. Emphysema
a. On the client’s right side
b. On the client’s left side 14. Marichu was given morphine sulfate for pain. She is sleeping and her
c. Directly in front of the client respiratory rate is 4 breaths/minute. If action isn’t taken quickly, she might have
d. Where the client like which of the following reactions?

3. A male client is admitted to the emergency department following an accident. a. Asthma attack b. Respiratory arrest
What are the first nursing actions of the nurse? c. Seizure d. Wake up on his own

a. Check respiration, circulation, neurological response. 15. A 77-year-old male client is admitted for elective knee surgery. Physical
b. Align the spine, check pupils, and check for hemorrhage. examination reveals shallow respirations but no sign of respiratory distress.
c. Check respirations, stabilize spine, and check circulation. Which of the following is a normal physiologic change related to aging?
d. Assess level of consciousness and circulation.
a. Increased elastic recoil of the lungs
4. In evaluating the effect of nitroglycerin, Nurse Arthur should know that it b. Increased number of functional capillaries in the alveoli
reduces preload and relieves angina by: c. Decreased residual volume
d. Decreased vital capacity
a. Increasing contractility and slowing heart rate.
b. Increasing AV conduction and heart rate. 16. Nurse John is caring for a male client receiving lidocaine I.V. Which factor is
c. Decreasing contractility and oxygen consumption. the most relevant to administration of this medication?
d. Decreasing venous return through vasodilation.
a. Decrease in arterial oxygen saturation (SaO2) when measured with a pulse
5. Nurse Patricia finds a female client who is post-myocardial infarction (MI) oximeter.
slumped on the side rails of the bed and unresponsive to shaking or shouting. b. Increase in systemic blood pressure.
Which is the nurse next action? c. Presence of premature ventricular contractions (PVCs) on a cardiac monitor.
d. Increase in intracranial pressure (ICP).
a. Call for help and note the time.
b. Clear the airway 17. Nurse Ron is caring for a male client taking an anticoagulant. The nurse
c. Give two sharp thumps to the precordium, and check the pulse. should teach the client to:
d. Administer two quick blows.
a. Report incidents of diarrhea. b. Avoid foods high in vitamin K
6. Nurse Monett is caring for a client recovering from gastro-intestinal bleeding. c. Use a straight razor when shaving. d. Take aspirin to pain relief.
The nurse should:
18. Nurse Lhynnette is preparing a site for the insertion of an I.V. catheter. The
a. Plan care so the client can receive 8 hours of uninterrupted sleep each night. nurse should treat excess hair at the site by:
b. Monitor vital signs every 2 hours.
c. Make sure that the client takes food and medications at prescribed intervals. a. Leaving the hair intact b. Shaving the area
d. Provide milk every 2 to 3 hours. c. Clipping the hair in the area d. Removing the hair with a depilatory.

7. A male client was on warfarin (Coumadin) before admission, and has been 19. Nurse Michelle is caring for an elderly female with osteoporosis. When
receiving heparin I.V. for 2 days. The partial thromboplastin time (PTT) is 68 teaching the client, the nurse should include information about which major
seconds. What should Nurse Carla do? complication:

a. Stop the I.V. infusion of heparin and notify the physician. a. Bone fracture b. Loss of estrogen
b. Continue treatment as ordered. c. Negative calcium balance d. Dowager’s hump
c. Expect the warfarin to increase the PTT.
d. Increase the dosage, because the level is lower than normal. 20. Nurse Len is teaching a group of women to perform BSE. The nurse should
explain that the purpose of performing the examination is to discover:
a. Cancerous lumps b. Areas of thickness or fullness c. Leave him to get assistance
c. Changes from previous examinations. d. Fibrocystic masses d. Stay with him but not intervene at this time.

21. When caring for a female client who is being treated for hyperthyroidism, it is 33. Nurse Ron is taking a health history of an 84 year old client. Which
important to: information will be most useful to the nurse for planning care?

a. Provide extra blankets and clothing to keep the client warm. a. General health for the last 10 years.
b. Monitor the client for signs of restlessness, sweating, and excessive weight b. Current health promotion activities.
loss during thyroid replacement therapy. c. Family history of diseases.
c. Balance the client’s periods of activity and rest. d. Marital status.
d. Encourage the client to be active to prevent constipation.
34. When performing oral care on a comatose client, Nurse Krina should:
22. Nurse Kris is teaching a client with history of atherosclerosis. To decrease
the risk of atherosclerosis, the nurse should encourage the client to: a. Apply lemon glycerin to the client’s lips at least every 2 hours.
b. Brush the teeth with client lying supine.
a. Avoid focusing on his weight. b. Increase his activity level. c. Place the client in a side lying position, with the head of the bed lowered.
c. Follow a regular diet. d. Continue leading a high-stress lifestyle. d. Clean the client’s mouth with hydrogen peroxide.

23. Nurse Greta is working on a surgical floor. Nurse Greta must logroll a client 35. A 77-year-old male client is admitted with a diagnosis of dehydration and
following a: change in mental status. He’s being hydrated with L.V. fluids. When the nurse
takes his vital signs, she notes he has a fever of 103°F (39.4°C) a cough
a. Laminectomy b. Thoracotomy c. Hemorrhoidectomy d. Cystectomy. producing yellow sputum and pleuritic chest pain. The nurse suspects this client
may have which of the following conditions?
24. A 55-year old client underwent cataract removal with intraocular lens implant.
Nurse Oliver is giving the client discharge instructions. These instructions should a. Adult respiratory distress syndrome (ARDS)
include which of the following? b. Myocardial infarction (MI)
c. Pneumonia
a. Avoid lifting objects weighing more than 5 lb (2.25 kg). d. Tuberculosis
b. Lie on your abdomen when in bed
c. Keep rooms brightly lit. 36. Nurse Oliver is working in a out patient clinic. He has been alerted that there
d. Avoiding straining during bowel movement or bending at the waist. is an outbreak of tuberculosis (TB). Which of the following clients entering the
clinic today most likely to have TB?
25. George should be taught about testicular examinations during:
a. A 16-year-old female high school student
a. when sexual activity starts b. After age 69 b. A 33-year-old day-care worker
c. After age 40 d. Before age 20. c. A 43-yesr-old homeless man with a history of alcoholism
d. A 54-year-old businessman
26. A male client undergone a colon resection. While turning him, wound
dehiscence with evisceration occurs. Nurse Trish first response is to: 37. Virgie with a positive Mantoux test result will be sent for a chest X-ray. The
nurse is aware that which of the following reasons this is done?
a. Call the physician
b. Place a saline-soaked sterile dressing on the wound. a. To confirm the diagnosis
c. Take a blood pressure and pulse. b. To determine if a repeat skin test is needed
d. Pull the dehiscence closed. c. To determine the extent of lesions
d. To determine if this is a primary or secondary infection
27. Nurse Audrey is caring for a client who has suffered a severe
cerebrovascular accident. During routine assessment, the nurse notices Cheyne- 38. Kennedy with acute asthma showing inspiratory and expiratory wheezes and
Strokes respirations. Cheyne-strokes respirations are: a decreased forced expiratory volume should be treated with which of the
following classes of medication right away?
a. A progressively deeper breaths followed by shallower breaths with apneic
periods. a. Beta-adrenergic blockers b. Bronchodilators
b. Rapid, deep breathing with abrupt pauses between each breath. c. Inhaled steroids d. Oral steroids
c. Rapid, deep breathing and irregular breathing without pauses.
d. Shallow breathing with an increased respiratory rate. 39. Mr. Vasquez 56-year-old client with a 40-year history of smoking one to two
packs of cigarettes per day has a chronic cough producing thick sputum,
28. Nurse Bea is assessing a male client with heart failure. The breath sounds peripheral edema and cyanotic nail beds. Based on this information, he most
commonly auscultated in clients with heart failure are: likely has which of the following conditions?

a. Tracheal b. Fine crackles c. Coarse crackles d. Friction rubs a. Adult respiratory distress syndrome (ARDS)
b. Asthma
29. The nurse is caring for Kenneth experiencing an acute asthma attack. The c. Chronic obstructive bronchitis
client stops wheezing and breath sounds aren’t audible. The reason for this d. Emphysema
change is that:
Situation: Francis, age 46 is admitted to the hospital with diagnosis of Chronic
a. The attack is over. Lymphocytic Leukemia.
b. The airways are so swollen that no air cannot get through.
c. The swelling has decreased. 40. The treatment for patients with leukemia is bone marrow transplantation.
d. Crackles have replaced wheezes. Which statement about bone marrow transplantation is not correct?

30. Mike with epilepsy is having a seizure. During the active seizure phase, the a. The patient is under local anesthesia during the procedure
nurse should: b. The aspirated bone marrow is mixed with heparin.
c. The aspiration site is the posterior or anterior iliac crest.
a. Place the client on his back remove dangerous objects, and insert a bite block. d. The recipient receives cyclophosphamide (Cytoxan) for 4 consecutive days
b. Place the client on his side, remove dangerous objects, and insert a bite block. before the procedure.
c. Place the client o his back, remove dangerous objects, and hold down his
arms. 41. After several days of admission, Francis becomes disoriented and complains
d. Place the client on his side, remove dangerous objects, and protect his head. of frequent headaches. The nurse in-charge first action would be:

31. After insertion of a cheat tube for a pneumothorax, a client becomes a. Call the physician b. Document the patient’s status in his charts.
hypotensive with neck vein distention, tracheal shift, absent breath sounds, and c. Prepare oxygen treatment d. Raise the side rails
diaphoresis. Nurse Amanda suspects a tension pneumothorax has occurred.
What cause of tension pneumothorax should the nurse check for? 42. During routine care, Francis asks the nurse, “How can I be anemic if this
disease causes increased my white blood cell production?” The nurse in-charge
a. Infection of the lung. best response would be that the increased number of white blood cells (WBC) is:
b. Kinked or obstructed chest tube
c. Excessive water in the water-seal chamber a. Crowd red blood cells b. Are not responsible for the anemia.
d. Excessive chest tube drainage c. Uses nutrients from other cells d. Have an abnormally short life span of cells.

32. Nurse Maureen is talking to a male client, the client begins choking on his 43. Diagnostic assessment of Francis would probably not reveal:
lunch. He’s coughing forcefully. The nurse should:
a. Predominance of lymhoblasts b. Leukocytosis
a. Stand him up and perform the abdominal thrust maneuver from behind. c. Abnormal blast cells in the bone marrow d. Elevated thrombocyte counts
b. Lay him down, straddle him, and perform the abdominal thrust maneuver.
44. Robert, a 57-year-old client with acute arterial occlusion of the left leg 54. What is the most important postoperative instruction that nurse Kate must
undergoes an emergency embolectomy. Six hours later, the nurse isn’t able to give a client who has just returned from the operating room after receiving a
obtain pulses in his left foot using Doppler ultrasound. The nurse immediately subarachnoid block?
notifies the physician, and asks her to prepare the client for surgery. As the nurse
enters the client’s room to prepare him, he states that he won’t have any more a. "Avoid drinking liquids until the gag reflex returns."
surgery. Which of the following is the best initial response by the nurse? b. "Avoid eating milk products for 24 hours."
c. "Notify a nurse if you experience blood in your urine."
a. Explain the risks of not having the surgery d. "Remain supine for the time specified by the physician."
b. Notifying the physician immediately
c. Notifying the nursing supervisor 55. A male client suspected of having colorectal cancer will require which
d. Recording the client’s refusal in the nurses’ notes diagnostic study to confirm the diagnosis?

45. During the endorsement, which of the following clients should the on-duty a. Stool Hematest b. Carcinoembryonic antigen (CEA)
nurse assess first? c. Sigmoidoscopy d. Abdominal computed tomography (CT) scan

a. The 58-year-old client who was admitted 2 days ago with heart failure, blood 56. During a breast examination, which finding most strongly suggests that the
pressure of 126/76 mm Hg, and a respiratory rate of 22 breaths/minute. Luz has breast cancer?
b. The 89-year-old client with end-stage right-sided heart failure, blood pressure
of 78/50 mm Hg, and a “do not resuscitate” order a. Slight asymmetry of the breasts.
c. The 62-year-old client who was admitted 1 day ago with thrombophlebitis and b. A fixed nodular mass with dimpling of the overlying skin
is receiving L.V. heparin c. Bloody discharge from the nipple
d. The 75-year-old client who was admitted 1 hour ago with new-onset atrial d. Multiple firm, round, freely movable masses that change with the menstrual
fibrillation and is receiving L.V. dilitiazem (Cardizem) cycle

46. Honey, a 23-year old client complains of substernal chest pain and states 57. A female client with cancer is being evaluated for possible metastasis. Which
that her heart feels like “it’s racing out of the chest”. She reports no history of of the following is one of the most common metastasis sites for cancer cells?
cardiac disorders. The nurse attaches her to a cardiac monitor and notes sinus
tachycardia with a rate of 136beats/minutes. Breath sounds are clear and the a. Liver b. Colon c. Reproductive tract d. White blood cells (WBCs)
respiratory rate is 26 breaths/minutes. Which of the following drugs should the
nurse question the client about using? 58. Nurse Mandy is preparing a client for magnetic resonance imaging (MRI) to
confirm or rule out a spinal cord lesion. During the MRI scan, which of the
a. Barbiturates b. Opioids c. Cocaine d. Benzodiazepines following would pose a threat to the client?

47. A 51-year-old female client tells the nurse in-charge that she has found a a. The client lies still.
painless lump in her right breast during her monthly self-examination. Which b. The client asks questions.
assessment finding would strongly suggest that this client's lump is cancerous? c. The client hears thumping sounds.
d. The client wears a watch and wedding band.
a. Eversion of the right nipple and mobile mass
b. Nonmobile mass with irregular edges 59. Nurse Cecile is teaching a female client about preventing osteoporosis.
c. Mobile mass that is soft and easily delineated Which of the following teaching points is correct?
d. Nonpalpable right axillary lymph nodes
a. Obtaining an X-ray of the bones every 3 years is recommended to detect bone
48. A 35-year-old client with vaginal cancer asks the nurse, "What is the usual loss.
treatment for this type of cancer?" Which treatment should the nurse name? b. To avoid fractures, the client should avoid strenuous exercise.
c. The recommended daily allowance of calcium may be found in a wide variety
a. Surgery b. Chemotherapy c. Radiation d. Immunotherapy of foods.
d. Obtaining the recommended daily allowance of calcium requires taking a
49. Cristina undergoes a biopsy of a suspicious lesion. The biopsy report calcium supplement.
classifies the lesion according to the TNM staging system as follows: TIS, N0,
M0. What does this classification mean? 60. Before Jacob undergoes arthroscopy, the nurse reviews the assessment
findings for contraindications for this procedure. Which finding is a
a. No evidence of primary tumor, no abnormal regional lymph nodes, and no contraindication?
evidence of distant metastasis
b. Carcinoma in situ, no abnormal regional lymph nodes, and no evidence of a. Joint pain b. Joint deformity
distant metastasis c. Joint flexion of less than 50% d. Joint stiffness
c. Can't assess tumor or regional lymph nodes and no evidence of metastasis
d. Carcinoma in situ, no demonstrable metastasis of the regional lymph nodes, 61. Mr. Rodriguez is admitted with severe pain in the knees. Which form of
and ascending degrees of distant metastasis arthritis is characterized by urate deposits and joint pain, usually in the feet and
legs, and occurs primarily in men over age 30?
50. Lydia undergoes a laryngectomy to treat laryngeal cancer. When teaching
the client how to care for the neck stoma, the nurse should include which a. Septic arthritis b. Traumatic arthritis
instruction? c. Intermittent arthritis d. Gouty arthritis

a. "Keep the stoma uncovered." 62. A heparin infusion at 1,500 unit/hour is ordered for a 64-year-old client with
b. "Keep the stoma dry." stroke in evolution. The infusion contains 25,000 units of heparin in 500 ml of
c. "Have a family member perform stoma care initially until you get used to the saline solution. How many milliliters per hour should be given?
procedure."
d. "Keep the stoma moist." a. 15 ml/hour b. 30 ml/hour c. 45 ml/hour d. 50 ml/hour

51. A 37-year-old client with uterine cancer asks the nurse, "Which is the most 63. A 76-year-old male client had a thromboembolic right stroke; his left arm is
common type of cancer in women?" The nurse replies that it's breast cancer. swollen. Which of the following conditions may cause swelling after a stroke?
Which type of cancer causes the most deaths in women?
a. Elbow contracture secondary to spasticity
a. Breast cancer b. Lung cancer b. Loss of muscle contraction decreasing venous return
c. Brain cancer d. Colon and rectal cancer c. Deep vein thrombosis (DVT) due to immobility of the ipsilateral side
d. Hypoalbuminemia due to protein escaping from an inflamed glomerulus
52. Antonio with lung cancer develops Horner's syndrome when the tumor
invades the ribs and affects the sympathetic nerve ganglia. When assessing for
signs and symptoms of this syndrome, the nurse should note: 64. Heberden’s nodes are a common sign of osteoarthritis. Which of the following
statement is correct about this deformity?
a. miosis, partial eyelid ptosis, and anhidrosis on the affected side of the face.
b. chest pain, dyspnea, cough, weight loss, and fever. a. It appears only in men
c. arm and shoulder pain and atrophy of arm and hand muscles, both on the b. It appears on the distal interphalangeal joint
affected side. c. It appears on the proximal interphalangeal joint
d. hoarseness and dysphagia. d. It appears on the dorsolateral aspect of the interphalangeal joint.

53. Vic asks the nurse what PSA is. The nurse should reply that it stands for:
65. Which of the following statements explains the main difference between
a. prostate-specific antigen, which is used to screen for prostate cancer. rheumatoid arthritis and osteoarthritis?
b. protein serum antigen, which is used to determine protein levels.
c. pneumococcal strep antigen, which is a bacteria that causes pneumonia. a. Osteoarthritis is gender-specific, rheumatoid arthritis isn’t
d. Papanicolaou-specific antigen, which is used to screen for cervical cancer. b. Osteoarthritis is a localized disease rheumatoid arthritis is systemic
c. Osteoarthritis is a systemic disease, rheumatoid arthritis is localized
d. Osteoarthritis has dislocations and subluxations, rheumatoid arthritis doesn’t b. Dyspnea, tachycardia, and pallor
c. Nausea, vomiting, and anorexia
66. Mrs. Cruz uses a cane for assistance in walking. Which of the following d. Itching, rash, and jaundice
statements is true about a cane or other assistive devices?
79. In teaching a female client who is HIV-positive about pregnancy, the nurse
a. A walker is a better choice than a cane. would know more teaching is necessary when the client says:
b. The cane should be used on the affected side
c. The cane should be used on the unaffected side a. The baby can get the virus from my placenta."
d. A client with osteoarthritis should be encouraged to ambulate without the cane b. "I'm planning on starting on birth control pills."
c. "Not everyone who has the virus gives birth to a baby who has the virus."
67. A male client with type 1 diabetes is scheduled to receive 30 U of 70/30 d. "I'll need to have a C-section if I become pregnant and have a baby."
insulin. There is no 70/30 insulin available. As a substitution, the nurse may give
the client: 80. When preparing Judy with acquired immunodeficiency syndrome (AIDS) for
discharge to the home, the nurse should be sure to include which instruction?
a. 9 U regular insulin and 21 U neutral protamine Hagedorn (NPH).
b. 21 U regular insulin and 9 U NPH. a. "Put on disposable gloves before bathing."
c. 10 U regular insulin and 20 U NPH. b. "Sterilize all plates and utensils in boiling water."
d. 20 U regular insulin and 10 U NPH. c. "Avoid sharing such articles as toothbrushes and razors."
d. "Avoid eating foods from serving dishes shared by other family members."
68. Nurse Len should expect to administer which medication to a client with
gout? 81. Nurse Marie is caring for a 32-year-old client admitted with pernicious
anemia. Which set of findings should the nurse expect when assessing the
a. aspirin b. furosemide (Lasix) client?
c. colchicines d. calcium gluconate (Kalcinate)
a. Pallor, bradycardia, and reduced pulse pressure
69. Mr. Domingo with a history of hypertension is diagnosed with primary b. Pallor, tachycardia, and a sore tongue
hyperaldosteronism. This diagnosis indicates that the client's hypertension is c. Sore tongue, dyspnea, and weight gain
caused by excessive hormone secretion from which of the following glands? d. Angina, double vision, and anorexia

a. Adrenal cortex b. Pancreas c. Adrenal medulla d. Parathyroid 82. After receiving a dose of penicillin, a client develops dyspnea and
hypotension. Nurse Celestina suspects the client is experiencing anaphylactic
70. For a diabetic male client with a foot ulcer, the doctor orders bed rest, a shock. What should the nurse do first?
wetto- dry dressing change every shift, and blood glucose monitoring before
meals and bedtime. Why are wet-to-dry dressings used for this client? a. Page an anesthesiologist immediately and prepare to intubate the client.
b. Administer epinephrine, as prescribed, and prepare to intubate the client if
a. They contain exudate and provide a moist wound environment. necessary.
b. They protect the wound from mechanical trauma and promote healing. c. Administer the antidote for penicillin, as prescribed, and continue to monitor
c. They debride the wound and promote healing by secondary intention. the client's vital signs.
d. They prevent the entrance of microorganisms and minimize wound discomfort. d. Insert an indwelling urinary catheter and begin to infuse I.V. fluids as ordered.

71. Nurse Zeny is caring for a client in acute addisonian crisis. Which laboratory 83. Mr. Marquez with rheumatoid arthritis is about to begin aspirin therapy to
data would the nurse expect to find? reduce inflammation. When teaching the client about aspirin, the nurse discusses
adverse reactions to prolonged aspirin therapy. These include:
a. Hyperkalemia b. Reduced blood urea nitrogen (BUN)
c. Hypernatremia d. Hyperglycemia a. weight gain. b. fine motor tremors
c. respiratory acidosis. d. bilateral hearing loss.
72. A client is admitted for treatment of the syndrome of inappropriate antidiuretic
hormone (SIADH). Which nursing intervention is appropriate? 84. A 23-year-old client is diagnosed with human immunodeficiency virus (HIV).
After recovering from the initial shock of the diagnosis, the client expresses a
a. Infusing I.V. fluids rapidly as ordered desire to learn as much as possible about HIV and acquired immunodeficiency
b. Encouraging increased oral intake syndrome (AIDS). When teaching the client about the immune system, the nurse
c. Restricting fluids states that adaptive immunity is provided by which type of white blood cell?
d. Administering glucose-containing I.V. fluids as ordered
a. Neutrophil b. Basophil c. Monocyte d. Lymphocyte
73. A female client tells nurse Nikki that she has been working hard for the last 3
months to control her type 2 diabetes mellitus with diet and exercise. To 85. In an individual with Sjögren's syndrome, nursing care should focus on:
determine the effectiveness of the client's efforts, the nurse should check:
a. moisture replacement. b. electrolyte balance.
a. urine glucose level. b. fasting blood glucose level. c. nutritional supplementation. d. arrhythmia management.
c. serum fructosamine level. d. glycosylated hemoglobin level.
86. During chemotherapy for lymphocytic leukemia, Mathew develops abdominal
74. Nurse Trinity administered neutral protamine Hagedorn (NPH) insulin to a pain, fever, and "horse barn" smelling diarrhea. It would be most important for the
diabetic client at 7 a.m. At what time would the nurse expect the client to be most nurse to advise the physician to order:
at risk for a hypoglycemic reaction?
a. enzyme-linked immunosuppressant assay (ELISA) test.
a. 10:00 am b. Noon c. 4:00 pm d. 10:00 pm b. electrolyte panel and hemogram.
c. stool for Clostridium difficile test.
75. The adrenal cortex is responsible for producing which substances? d. flat plate X-ray of the abdomen.

a. Glucocorticoids and androgens 87. A male client seeks medical evaluation for fatigue, night sweats, and a 20-lb
b. Catecholamines and epinephrine weight loss in 6 weeks. To confirm that the client has been infected with the
c. Mineralocorticoids and catecholamines human immunodeficiency virus (HIV), the nurse expects the physician to order:
d. Norepinephrine and epinephrine
a. E-rosette immunofluorescence.
76. On the third day after a partial thyroidectomy, Proserfina exhibits muscle b. quantification of T-lymphocytes.
twitching and hyperirritability of the nervous system. When questioned, the client c. enzyme-linked immunosorbent assay (ELISA).
reports numbness and tingling of the mouth and fingertips. Suspecting a d. Western blot test with ELISA.
lifethreatening electrolyte disturbance, the nurse notifies the surgeon
immediately. Which electrolyte disturbance most commonly follows thyroid 88. A complete blood count is commonly performed before a Joe goes into
surgery? surgery. What does this test seek to identify?

a. Hypocalcemia b. Hyponatremia c. Hyperkalemia d. Hypermagnesemia a. Potential hepatic dysfunction indicated by decreased blood urea nitrogen
(BUN) and creatinine levels
77. Which laboratory test value is elevated in clients who smoke and can't be b. Low levels of urine constituents normally excreted in the urine
used as a general indicator of cancer? c. Abnormally low hematocrit (HCT) and hemoglobin (Hb) levels
d. Electrolyte imbalance that could affect the blood's ability to coagulate properly
a. Acid phosphatase level b. Serum calcitonin level
c. Alkaline phosphatase level d. Carcinoembryonic antigen level 89. While monitoring a client for the development of disseminated intravascular
coagulation (DIC), the nurse should take note of what assessment parameters?
78. Francis with anemia has been admitted to the medical-surgical unit. Which
assessment findings are characteristic of iron-deficiency anemia? a. Platelet count, prothrombin time, and partial thromboplastin time
b. Platelet count, blood glucose levels, and white blood cell (WBC) count
a. Nights sweats, weight loss, and diarrhea c. Thrombin time, calcium levels, and potassium levels
d. Fibrinogen level, WBC, and platelet count 99. Nurse Deric is supervising a group of elderly clients in a residential home
setting. The nurse knows that the elderly are at greater risk of developing
90. When taking a dietary history from a newly admitted female client, Nurse Len sensory deprivation for what reason?
should remember that which of the following foods is a common allergen?
a. Increased sensitivity to the side effects of medications.
a. Bread b. Carrots c. Orange d. Strawberries b. Decreased visual, auditory, and gustatory abilities.
c. Isolation from their families and familiar surroundings.
91. Nurse John is caring for clients in the outpatient clinic. Which of the following d. Decrease musculoskeletal function and mobility.
phone calls should the nurse return first?
100. A male client with emphysema becomes restless and confused. What step
a. A client with hepatitis A who states, “My arms and legs are itching.” should nurse Jasmine take next?
b. A client with cast on the right leg who states, “I have a funny feeling in my right
leg.” a. Encourage the client to perform pursed lip breathing.
c. A client with osteomyelitis of the spine who states, “I am so nauseous that I b. Check the client’s temperature.
can’t eat.” c. Assess the client’s potassium level.
d. A client with rheumatoid arthritis who states, “I am having trouble sleeping.” d. Increase the client’s oxygen flow rate.
RATIO 3
92. Nurse Sarah is caring for clients on the surgical floor and has just received 1. Answer: (C) Loose, bloody
report from the previous shift. Which of the following clients should the nurse see Rationale: Normal bowel function and soft-formed stool usually do not occur until
first? around the seventh day following surgery. The stool consistency is related to how
much water is being absorbed.
a. A 35-year-old admitted three hours ago with a gunshot wound; 1.5 cm area of
dark drainage noted on the dressing. 2. Answer: (A) On the client’s right side
b. A 43-year-old who had a mastectomy two days ago; 23 ml of serosanguinous Rationale: The client has left visual field blindness. The client will see only from
fluid noted in the Jackson-Pratt drain. the right side.
c. A 59-year-old with a collapsed lung due to an accident; no drainage noted in
the previous eight hours. 3. Answer: (C) Check respirations, stabilize spine, and check circulation
d. A 62-year-old who had an abdominal-perineal resection three days ago; client Rationale: Checking the airway would be priority, and a neck injury should be
complaints of chills. suspected.

93. Nurse Eve is caring for a client who had a thyroidectomy 12 hours ago for 4. Answer: (D) Decreasing venous return through vasodilation.
treatment of Grave’s disease. The nurse would be most concerned if which of the Rationale: The significant effect of nitroglycerin is vasodilation and decreased
following was observed? venous return, so the heart does not have to work hard.

a. Blood pressure 138/82, respirations 16, oral temperature 99 degrees 5. Answer: (A) Call for help and note the time.
Fahrenheit. Rationale: Having established, by stimulating the client, that the client is
b. The client supports his head and neck when turning his head to the right. unconscious rather than sleep, the nurse should immediately call for help. This
c. The client spontaneously flexes his wrist when the blood pressure is obtained. may be done by dialing the operator from the client’s phone and giving the
d. The client is drowsy and complains of sore throat. hospital code for cardiac arrest and the client’s room number to the operator, of if
the phone is not available, by pulling the emergency call button. Noting the time
94. Julius is admitted with complaints of severe pain in the lower right quadrant of is important baseline information for cardiac arrest procedure.
the abdomen. To assist with pain relief, the nurse should take which of the
following actions? 6. Answer: (C) Make sure that the client takes food and medications at
prescribed intervals.
a. Encourage the client to change positions frequently in bed. Rationale: Food and drug therapy will prevent the accumulation of hydrochloric
b. Administer Demerol 50 mg IM q 4 hours and PRN. acid, or will neutralize and buffer the acid that does accumulate.
c. Apply warmth to the abdomen with a heating pad.
d. Use comfort measures and pillows to position the client. 7. Answer: (B) Continue treatment as ordered.
Rationale: The effects of heparin are monitored by the PTT is normally 30 to 45
95. Nurse Tina prepares a client for peritoneal dialysis. Which of the following seconds; the therapeutic level is 1.5 to 2 times the normal level.
actions should the nurse take first?
8. Answer: (B) In the operating room.
a. Assess for a bruit and a thrill. b. Warm the dialysate solution. Rationale: The stoma drainage bag is applied in the operating room. Drainage
c. Position the client on the left side. d. Insert a Foley catheter from the ileostomy contains secretions that are rich in digestive enzymes and
highly irritating to the skin. Protection of the skin from the effects of these
96. Nurse Jannah teaches an elderly client with right-sided weakness how to use enzymes is begun at once. Skin exposed to these enzymes even for a short time
cane. Which of the following behaviors, if demonstrated by the client to the nurse, becomes reddened, painful, and excoriated.
indicates that the teaching was effective?
9. Answer: (B) Flat on back.
a. The client holds the cane with his right hand, moves the can forward followed Rationale: To avoid the complication of a painful spinal headache that can last for
by the right leg, and then moves the left leg. several days, the client is kept in flat in a supine position for approximately 4 to
b. The client holds the cane with his right hand, moves the cane forward followed 12 hours postoperatively. Headaches are believed to be causes by the seepage
by his left leg, and then moves the right leg. of cerebral spinal fluid from the puncture site. By keeping the client flat, cerebral
c. The client holds the cane with his left hand, moves the cane forward followed spinal fluid pressures are equalized, which avoids trauma to the neurons.
by the right leg, and then moves the left leg.
d. The client holds the cane with his left hand, moves the cane forward followed 10. Answer: (C) The client is oriented when aroused from sleep, and goes back
by his left leg, and then moves the right leg. to sleep immediately.
Rationale: This finding suggest that the level of consciousness is decreasing.
97. An elderly client is admitted to the nursing home setting. The client is
occasionally confused and her gait is often unsteady. Which of the following 11. Answer: (A) Altered mental status and dehydration
actions, if taken by the nurse, is most appropriate? Rationale: Fever, chills, hemortysis, dyspnea, cough, and pleuritic chest pain are
the common symptoms of pneumonia, but elderly clients may first appear with
a. Ask the woman’s family to provide personal items such as photos or only an altered lentil status and dehydration due to a blunted immune response.
mementos.
b. Select a room with a bed by the door so the woman can look down the hall. 12. Answer: (B) Chills, fever, night sweats, and hemoptysis
c. Suggest the woman eat her meals in the room with her roommate. Rationale: Typical signs and symptoms are chills, fever, night sweats, and
d. Encourage the woman to ambulate in the halls twice a day. hemoptysis. Chest pain may be present from coughing, but isn’t usual. Clients
with TB typically have low-grade fevers, not higher than 102°F (38.9°C). Nausea,
98. Nurse Evangeline teaches an elderly client how to use a standard aluminum headache, and photophobia aren’t usual TB symptoms.
walker. Which of the following behaviors, if demonstrated by the client, indicates
that the nurse’s teaching was effective? 13. Answer:(A) Acute asthma
Rationale: Based on the client’s history and symptoms, acute asthma is the most
a. The client slowly pushes the walker forward 12 inches, then takes small steps likely diagnosis. He’s unlikely to have bronchial pneumonia without a productive
forward while leaning on the walker. cough and fever and he’s too young to have developed (COPD) and
b. The client lifts the walker, moves it forward 10 inches, and then takes several emphysema.
small steps forward.
c. The client supports his weight on the walker while advancing it forward, then 14. Answer: (B) Respiratory arrest
takes small steps while balancing on the walker. Rationale: Narcotics can cause respiratory arrest if given in large quantities. It’s
d. The client slides the walker 18 inches forward, then takes small steps while unlikely the client will have asthma attack or a seizure or wake up on his own.
holding onto the walker for balance.
15. Answer: (D) Decreased vital capacity
Rationale: Reduction in vital capacity is a normal physiologic changes include
decreased elastic recoil of the lungs, fewer functional capillaries in the alveoli, 30. Answer: (D) Place the client on his side, remove dangerous objects, and
and an increased in residual volume. protect his head.
Rationale: During the active seizure phase, initiate precautions by placing the
16. Answer: (C) Presence of premature ventricular contractions (PVCs) on a client on his side, removing dangerous objects, and protecting his head from
cardiac monitor. injury. A bite block should never be inserted during the active seizure phase.
Rationale: Lidocaine drips are commonly used to treat clients whose arrhythmias Insertion can break the teeth and lead to aspiration.
haven’t been controlled with oral medication and who are having PVCs that are
visible on the cardiac monitor. SaO2, blood pressure, and ICP are important 31. Answer: (B) Kinked or obstructed chest tube
factors but aren’t as significant as PVCs in the situation. Rationales: Kinking and blockage of the chest tube is a common cause of a
tension pneumothorax. Infection and excessive drainage won’t cause a tension
17. Answer: (B) Avoid foods high in vitamin K pneumothorax. Excessive water won’t affect the chest tube drainage.
Rationale: The client should avoid consuming large amounts of vitamin K
because vitamin K can interfere with anticoagulation. The client may need to 32. Answer: (D) Stay with him but not intervene at this time.
report diarrhea, but isn’t effect of taking an anticoagulant. An electric razor-not a Rationale: If the client is coughing, he should be able to dislodge the object or
straight razor-should be used to prevent cuts that cause bleeding. Aspirin may cause a complete obstruction. If complete obstruction occurs, the nurse should
increase the risk of bleeding; acetaminophen should be used to pain relief. perform the abdominal thrust maneuver with the client standing. If the client is
unconscious, she should lay him down. A nurse should never leave a choking
18. Answer: (C) Clipping the hair in the area client alone.
Rationale: Hair can be a source of infection and should be removed by clipping.
Shaving the area can cause skin abrasions and depilatories can irritate the skin. 33. Answer: (B) Current health promotion activities
Rationale: Recognizing an individual’s positive health measures is very useful.
19. Answer: (A) Bone fracture General health in the previous 10 years is important, however, the current
Rationale: Bone fracture is a major complication of osteoporosis that results activities of an 84 year old client are most significant in planning care. Family
when loss of calcium and phosphate increased the fragility of bones. Estrogen history of disease for a client in later years is of minor significance. Marital status
deficiencies result from menopause-not osteoporosis. Calcium and vitamin D information may be important for discharge planning but is not as significant for
supplements may be used to support normal bone metabolism, But a negative addressing the immediate medical problem.
calcium balance isn’t a complication of osteoporosis. Dowager’s hump results
from bone fractures. It develops when repeated vertebral fractures increase 34. Answer: (C) Place the client in a side lying position, with the head of the bed
spinal curvature. lowered.
Rationale: The client should be positioned in a side-lying position with the head
20. Answer: (C) Changes from previous examinations. of the bed lowered to prevent aspiration. A small amount of toothpaste should be
Rationale: Women are instructed to examine themselves to discover changes used and the mouth swabbed or suctioned to remove pooled secretions. Lemon
that have occurred in the breast. Only a physician can diagnose lumps that are glycerin can be drying if used for extended periods. Brushing the teeth with the
cancerous, areas of thickness or fullness that signal the presence of a client lying supine may lead to aspiration. Hydrogen peroxide is caustic to tissues
malignancy, or masses that are fibrocystic as opposed to malignant. and should not be used.

21. Answer: (C) Balance the client’s periods of activity and rest. 35. Answer: (C) Pneumonia
Rationale: A client with hyperthyroidism needs to be encouraged to balance Rationale: Fever productive cough and pleuritic chest pain are common signs
periods of activity and rest. Many clients with hyperthyroidism are hyperactive and symptoms of pneumonia. The client with ARDS has dyspnea and hypoxia
and complain of feeling very warm. with worsening hypoxia over time, if not treated aggressively. Pleuritic chest pain
varies with respiration, unlike the constant chest pain during an MI; so this client
22. Answer: (B) Increase his activity level. most likely isn’t having an MI. the client with TB typically has a cough producing
Rationale: The client should be encouraged to increase his activity level. blood-tinged sputum. A sputum culture should be obtained to confirm the nurse’s
Maintaining an ideal weight; following a low-cholesterol, low sodium diet; and suspicions.
avoiding stress are all important factors in decreasing the risk of atherosclerosis.
36. Answer: (C) A 43-yesr-old homeless man with a history of alcoholism
23. Answer: (A) Laminectomy Rationale: Clients who are economically disadvantaged, malnourished, and have
Rationale: The client who has had spinal surgery, such as laminectomy, must be reduced immunity, such as a client with a history of alcoholism, are at extremely
log rolled to keep the spinal column straight when turning. Thoracotomy and high risk for developing TB. A high school student, daycare worker, and
cystectomy may turn themselves or may be assisted into a comfortable position. businessman probably have a much low risk of contracting TB.
Under normal circumstances, hemorrhoidectomy is an outpatient procedure, and
the client may resume normal activities immediately after surgery. 37. Answer: (C ) To determine the extent of lesions
Rationale: If the lesions are large enough, the chest X-ray will show their
24. Answer: (D) Avoiding straining during bowel movement or bending at the presence in the lungs. Sputum culture confirms the diagnosis. There can be
waist. false-positive and false-negative skin test results. A chest X-ray can’t determine if
Rationale: The client should avoid straining, lifting heavy objects, and coughing this is a primary or secondary infection.
harshly because these activities increase intraocular pressure. Typically, the
client is instructed to avoid lifting objects weighing more than 15 lb (7kg) – not 38. Answer: (B) Bronchodilators
5lb. instruct the client when lying in bed to lie on either the side or back. The Rationale: Bronchodilators are the first line of treatment for asthma because
client should avoid bright light by wearing sunglasses. broncho-constriction is the cause of reduced airflow. Beta adrenergic blockers
aren’t used to treat asthma and can cause bronchoconstriction. Inhaled oral
25. Answer: (D) Before age 20. steroids may be given to reduce the inflammation but aren’t used for emergency
Rationale: Testicular cancer commonly occurs in men between ages 20 and 30. relief.
A male client should be taught how to perform testicular selfexamination before
age 20, preferably when he enters his teens. 39. Answer: (C) Chronic obstructive bronchitis
Rationale: Because of this extensive smoking history and symptoms the client
26. Answer: (B) Place a saline-soaked sterile dressing on the wound. most likely has chronic obstructive bronchitis. Client with ARDS have acute
Rationale: The nurse should first place saline-soaked sterile dressings on the symptoms of hypoxia and typically need large amounts of oxygen. Clients with
open wound to prevent tissue drying and possible infection. Then the nurse asthma and emphysema tend not to have chronic cough or peripheral edema.
should call the physician and take the client’s vital signs. The dehiscence needs
to be surgically closed, so the nurse should never try to close it. 40. Answer: (A) The patient is under local anesthesia during the procedure
Rationale: Before the procedure, the patient is administered with drugs that
27. Answer: (A) A progressively deeper breaths followed by shallower breaths would help to prevent infection and rejection of the transplanted cells such as
with apneic periods. antibiotics, cytotoxic, and corticosteroids. During the transplant, the patient is
Rationale: Cheyne-Strokes respirations are breaths that become progressively placed under general anesthesia.
deeper fallowed by shallower respirations with apneas periods. Biot’s respirations
are rapid, deep breathing with abrupt pauses between each breath, and equal 41. Answer: (D) Raise the side rails
depth between each breath. Kussmaul’s respirations are rapid, deep breathing Rationale: A patient who is disoriented is at risk of falling out of bed. The initial
without pauses. Tachypnea is shallow breathing with increased respiratory rate. action of the nurse should be raising the side rails to ensure patients safety.

28. Answer: (B) Fine crackles 42. Answer: (A) Crowd red blood cells
Rationale: Fine crackles are caused by fluid in the alveoli and commonly occur in Rationale: The excessive production of white blood cells crowd out red blood
clients with heart failure. Tracheal breath sounds are auscultated over the cells production which causes anemia to occur.
trachea. Coarse crackles are caused by secretion accumulation in the airways.
Friction rubs occur with pleural inflammation. 43. Answer: (B) Leukocytosis
Rationale: Chronic Lymphocytic leukemia (CLL) is characterized by increased
29. Answer: (B) The airways are so swollen that no air cannot get through production of leukocytes and lymphocytes resulting in leukocytosis, and
Rationale: During an acute attack, wheezing may stop and breath sounds proliferation of these cells within the bone marrow, spleen and liver.
become inaudible because the airways are so swollen that air can’t get through.
If the attack is over and swelling has decreased, there would be no more 44. Answer: (A) Explain the risks of not having the surgery
wheezing and less emergent concern. Crackles do not replace wheezes during Rationale: The best initial response is to explain the risks of not having the
an acute asthma attack. surgery. If the client understands the risks but still refuses the nurse should notify
the physician and the nurse supervisor and then record the client’s refusal in the Rationale: A fixed nodular mass with dimpling of the overlying skin is common
nurses’ notes. during late stages of breast cancer. Many women have slightly asymmetrical
breasts. Bloody nipple discharge is a sign of intraductal papilloma, a benign
45. Answer: (D) The 75-year-old client who was admitted 1 hour ago with new- condition. Multiple firm, round, freely movable masses that change with the
onset atrial fibrillation and is receiving L.V. dilitiazem (Cardizem) menstrual cycle indicate fibrocystic breasts, a benign condition.
Rationale: The client with atrial fibrillation has the greatest potential to become
unstable and is on L.V. medication that requires close monitoring. After 57. Answer: (A) Liver
assessing this client, the nurse should assess the client with thrombophlebitis Rationale: The liver is one of the five most common cancer metastasis sites. The
who is receiving a heparin infusion, and then the 58- year-old client admitted 2 others are the lymph nodes, lung, bone, and brain. The colon, reproductive tract,
days ago with heart failure (his signs and symptoms are resolving and don’t and WBCs are occasional metastasis sites.
require immediate attention). The lowest priority is the 89-year-old with end stage
right-sided heart failure, who requires time-consuming supportive measures. 58. Answer: (D) The client wears a watch and wedding band.
Rationale: During an MRI, the client should wear no metal objects, such as
46. Answer: (C) Cocaine jewelry, because the strong magnetic field can pull on them, causing injury to the
Rationale: Because of the client’s age and negative medical history, the nurse client and (if they fly off) to others. The client must lie still during the MRI but can
should question her about cocaine use. Cocaine increases myocardial oxygen talk to those performing the test by way of the microphone inside the scanner
consumption and can cause coronary artery spasm, leading to tachycardia, tunnel. The client should hear thumping sounds, which are caused by the sound
ventricular fibrillation, myocardial ischemia, and myocardial infarction. Barbiturate waves thumping on the magnetic field.
overdose may trigger respiratory depression and slow pulse. Opioids can cause
marked respiratory depression, while benzodiazepines can cause drowsiness 59. Answer: (C) The recommended daily allowance of calcium may be found in a
and confusion. wide variety of foods.
Rationale: Premenopausal women require 1,000 mg of calcium per day.
47. Answer: (B) Nonmobile mass with irregular edges Postmenopausal women require 1,500 mg per day. It's often, though not always,
Rationale: Breast cancer tumors are fixed, hard, and poorly delineated with possible to get the recommended daily requirement in the foods we eat.
irregular edges. A mobile mass that is soft and easily delineated is most often a Supplements are available but not always necessary. Osteoporosis doesn't show
fluid-filled benign cyst. Axillary lymph nodes may or may not be palpable on initial up on ordinary X-rays until 30% of the bone loss has occurred. Bone
detection of a cancerous mass. Nipple retraction — not eversion — may be a densitometry can detect bone loss of 3% or less. This test is sometimes
sign of cancer. recommended routinely for women over 35 who are at risk. Strenuous exercise
won't cause fractures.
48. Answer: (C) Radiation
Rationale: The usual treatment for vaginal cancer is external or intravaginal 60. Answer: (C) Joint flexion of less than 50%
radiation therapy. Less often, surgery is performed. Chemotherapy typically is Rationale: Arthroscopy is contraindicated in clients with joint flexion of less than
prescribed only if vaginal cancer is diagnosed in an early stage, which is rare. 50% because of technical problems in inserting the instrument into the joint to
Immunotherapy isn't used to treat vaginal cancer. see it clearly. Other contraindications for this procedure include skin and wound
infections. Joint pain may be an indication, not a contraindication, for
49. Answer: (B) Carcinoma in situ, no abnormal regional lymph nodes, and no arthroscopy. Joint deformity and joint stiffness aren't contraindications for this
evidence of distant metastasis procedure.
Rationale: TIS, N0, M0 denotes carcinoma in situ, no abnormal regional lymph
nodes, and no evidence of distant metastasis. No evidence of primary tumor, no 61. Answer: (D) Gouty arthritis
abnormal regional lymph nodes, and no evidence of distant metastasis is Rationale: Gouty arthritis, a metabolic disease, is characterized by urate deposits
classified as T0, N0, M0. If the tumor and regional lymph nodes can't be and pain in the joints, especially those in the feet and legs. Urate
assessed and no evidence of metastasis exists, the lesion is classified as TX, deposits don't occur in septic or traumatic arthritis. Septic arthritis results from
NX, M0. A progressive increase in tumor size, no demonstrable metastasis of the bacterial invasion of a joint and leads to inflammation of the synovial
regional lymph nodes, and ascending degrees of distant metastasis is classified lining. Traumatic arthritis results from blunt trauma to a joint or ligament.
as T1, T2, T3, or T4; N0; and M1, M2, or M3. Intermittent arthritis is a rare, benign condition marked by regular, recurrent joint
effusions, especially in the knees.
50. Answer: (D) "Keep the stoma moist."
Rationale: The nurse should instruct the client to keep the stoma moist, such as 62. Answer: (B) 30 ml/hou
by applying a thin layer of petroleum jelly around the edges, because a dry stoma Rationale: An infusion prepared with 25,000 units of heparin in 500 ml of saline
may become irritated. The nurse should recommend placing a stoma bib over the solution yields 50 units of heparin per milliliter of solution. The equation is set up
stoma to filter and warm air before it enters the stoma. The client should begin as 50 units times X (the unknown quantity) equals 1,500 units/hour, X equals 30
performing stoma care without assistance as soon as possible to gain ml/hour.
independence in self-care activities.
63. Answer: (B) Loss of muscle contraction decreasing venous return
51. Answer: (B) Lung cancer Rationale: In clients with hemiplegia or hemiparesis loss of muscle contraction
Rationale: Lung cancer is the most deadly type of cancer in both women and decreases venous return and may cause swelling of the affected extremity.
men. Breast cancer ranks second in women, followed (in descending order) by Contractures, or bony calcifications may occur with a stroke, but don’t appear
colon and rectal cancer, pancreatic cancer, ovarian cancer, uterine cancer, with swelling. DVT may develop in clients with a stroke but is more likely to occur
lymphoma, leukemia, liver cancer, brain cancer, stomach cancer, and multiple in the lower extremities. A stroke isn’t linked to protein loss.
myeloma.
64. Answer: (B) It appears on the distal interphalangeal joint
52. Answer: (A) miosis, partial eyelid ptosis, and anhidrosis on the affected side Rationale: Heberden’s nodes appear on the distal interphalageal joint on both
of the face. men and women. Bouchard’s node appears on the dorsolateral aspect of the
Rationale: Horner's syndrome, which occurs when a lung tumor invades the ribs proximal interphalangeal joint.
and affects the sympathetic nerve ganglia, is characterized by miosis, partial
eyelid ptosis, and anhidrosis on the affected side of the face. Chest pain, 65. Answer: (B) Osteoarthritis is a localized disease rheumatoid arthritis is
dyspnea, cough, weight loss, and fever are associated with pleural tumors. Arm systemic
and shoulder pain and atrophy of the arm and hand muscles on the affected side Rationale: Osteoarthritis is a localized disease, rheumatoid arthritis is systemic.
suggest Pancoast's tumor, a lung tumor involving the first thoracic and eighth Osteoarthritis isn’t gender-specific, but rheumatoid arthritis is. Clients have
cervical nerves within the brachial plexus. Hoarseness in a client with lung dislocations and subluxations in both disorders.
cancer suggests that the tumor has extended to the recurrent laryngeal nerve;
dysphagia suggests that the lung tumor is compressing the esophagus. 66. Answer: (C) The cane should be used on the unaffected side
Rationale: A cane should be used on the unaffected side. A client with
53. Answer: (A) prostate-specific antigen, which is used to screen for prostate osteoarthritis should be encouraged to ambulate with a cane, walker, or other
cancer. assistive device as needed; their use takes weight and stress off joints.
Rationale: PSA stands for prostate-specific antigen, which is used to screen for
prostate cancer. The other answers are incorrect. 67. Answer: (A) a. 9 U regular insulin and 21 U neutral protamine Hagedorn
(NPH).
54. Answer: (D) "Remain supine for the time specified by the physician." Rationale: A 70/30 insulin preparation is 70% NPH and 30% regular insulin.
Rationale: The nurse should instruct the client to remain supine for the time Therefore, a correct substitution requires mixing 21 U of NPH and 9 U of regular
specified by the physician. Local anesthetics used in a subarachnoid block don't insulin. The other choices are incorrect dosages for the prescribed insulin.
alter the gag reflex. No interactions between local anesthetics and food occur.
Local anesthetics don't cause hematuria. 68. Answer: (C) colchicines
Rationale: A disease characterized by joint inflammation (especially in the great
55. Answer: (C) Sigmoidoscopy toe), gout is caused by urate crystal deposits in the joints. The physician
Rationale: Used to visualize the lower GI tract, sigmoidoscopy and proctoscopy prescribes colchicine to reduce these deposits and thus ease joint inflammation.
aid in the detection of two-thirds of all colorectal cancers. Stool Hematest detects Although aspirin is used to reduce joint inflammation and pain in clients with
blood, which is a sign of colorectal cancer; however, the test doesn't confirm the osteoarthritis and rheumatoid arthritis, it isn't indicated for gout because it has no
diagnosis. CEA may be elevated in colorectal cancer but isn't considered a effect on urate crystal formation. Furosemide, a diuretic, doesn't relieve gout.
confirming test. An abdominal CT scan is used to stage the presence of Calcium gluconate is used to reverse a negative calcium balance and relieve
colorectal cancer. muscle cramps, not to treat gout.

56. Answer: (B) A fixed nodular mass with dimpling of the overlying skin 69. Answer: (A) Adrenal cortex
Rationale: Excessive secretion of aldosterone in the adrenal cortex is responsible weakness; fatigue; and paresthesia of the hands and feet. Bradycardia, reduced
for the client's hypertension. This hormone acts on the renal tubule, where it pulse pressure, weight gain, and double vision aren't characteristic findings in
promotes reabsorption of sodium and excretion of potassium and hydrogen ions. pernicious anemia.
The pancreas mainly secretes hormones involved in fuel metabolism. The
adrenal medulla secretes the catecholamines — epinephrine and 82. Answer: (B) Administer epinephrine, as prescribed, and prepare to intubate
norepinephrine. The parathyroids secrete parathyroid hormone. the client if necessary.
Rationale: To reverse anaphylactic shock, the nurse first should administer
70. Answer: (C) They debride the wound and promote healing by secondary epinephrine, a potent bronchodilator as prescribed. The physician is likely to
intention order additional medications, such as antihistamines and corticosteroids; if these
Rationale: For this client, wet-to-dry dressings are most appropriate because medications don't relieve the respiratory compromise associated with
they clean the foot ulcer by debriding exudate and necrotic tissue, thus promoting anaphylaxis, the nurse should prepare to intubate the client. No antidote for
healing by secondary intention. Moist, transparent dressings contain exudate and penicillin exists; however, the nurse should continue to monitor the client's vital
provide a moist wound environment. Hydrocolloid dressings prevent the entrance signs. A client who remains hypotensive may need fluid resuscitation and fluid
of microorganisms and minimize wound discomfort. Dry sterile dressings protect intake and output monitoring; however, administering epinephrine is the first
the wound from mechanical trauma and promote healing. priority.

71. Answer: (A) Hyperkalemia 83. Answer: (D) bilateral hearing loss.
Rationale: In adrenal insufficiency, the client has hyperkalemia due to reduced Rationale: Prolonged use of aspirin and other salicylates sometimes causes
aldosterone secretion. BUN increases as the glomerular filtration rate is reduced. bilateral hearing loss of 30 to 40 decibels. Usually, this adverse effect resolves
Hyponatremia is caused by reduced aldosterone secretion. Reduced cortisol within 2 weeks after the therapy is discontinued. Aspirin doesn't lead to weight
secretion leads to impaired glyconeogenesis and a reduction of glycogen in the gain or fine motor tremors. Large or toxic salicylate doses may cause respiratory
liver and muscle, causing hypoglycemia. alkalosis, not respiratory acidosis.

72. Answer: (C) Restricting fluids 84. Answer: (D) Lymphocyte


Rationale: To reduce water retention in a client with the SIADH, the nurse should Rationale: The lymphocyte provides adaptive immunity — recognition of a foreign
restrict fluids. Administering fluids by any route would further increase the client's antigen and formation of memory cells against the antigen.
already heightened fluid load. Adaptive immunity is mediated by B and T lymphocytes and can be acquired
actively or passively. The neutrophil is crucial to phagocytosis. The basophil
73. Answer: (D) glycosylated hemoglobin level. plays an important role in the release of inflammatory mediators. The monocyte
Rationale: Because some of the glucose in the bloodstream attaches to some of functions in phagocytosis and monokine production.
the hemoglobin and stays attached during the 120-day life span of red blood
cells, glycosylated hemoglobin levels provide information about blood glucose 85. Answer: (A) moisture replacement.
levels during the previous 3 months. Fasting blood glucose and urine glucose Rationale: Sjogren's syndrome is an autoimmune disorder leading to progressive
levels only give information about glucose levels at the point in time when they loss of lubrication of the skin, GI tract, ears, nose, and vagina. Moisture
were obtained. Serum fructosamine levels provide information about blood replacement is the mainstay of therapy. Though malnutrition and electrolyte
glucose control over the past 2 to 3 weeks. imbalance may occur as a result of Sjogren's syndrome's effect on the GI tract, it
isn't the predominant problem. Arrhythmias aren't a problem associated with
74. Answer: (C) 4:00 pm Sjogren's syndrome.
Rationale: NPH is an intermediate-acting insulin that peaks 8 to 12 hours after
administration. Because the nurse administered NPH insulin at 7 a.m., the client 86. Answer: (C) stool for Clostridium difficile test.
is at greatest risk for hypoglycemia from 3 p.m. to 7 p.m. Rationale: Immunosuppressed clients — for example, clients receiving
chemotherapy, — are at risk for infection with C. difficile, which causes "horse
75. Answer: (A) Glucocorticoids and androgens barn" smelling diarrhea. Successful treatment begins with an accurate diagnosis,
Rationale: The adrenal glands have two divisions, the cortex and medulla. The which includes a stool test. The ELISA test is diagnostic for human
cortex produces three types of hormones: glucocorticoids, mineralocorticoids, immunodeficiency virus (HIV) and isn't indicated in this case. An electrolyte panel
and androgens. The medulla produces catecholamines — epinephrine and and hemogram may be useful in the overall evaluation of a client but aren't
norepinephrine. diagnostic for specific causes of diarrhea. A flat plate of the abdomen may
provide useful information about bowel function but isn't indicated in the case of
76. Answer: (A) Hypocalcemia "horse barn" smelling diarrhea.
Rationale: Hypocalcemia may follow thyroid surgery if the parathyroid glands
were removed accidentally. Signs and symptoms of hypocalcemia may be 87. Answer: (D) Western blot test with ELISA.
delayed for up to 7 days after surgery. Thyroid surgery doesn't directly cause Rationale: HIV infection is detected by analyzing blood for antibodies to HIV,
serum sodium, potassium, or magnesium abnormalities. Hyponatremia may which form approximately 2 to 12 weeks after exposure to HIV and denote
occur if the client inadvertently received too much fluid; however, this can happen infection. The Western blot test — electrophoresis of antibody proteins — is more
to any surgical client receiving I.V. fluid therapy, not just one recovering from than 98% accurate in detecting HIV antibodies when used in conjunction with the
thyroid surgery. Hyperkalemia and hypermagnesemia usually are associated with ELISA. It isn't specific when used alone. Erosette immunofluorescence is used to
reduced renal excretion of potassium and magnesium, not thyroid surgery. detect viruses in general; it doesn't confirm HIV infection. Quantification of T-
lymphocytes is a useful monitoring test but isn't diagnostic for HIV. The ELISA
77. Answer: (D) Carcinoembryonic antigen level test detects HIV antibody particles but may yield inaccurate results; a positive
Rationale: In clients who smoke, the level of carcinoembryonic antigen is ELISA result must be confirmed by the Western blot test.
elevated. Therefore, it can't be used as a general indicator of cancer. However, it
is helpful in monitoring cancer treatment because the level usually falls to normal 88. Answer: (C) Abnormally low hematocrit (HCT) and hemoglobin (Hb) levels
within 1 month if treatment is successful. An elevated acid phosphatase level Rationale: Low preoperative HCT and Hb levels indicate the client may require a
may indicate prostate cancer. An elevated alkaline phosphatase level may reflect blood transfusion before surgery. If the HCT and Hb levels decrease during
bone metastasis. An elevated serum calcitonin level usually signals thyroid surgery because of blood loss, the potential need for a transfusion increases.
cancer. Possible renal failure is indicated by elevated BUN or creatinine levels. Urine
constituents aren't found in the blood. Coagulation is determined by the presence
78. Answer: (B) Dyspnea, tachycardia, and pallor of appropriate clotting factors, not electrolytes.
Rationale: Signs of iron-deficiency anemia include dyspnea, tachycardia, and
pallor as well as fatigue, listlessness, irritability, and headache. Night sweats, 89. Answer: (A) Platelet count, prothrombin time, and partial thromboplastin time
weight loss, and diarrhea may signal acquired immunodeficiency syndrome Rationale: The diagnosis of DIC is based on the results of laboratory studies of
(AIDS). Nausea, vomiting, and anorexia may be signs of hepatitis B. Itching, prothrombin time, platelet count, thrombin time, partial thromboplastin time, and
rash, and jaundice may result from an allergic or hemolytic reaction. fibrinogen level as well as client history and other assessment factors. Blood
glucose levels, WBC count, calcium levels, and potassium levels aren't used to
79. Answer: (D) "I'll need to have a C-section if I become pregnant and have a confirm a diagnosis of DIC.
baby."
Rationale: The human immunodeficiency virus (HIV) is transmitted from mother 90. Answer: (D) Strawberries
to child via the transplacental route, but a Cesarean section delivery isn't Rationale: Common food allergens include berries, peanuts, Brazil nuts,
necessary when the mother is HIV-positive. The use of birth control will prevent cashews, shellfish, and eggs. Bread, carrots, and oranges rarely cause allergic
the conception of a child who might have HIV. It's true that a mother who's HIV reactions.
positive can give birth to a baby who's HIV negative.
91. Answer: (B) A client with cast on the right leg who states, “I have a funny
80. Answer: (C) "Avoid sharing such articles as toothbrushes and razors." feeling in my right leg.”
Rationale: The human immunodeficiency virus (HIV), which causes AIDS, is Rationale: It may indicate neurovascular compromise, requires immediate
most concentrated in the blood. For this reason, the client shouldn't share assessment.
personal articles that may be blood-contaminated, such as toothbrushes and
razors, with other family members. HIV isn't transmitted by bathing or by eating 92. Answer: (D) A 62-year-old who had an abdominal-perineal resection three
from plates, utensils, or serving dishes used by a person with AIDS. days ago; client complaints of chills.
Rationale: The client is at risk for peritonitis; should be assessed for further
81. Answer: (B) Pallor, tachycardia, and a sore tongue symptoms and infection.
Rationale: Pallor, tachycardia, and a sore tongue are all characteristic findings in
pernicious anemia. Other clinical manifestations include anorexia; weight loss; a 93. Answer: (C) The client spontaneously flexes his wrist when the blood
smooth, beefy red tongue; a wide pulse pressure; palpitations; angina; pressure is obtained.
Rationale: Carpal spasms indicate hypocalcemia. a. 18 gtt/min b. 28 gtt/min c. 32 gtt/min d. 36 gtt/min

94. Answer: (D) Use comfort measures and pillows to position the client. 10.Terence suffered form burn injury. Using the rule of nines, which has the
Rationale: Using comfort measures and pillows to position the client is a non- largest percent of burns?
pharmacological methods of pain relief.
a. Face and neck b. Right upper arm and penis
95. Answer: (B) Warm the dialysate solution. c. Right thigh and penis d. Upper trunk
Rationale: Cold dialysate increases discomfort. The solution should be warmed
to body temperature in warmer or heating pad; don’t use microwave oven. 11. Herbert, a 45 year old construction engineer is brought to the hospital
unconscious after falling from a 2-story building. When assessing the client, the
96. Answer: (C) The client holds the cane with his left hand, moves the cane nurse would be most concerned if the assessment revealed:
forward followed by the right leg, and then moves the left leg.
Rationale: The cane acts as a support and aids in weight bearing for the weaker a. Reactive pupils b. A depressed fontanel
right leg. c. Bleeding from ears d. An elevated temperature

97. Answer: (A) Ask the woman’s family to provide personal items such as 12. Nurse Sherry is teaching male client regarding his permanent artificial
photos or mementos. pacemaker. Which information given by the nurse shows her knowledge deficit
Rationale: Photos and mementos provide visual stimulation to reduce sensory about the artificial cardiac pacemaker?
deprivation.
a. take the pulse rate once a day, in the morning upon awakening
98. Answer: (B) The client lifts the walker, moves it forward 10 inches, and then b. May be allowed to use electrical appliances
takes several small steps forward. c. Have regular follow up care
Rationale: A walker needs to be picked up, placed down on all legs. d. May engage in contact sports

99. Answer: (C) Isolation from their families and familiar surroundings. 13.The nurse is ware that the most relevant knowledge about oxygen
Rationale: Gradual loss of sight, hearing, and taste interferes with normal administration to a male client with COPD is
functioning.
a. Oxygen at 1-2L/min is given to maintain the hypoxic stimulus for breathing.
100. Answer: (A) Encourage the client to perform pursed lip breathing. b. Hypoxia stimulates the central chemoreceptors in the medulla that makes the
Rationale: Purse lip breathing prevents the collapse of lung unit and helps client client breath.
control rate and depth of breathing. c. Oxygen is administered best using a non-rebreathing mask
NP4- MS/PSYCH d. Blood gases are monitored using a pulse oximeter.
1. Randy has undergone kidney transplant, what assessment would prompt
Nurse Katrina to suspect organ rejection? 14.Tonny has undergoes a left thoracotomy and a partial pneumonectomy. Chest
tubes are inserted, and one-bottle water-seal drainage is instituted in the
a. Sudden weight loss b. Polyuria c. Hypertension d. Shock operating room. In the postanesthesia care unit Tonny is placed in Fowler's
position on either his right side or on his back. The nurse is aware that this
2. The immediate objective of nursing care for an overweight, mildly hypertensive position:
male client with ureteral colic and hematuria is to decrease:
a. Reduce incisional pain. b. Facilitate ventilation of the left lung.
a. Pain b. Weight c. Hematuria d. Hypertension c. Equalize pressure in the pleural space. d. Increase venous return

3. Matilda, with hyperthyroidism is to receive Lugol’s iodine solution before a 15.Kristine is scheduled for a bronchoscopy. When teaching Kristine what to
subtotal thyroidectomy is performed. The nurse is aware that this medication is expect afterward, the nurse's highest priority of information would be:
given to:
a. Food and fluids will be withheld for at least 2 hours.
a. Decrease the total basal metabolic rate. b. Warm saline gargles will be done q 2h.
b. Maintain the function of the parathyroid glands. c. Coughing and deep-breathing exercises will be done q2h.
c. Block the formation of thyroxine by the thyroid gland. d. Only ice chips and cold liquids will be allowed initially.
d. Decrease the size and vascularity of the thyroid gland.
16.Nurse Tristan is caring for a male client in acute renal failure. The nurse
4. Ricardo, was diagnosed with type I diabetes. The nurse is aware that acute should expect hypertonic glucose, insulin infusions, and sodium bicarbonate to
hypoglycemia also can develop in the client who is diagnosed with: be used to treat:

a. Liver disease b. Hypertension c. Type 2 diabetes d. Hyperthyroidism a. hypernatremia. b. hypokalemia. c. hyperkalemia. d. hypercalcemia.

5. Tracy is receiving combination chemotherapy for treatment of metastatic 17.Ms. X has just been diagnosed with condylomata acuminata (genital warts).
carcinoma. Nurse Ruby should monitor the client for the systemic side effect of: What information is appropriate to tell this client?

a. Ascites b. Nystagmus c. Leukopenia d. Polycythemia a. This condition puts her at a higher risk for cervical cancer; therefore, she
should have a Papanicolaou (Pap) smear annually.
6. Norma, with recent colostomy expresses concern about the inability to control b. The most common treatment is metronidazole (Flagyl), which should eradicate
the passage of gas. Nurse Oliver should suggest that the client plan to: the problem within 7 to 10 days.
c. The potential for transmission to her sexual partner will be eliminated if
a. Eliminate foods high in cellulose. condoms are used every time they have sexual intercourse.
b. Decrease fluid intake at meal times. d. The human papillomavirus (HPV), which causes condylomata acuminata, can't
c. Avoid foods that in the past caused flatus. be transmitted during oral sex.
d. Adhere to a bland diet prior to social events.
18.Maritess was recently diagnosed with a genitourinary problem and is being
7. Nurse Ron begins to teach a male client how to perform colostomy irrigations. examined in the emergency department. When palpating the her kidneys, the
The nurse would evaluate that the instructions were understood when the client nurse should keep which anatomical fact in mind?
states, “I should:
a. The left kidney usually is slightly higher than the right one.
a. Lie on my left side while instilling the irrigating solution.” b. The kidneys are situated just above the adrenal glands.
b. Keep the irrigating container less than 18 inches above the stoma.” c. The average kidney is approximately 5 cm (2") long and 2 to 3 cm (¾" to 1-
c. Instill a minimum of 1200 ml of irrigating solution to stimulate 1/8") wide.
evacuation of the bowel.” d. The kidneys lie between the 10th and 12th thoracic vertebrae.
d. Insert the irrigating catheter deeper into the stoma if cramping
occurs during the procedure.” 19.Jestoni with chronic renal failure (CRF) is admitted to the urology unit. The
nurse is aware that the diagnostic test are consistent with CRF if the result is:
8. Patrick is in the oliguric phase of acute tubular necrosis and is experiencing
fluid and electrolyte imbalances. The client is somewhat confused and complains a. Increased pH with decreased hydrogen ions.
of nausea and muscle weakness. As part of the prescribed therapy to correct this b. Increased serum levels of potassium, magnesium, and calcium.
electrolyte imbalance, the nurse would expect to: c. Blood urea nitrogen (BUN) 100 mg/dl and serum creatinine 6.5 mg/ dl.
d. Uric acid analysis 3.5 mg/dl and phenolsulfonphthalein (PSP) excretion 75%.
a. Administer Kayexalate
b. Restrict foods high in protein 20. Katrina has an abnormal result on a Papanicolaou test. After admitting that
c. Increase oral intake of cheese and milk. she read her chart while the nurse was out of the room, Katrina asks what
d. Administer large amounts of normal saline via I.V. dysplasia means. Which definition should the nurse provide?

9. Mario has burn injury. After Forty48 hours, the physician orders for Mario 2 a. Presence of completely undifferentiated tumor cells that don't resemble cells of
liters of IV fluid to be administered q12 h. The drop factor of the tubing is 10 the tissues of their origin.
gtt/ml. The nurse should set the flow to provide:
b. Increase in the number of normal cells in a normal arrangement in a tissue or 32.The nurse is aware that one of the following classes of medications
an organ. maximizes cardiac performance in clients with heart failure by increasing
c. Replacement of one type of fully differentiated cell by another in tissues where ventricular contractility?
the second type normally isn't found.
d. Alteration in the size, shape, and organization of differentiated cells. a. Beta-adrenergic blockers b. Calcium channel blocker
c. Diuretics d. Inotropic agents
21. During a routine checkup, Nurse Mariane assesses a male client with
acquired immunodeficiency syndrome (AIDS) for signs and symptoms of cancer. 33.A male client has a reduced serum high-density lipoprotein (HDL) level and an
What is the most common AIDS-related cancer? elevated low-density lipoprotein (LDL) level. Which of the following dietary
modifications is not appropriate for this client?
a. Squamous cell carcinoma b. Multiple myeloma
c. Leukemia d. Kaposi's sarcoma a. Fiber intake of 25 to 30 g daily
b. Less than 30% of calories form fat
22.Ricardo is scheduled for a prostatectomy, and the anesthesiologist plans to c. Cholesterol intake of less than 300 mg daily
use a spinal (subarachnoid) block during surgery. In the operating room, the d. Less than 10% of calories from saturated fat
nurse positions the client according to the anesthesiologist's instructions. Why
does the client require special positioning for this type of anesthesia? 34. A 37-year-old male client was admitted to the coronary care unit (CCU) 2
days ago with an acute myocardial infarction. Which of the following actions
a. To prevent confusion would breach the client confidentiality?
b. To prevent seizures
c. To prevent cerebrospinal fluid (CSF) leakage a. The CCU nurse gives a verbal report to the nurse on the telemetry unit before
d. To prevent cardiac arrhythmias transferring the client to that unit
b. The CCU nurse notifies the on-call physician about a change in the client’s
23.A male client had a nephrectomy 2 days ago and is now complaining of condition
abdominal pressure and nausea. The first nursing action should be to: c. The emergency department nurse calls up the latest electrocardiogram results
to check the client’s progress.
a. Auscultate bowel sounds. b. Palpate the abdomen. d. At the client’s request, the CCU nurse updates the client’s wife on his condition
c. Change the client's position. d. Insert a rectal tube.
35. A male client arriving in the emergency department is receiving
24.Wilfredo with a recent history of rectal bleeding is being prepared for a cardiopulmonary resuscitation from paramedics who are giving ventilations
colonoscopy. How should the nurse Patricia position the client for this test through an endotracheal (ET) tube that they placed in the client’s home. During a
initially? pause in compressions, the cardiac monitor shows narrow QRS complexes and
a heart rate of beats/minute with a palpable pulse. Which of the following actions
a. Lying on the right side with legs straight should the nurse take first?
b. Lying on the left side with knees bent
c. Prone with the torso elevated a. Start an L.V. line and administer amiodarone (Cardarone), 300 mg L.V. over
d. Bent over with hands touching the floor 10 minutes.
b. Check endotracheal tube placement.
25.A male client with inflammatory bowel disease undergoes an ileostomy. On c. Obtain an arterial blood gas (ABG) sample.
the first day after surgery, Nurse Oliver notes that the client's stoma appears d. Administer atropine, 1 mg L.V.
dusky. How should the nurse interpret this finding?
36. After cardiac surgery, a client’s blood pressure measures 126/80 mm Hg.
a. Blood supply to the stoma has been interrupted. Nurse Katrina determines that mean arterial pressure (MAP) is which of the
b. This is a normal finding 1 day after surgery. following?
c. The ostomy bag should be adjusted.
d. An intestinal obstruction has occurred. a. 46 mm Hg b. 80 mm Hg c. 95 mm Hg d. 90 mm Hg

26.Anthony suffers burns on the legs, which nursing intervention helps prevent 37. A female client arrives at the emergency department with chest and stomach
contractures? pain and a report of black tarry stool for several months. Which of the following
order should the nurse Oliver anticipate?
a. Applying knee splints
b. Elevating the foot of the bed a. Cardiac monitor, oxygen, creatine kinase and lactate dehydrogenase levels
c. Hyperextending the client's palms b. Prothrombin time, partial thromboplastin time, fibrinogen and fibrin split
d. Performing shoulder range-of-motion exercises product values.
c. Electrocardiogram, complete blood count, testing for occult blood,
27.Nurse Ron is assessing a client admitted with second- and third-degree burns comprehensive serum metabolic panel.
on the face, arms, and chest. Which finding indicates a potential problem? d. Electroencephalogram, alkaline phosphatase and aspartate aminotransferase
levels, basic serum metabolic panel
a. Partial pressure of arterial oxygen (PaO2) value of 80 mm Hg.
b. Urine output of 20 ml/hour. 38. Macario had coronary artery bypass graft (CABG) surgery 3 days ago. Which
c. White pulmonary secretions. of the following conditions is suspected by the nurse when a decrease in platelet
d. Rectal temperature of 100.6° F (38° C). count from 230,000 ul to 5,000 ul is noted?

28. Mr. Mendoza who has suffered a cerebrovascular accident (CVA) is too weak a. Pancytopenia
to move on his own. To help the client avoid pressure ulcers, Nurse Celia should: b. Idiopathic thrombocytopemic purpura (ITP)
c. Disseminated intravascular coagulation (DIC)
a. Turn him frequently. d. Heparin-associated thrombosis and thrombocytopenia (HATT)
b. Perform passive range-of-motion (ROM) exercises.
c. Reduce the client's fluid intake. 39. Which of the following drugs would be ordered by the physician to improve
d. Encourage the client to use a footboard. the platelet count in a male client with idiopathic thrombocytopenic purpura
(ITP)?
29.Nurse Maria plans to administer dexamethasone cream to a female client who
has dermatitis over the anterior chest. How should the nurse apply this topical a. Acetylsalicylic acid (ASA) b. Corticosteroids
agent? c. Methotrezate d. Vitamin K

a. With a circular motion, to enhance absorption. 40. A female client is scheduled to receive a heart valve replacement with a
b. With an upward motion, to increase blood supply to the affected area porcine valve. Which of the following types of transplant is this?
c. In long, even, outward, and downward strokes in the direction of hair growth
d. In long, even, outward, and upward strokes in the direction opposite hair a. Allogeneic b. Autologous c. Syngeneic d. Xenogeneic
growth
41. Marco falls off his bicycle and injuries his ankle. Which of the following
30.Nurse Kate is aware that one of the following classes of medication protect actions shows the initial response to the injury in the extrinsic pathway?
the ischemic myocardium by blocking catecholamines and
sympathetic nerve stimulation is: a. Release of Calcium
b. Release of tissue thromboplastin
a. Beta -adrenergic blockers b. Calcium channel blocker c. Conversion of factors XII to factor XIIa
c. Narcotics d. Nitrates d. Conversion of factor VIII to factor VIIIa

31.A male client has jugular distention. On what position should the nurse place 42. Instructions for a client with systemic lupus erythematosus (SLE) would
the head of the bed to obtain the most accurate reading of jugular vein include information about which of the following blood dyscrasias?
distention?
a. Dressler’s syndrome b. Polycythemia
a. High Fowler’s b. Raised 10 degrees c. Raised 30 degrees d. Supine position c. Essential thrombocytopenia d. Von Willebrand’s disease
43. The nurse is aware that the following symptoms is most commonly an early a. Adult respiratory distress syndrome (ARDS)
indication of stage 1 Hodgkin’s disease? b. Asthma
c. Chronic obstructive bronchitis
a. Pericarditis b. Night sweat c. Splenomegaly d. Persistent hypothermia d. Emphysema

44. Francis with leukemia has neutropenia. Which of the following functions must 56. Jose is in danger of respiratory arrest following the administration of a
frequently assessed? narcotic analgesic. An arterial blood gas value is obtained. Nurse Oliver would
expect the paco2 to be which of the following values?
a. Blood pressure b. Bowel sounds c. Heart sounds d. Breath sounds
a. 15 mm Hg b. 30 mm Hg c. 40 mm Hg d. 80 mm Hg
45. The nurse knows that neurologic complications of multiple myeloma (MM)
usually involve which of the following body system? 57. Timothy’s arterial blood gas (ABG) results are as follows; pH 7.16; Paco2 80
mm Hg; Pao2 46 mm Hg; HCO3- 24mEq/L; Sao2 81%. This ABG result
a. Brain b. Muscle spasm c. Renal dysfunction d. Myocardial irritability represents which of the following conditions?

46. Nurse Patricia is aware that the average length of time from human a. Metabolic acidosis b. Metabolic alkalosis
immunodeficiency virus (HIV) infection to the development of acquired c. Respiratory acidosis d. Respirator y alkalosis
immunodeficiency syndrome (AIDS)?
58. Norma has started a new drug for hypertension. Thirty minutes after she
a. Less than 5 years b. 5 to 7 years c. 10 years d. More than 10 years takes the drug, she develops chest tightness and becomes short of breath and
tachypneic. She has a decreased level of consciousness. These signs indicate
47. An 18-year-old male client admitted with heat stroke begins to show signs of which of the following conditions?
disseminated intravascular coagulation (DIC). Which of the following laboratory
findings is most consistent with DIC? a. Asthma attack b. Pulmonary embolism
c. Respiratory failure d. Rheumatoid arthritis
a. Low platelet count
b. Elevated fibrinogen levels Situation: Mr. Gonzales was admitted to the hospital with ascites and jaundice.
c. Low levels of fibrin degradation products To rule out cirrhosis of the liver:
d. Reduced prothrombin time
59. Which laboratory test indicates liver cirrhosis?
48. Mario comes to the clinic complaining of fever, drenching night sweats, and
unexplained weight loss over the past 3 months. Physical examination reveals a a. Decreased red blood cell count b. Decreased serum acid phosphate level
single enlarged supraclavicular lymph node. Which of the following is the most c. Elevated white blood cell count d. Elevated serum aminotransferase
probable diagnosis?
60.The biopsy of Mr. Gonzales confirms the diagnosis of cirrhosis. Mr. Gonzales
a. Influenza b. Sickle cell anemia c. Leukemia d. Hodgkin’s disease is at increased risk for excessive bleeding primarily because of:

49. A male client with a gunshot wound requires an emergency blood a. Impaired clotting mechanism b. Varix formation
transfusion. His blood type is AB negative. Which blood type would be the safest c. Inadequate nutrition d. Trauma of invasive procedure
for him to receive?
61. Mr. Gonzales develops hepatic encephalopathy. Which clinical manifestation
a. AB Rh-positive b. A Rh-positive c. A Rh-negative d. O Rh-positive is most common with this condition?

Situation: Stacy is diagnosed with acute lymphoid leukemia (ALL) and beginning a. Increased urine output b. Altered level of consciousness
chemotherapy. c. Decreased tendon reflex d. Hypotension

50. Stacy is discharged from the hospital following her chemotherapy treatments. 62. When Mr. Gonzales regained consciousness, the physician orders 50 ml of
Which statement of Stacy’s mother indicated that she understands when she will Lactose p.o. every 2 hours. Mr. Gozales develops diarrhea. The nurse best
contact the physician? action would be:

a. “I should contact the physician if Stacy has difficulty in sleeping”. a. “I’ll see if your physician is in the hospital”.
b. “I will call my doctor if Stacy has persistent vomiting and diarrhea”. b. “Maybe your reacting to the drug; I will withhold the next dose”.
c. “My physician should be called if Stacy is irritable and unhappy”. c. “I’ll lower the dosage as ordered so the drug causes only 2 to 4 stools a day”.
d. “Should Stacy have continued hair loss, I need to call the doctor”. d. “Frequently, bowel movements are needed to reduce sodium level”.

51. Stacy’s mother states to the nurse that it is hard to see Stacy with no hair. 63. Which of the following groups of symptoms indicates a ruptured abdominal
The best response for the nurse is: aortic aneurysm?

a. “Stacy looks very nice wearing a hat”. a. Lower back pain, increased blood pressure, decreased re blood cell (RBC)
b. “You should not worry about her hair, just be glad that she is alive”. count, increased white blood (WBC) count.
c. “Yes it is upsetting. But try to cover up your feelings when you are with her or b. Severe lower back pain, decreased blood pressure, decreased RBC count,
else she may be upset”. increased WBC count.
d. “This is only temporary; Stacy will re-grow new hair in 3-6 months, but may be c. Severe lower back pain, decreased blood pressure, decreased RBC count,
different in texture”. decreased RBC count, decreased WBC count.
d. Intermitted lower back pain, decreased blood pressure, decreased RBC count,
52. Stacy has beginning stomatitis. To promote oral hygiene and comfort, the increased WBC count.
nurse in-charge should:
64. After undergoing a cardiac catheterization, Tracy has a large puddle of blood
a. Provide frequent mouthwash with normal saline. under his buttocks. Which of the following steps should the nurse take first?
b. Apply viscous Lidocaine to oral ulcers as needed.
c. Use lemon glycerine swabs every 2 hours. a. Call for help. b. Obtain vital signs
d. Rinse mouth with Hydrogen Peroxide. c. Ask the client to “lift up” d. Apply gloves and assess the groin site

53. During the administration of chemotherapy agents, Nurse Oliver observed 65. Which of the following treatment is a suitable surgical intervention for a client
that the IV site is red and swollen, when the IV is touched Stacy shouts in pain. with unstable angina?
The first nursing action to take is:
a. Cardiac catheterization b. Echocardiogram
a. Notify the physician c. Nitroglycerin d. Percutaneous transluminal coronary angioplasty (PTCA)
b. Flush the IV line with saline solution
c. Immediately discontinue the infusion 66. The nurse is aware that the following terms used to describe reduced cardiac
d. Apply an ice pack to the site, followed by warm compress. output and perfusion impairment due to ineffective pumping of the heart is:

54. The term “blue bloater” refers to a male client which of the following a. Anaphylactic shock b. Cardiogenic shock
conditions? c. Distributive shock d. Myocardial infarction (MI)

a. Adult respiratory distress syndrome (ARDS) 67. A client with hypertension ask the nurse which factors can cause blood
b. Asthma pressure to drop to normal levels?
c. Chronic obstructive bronchitis
d. Emphysema a. Kidneys’ excretion to sodium only.
b. Kidneys’ retention of sodium and water
55. The term “pink puffer” refers to the female client with which of the following c. Kidneys’ excretion of sodium and water
conditions? d. Kidneys’ retention of sodium and excretion of water
68. Nurse Rose is aware that the statement that best explains why furosemide b. "If I experience trembling, weakness, and headache, I should drink a glass of
(Lasix) is administered to treat hypertension is: soda that contains sugar."
c. "I will have to monitor my blood glucose level closely and notify the physician if
a. It dilates peripheral blood vessels. it's constantly elevated."
b. It decreases sympathetic cardioacceleration. d. "If I begin to feel especially hungry and thirsty, I'll eat a snack high in
c. It inhibits the angiotensin-coverting enzymes carbohydrates."
d. It inhibits reabsorption of sodium and water in the loop of Henle.
79. A 66-year-old client has been complaining of sleeping more, increased
69. Nurse Nikki knows that laboratory results supports the diagnosis of systemic urination, anorexia, weakness, irritability, depression, and bone pain that
lupus erythematosus (SLE) is: interferes with her going outdoors. Based on these assessment findings, the
nurse would suspect which of the following disorders?
a. Elavated serum complement level
b. Thrombocytosis, elevated sedimentation rate a. Diabetes mellitus b. Diabetes insipidus
c. Pancytopenia, elevated antinuclear antibody (ANA) titer c. Hypoparathyroidism d. Hyperparathyroidism
d. Leukocysis, elevated blood urea nitrogen (BUN) and creatinine levels
80. Nurse Lourdes is teaching a client recovering from addisonian crisis about
70. Arnold, a 19-year-old client with a mild concussion is discharged from the the need to take fludrocortisone acetate and hydrocortisone at home. Which
emergency department. Before discharge, he complains of a headache. When statement by the client indicates an understanding of the instructions?
offered acetaminophen, his mother tells the nurse the headache is severe and
she would like her son to have something stronger. Which of the following a. "I'll take my hydrocortisone in the late afternoon, before dinner."
responses by the nurse is appropriate? b. "I'll take all of my hydrocortisone in the morning, right after I wake up."
c. "I'll take two-thirds of the dose when I wake up and one-third in the late
a. “Your son had a mild concussion, acetaminophen is strong enough.” afternoon."
b. “Aspirin is avoided because of the danger of Reye’s syndrome in children or d. "I'll take the entire dose at bedtime."
young adults.”
c. “Narcotics are avoided after a head injury because they may hide a worsening 81. Which of the following laboratory test results would suggest to the nurse Len
condition.” that a client has a corticotropin-secreting pituitary adenoma?
d. Stronger medications may lead to vomiting, which increases the intracarnial
pressure (ICP).” a. High corticotropin and low cortisol levels
b. Low corticotropin and high cortisol levels
71. When evaluating an arterial blood gas from a male client with a subdural c. High corticotropin and high cortisol levels
hematoma, the nurse notes the Paco2 is 30 mm Hg. Which of the following d. Low corticotropin and low cortisol levels
responses best describes the result?
82. A male client is scheduled for a transsphenoidal hypophysectomy to remove
a. Appropriate; lowering carbon dioxide (CO2) reduces intracranial pressure a pituitary tumor. Preoperatively, the nurse should assess for potential
(ICP) complications by doing which of the following?
b. Emergent; the client is poorly oxygenated
c. Normal a. Testing for ketones in the urine
d. Significant; the client has alveolar hypoventilation b. Testing urine specific gravity
c. Checking temperature every 4 hours
72. When prioritizing care, which of the following clients should the nurse Olivia d. Performing capillary glucose testing every 4 hours
assess first?
83. Capillary glucose monitoring is being performed every 4 hours for a client
a. A 17-year-old clients 24-hours postappendectomy diagnosed with diabetic ketoacidosis. Insulin is administered using a scale of
b. A 33-year-old client with a recent diagnosis of Guillain-Barre syndrome regular insulin according to glucose results. At 2 p.m., the client has a capillary
c. A 50-year-old client 3 days postmyocardial infarction glucose level of 250 mg/dl for which he receives 8 U of regular insulin. Nurse
d. A 50-year-old client with diverticulitis Mariner should expect the dose's:

73. JP has been diagnosed with gout and wants to know why colchicine is used a. onset to be at 2 p.m. and its peak to be at 3 p.m.
in the treatment of gout. Which of the following actions of colchicines explains b. onset to be at 2:15 p.m. and its peak to be at 3 p.m.
why it’s effective for gout? c. onset to be at 2:30 p.m. and its peak to be at 4 p.m.
d. onset to be at 4 p.m. and its peak to be at 6 p.m.
a. Replaces estrogen b. Decreases infection
c. Decreases inflammation d. Decreases bone demineralization 84. The physician orders laboratory tests to confirm hyperthyroidism in a female
client with classic signs and symptoms of this disorder. Which test result would
74. Norma asks for information about osteoarthritis. Which of the following confirm the diagnosis?
statements about osteoarthritis is correct?
a. No increase in the thyroid-stimulating hormone (TSH) level after 30 minutes
a. Osteoarthritis is rarely debilitating during the TSH stimulation test
b. Osteoarthritis is a rare form of arthritis b. A decreased TSH level
c. Osteoarthritis is the most common form of arthritis c. An increase in the TSH level after 30 minutes during the TSH stimulation test
d. Osteoarthritis afflicts people over 60 d. Below-normal levels of serum triiodothyronine (T3) and serum thyroxine (T4)
as detected by radioimmunoassay
75. Ruby is receiving thyroid replacement therapy develops the flu and forgets to
take her thyroid replacement medicine. The nurse understands that skipping this 85. Rico with diabetes mellitus must learn how to self-administer insulin. The
medication will put the client at risk for developing which of the following physician has prescribed 10 U of U-100 regular insulin and 35 U of U-100
lifethreatening complications? isophane insulin suspension (NPH) to be taken before breakfast. When teaching
the client how to select and rotate insulin injection sites, the nurse should provide
a. Exophthalmos b. Thyroid storm c. Myxedema coma d. Tibial myxedema which instruction?

76. Nurse Sugar is assessing a client with Cushing's syndrome. Which a. "Inject insulin into healthy tissue with large blood vessels and nerves."
observation should the nurse report to the physician immediately? b. "Rotate injection sites within the same anatomic region, not among different
regions."
a. Pitting edema of the legs b. An irregular apical pulse c. "Administer insulin into areas of scar tissue or hypotrophy whenever possible."
c. Dry mucous membranes d. Frequent urination d. "Administer insulin into sites above muscles that you plan to exercise heavily
later that day."
77. Cyrill with severe head trauma sustained in a car accident is admitted to the
intensive care unit. Thirty-six hours later, the client's urine output suddenly rises 86. Nurse Sarah expects to note an elevated serum glucose level in a client with
above 200 ml/hour, leading the nurse to suspect diabetes insipidus. Which hyperosmolar hyperglycemic nonketotic syndrome (HHNS). Which other
laboratory findings support the nurse's suspicion of diabetes insipidus? laboratory finding should the nurse anticipate?

a. Above-normal urine and serum osmolality levels a. Elevated serum acetone level b. Serum ketone bodies
b. Below-normal urine and serum osmolality levels c. Serum alkalosis d. Below-normal serum potassium level
c. Above-normal urine osmolality level, below-normal serum osmolality level
d. Below-normal urine osmolality level, above-normal serum osmolality level 87. For a client with Graves' disease, which nursing intervention promotes
comfort?
78. Jomari is diagnosed with hyperosmolar hyperglycemic nonketotic syndrome
(HHNS) is stabilized and prepared for discharge. When preparing the client for a. Restricting intake of oral fluids
discharge and home management, which of the following statements indicates b. Placing extra blankets on the client's bed
that the client understands her condition and how to control it? c. Limiting intake of high-carbohydrate foods
d. Maintaining room temperature in the low-normal range
a. "I can avoid getting sick by not becoming dehydrated and by paying attention
to my need to urinate, drink, or eat more than usual."
88. Patrick is treated in the emergency department for a Colles' fracture d. “I should put on the stockings before getting out of bed in the morning.”
sustained during a fall. What is a Colles' fracture? RATIO 4
1. Answer: (C) Hypertension
a. Fracture of the distal radius b. Fracture of the olecranon Rationale: Hypertension, along with fever, and tenderness over the grafted
c. Fracture of the humerus d. Fracture of the carpal scaphoid kidney, reflects acute rejection.

89. Cleo is diagnosed with osteoporosis. Which electrolytes are involved in the 2. Answer: (A) Pain
development of this disorder? Rationale: Sharp, severe pain (renal colic) radiating toward the genitalia and
thigh is caused by uretheral distention and smooth muscle spasm; relief form
a. Calcium and sodium b. Calcium and phosphorous pain is the priority.
c. Phosphorous and potassium d. Potassium and sodium
3. Answer: (D) Decrease the size and vascularity of the thyroid gland.
90. Johnny a firefighter was involved in extinguishing a house fire and is being Rationale: Lugol’s solution provides iodine, which aids in decreasing the
treated to smoke inhalation. He develops severe hypoxia 48 hours after the vascularity of the thyroid gland, which limits the risk of hemorrhage when surgery
incident, requiring intubation and mechanical ventilation. He most likely has is performed.
developed which of the following conditions?
4. Answer: (A) Liver Disease
a. Adult respiratory distress syndrome (ARDS) Rationale: The client with liver disease has a decreased ability to metabolize
b. Atelectasis carbohydrates because of a decreased ability to form glycogen (glycogenesis)
c. Bronchitis and to form glucose from glycogen.
d. Pneumonia
5. Answer: (C) Leukopenia
91. A 67-year-old client develops acute shortness of breath and progressive Rationale: Leukopenia, a reduction in WBCs, is a systemic effect of
hypoxia requiring right femur. The hypoxia was probably caused by which of the chemotherapy as a result of myelosuppression.
following conditions?
6. Answer: (C) Avoid foods that in the past caused flatus.
a. Asthma attack b. Atelectasis c. Bronchitis d. Fat embolism Rationale: Foods that bothered a person preoperatively will continue to do so
after a colostomy.
92. A client with shortness of breath has decreased to absent breath sounds on
the right side, from the apex to the base. Which of the following conditions would 7. Answer: (B) Keep the irrigating container less than 18 inches above the
best explain this? stoma.”
Rationale: This height permits the solution to flow slowly with little force so that
a. Acute asthma b. Chronic bronchitis excessive peristalsis is not immediately precipitated.
c. Pneumonia d. Spontaneous pneumothorax
8. Answer: (A) Administer Kayexalate
93. A 62-year-old male client was in a motor vehicle accident as an unrestrained Rationale: Kayexalate,a potassium exchange resin, permits sodium to be
driver. He’s now in the emergency department complaining of difficulty of exchanged for potassium in the intestine, reducing the serum potassium level.
breathing and chest pain. On auscultation of his lung field, no breath sounds are
present in the upper lobe. This client may have which of the following conditions? 9. Answer:(B) 28 gtt/min
Rationale: This is the correct flow rate; multiply the amount to be infused (2000
a. Bronchitis b. Pneumonia c. Pneumothorax d. Tuberculosis (TB) ml) by the drop factor (10) and divide the result by the amount of time in minutes
(12 hours x 60 minutes)
94. If a client requires a pneumonectomy, what fills the area of the thoracic
cavity? 10. Answer: (D) Upper trunk
Rationale: The percentage designated for each burned part of the body using the
a. The space remains filled with air only rule of nines: Head and neck 9%; Right upper extremity 9%; Left upper extremity
b. The surgeon fills the space with a gel 9%; Anterior trunk 18%; Posterior trunk 18%; Right lower extremity 18%; Left
c. Serous fluids fills the space and consolidates the region lower extremity 18%; Perineum 1%.
d. The tissue from the other lung grows over to the other side
11. Answer: (C) Bleeding from ears
95. Hemoptysis may be present in the client with a pulmonary embolism because Rationale: The nurse needs to perform a thorough assessment that could
of which of the following reasons? indicate alterations in cerebral function, increased intracranial pressures,
fractures and bleeding. Bleeding from the ears occurs only with basal skull
a. Alveolar damage in the infracted area fractures that can easily contribute to increased intracranial pressure and brain
b. Involvement of major blood vessels in the occluded area herniation.
c. Loss of lung parenchyma
d. Loss of lung tissue 12. Answer: (D) may engage in contact sports
Rationale: The client should be advised by the nurse to avoid contact sports. This
96. Aldo with a massive pulmonary embolism will have an arterial blood gas will prevent trauma to the area of the pacemaker generator.
analysis performed to determine the extent of hypoxia. The acid-base disorder
that may be present is? 13. Answer: (A) Oxygen at 1-2L/min is given to maintain the hypoxic stimulus for
breathing.
a. Metabolic acidosis b. Metabolic alkalosis Rationale: COPD causes a chronic CO2 retention that renders the medulla
c. Respiratory acidosis d. Respiratory alkalosis insensitive to the CO2 stimulation for breathing. The hypoxic state of the client
then becomes the stimulus for breathing. Giving the client oxygen in low
97. After a motor vehicle accident, Armand an 22-year-old client is admitted with concentrations will maintain the client’s hypoxic drive.
a pneumothorax. The surgeon inserts a chest tube and attaches it to a chest
drainage system. Bubbling soon appears in the water seal chamber. Which of the 14. Answer: (B) Facilitate ventilation of the left lung.
following is the most likely cause of the bubbling? Rationale: Since only a partial pneumonectomy is done, there is a need to
promote expansion of this remaining Left lung by positioning the client on the
a. Air leak b. Adequate suction c. Inadequate suction d. Kinked chest tube opposite unoperated side.

98. Nurse Michelle calculates the IV flow rate for a postoperative client. The 15. Answer: (A) Food and fluids will be withheld for at least 2 hours.
client receives 3,000 ml of Ringer’s lactate solution IV to run over 24 hours. The Rationale: Prior to bronchoscopy, the doctors sprays the back of the throat with
IV infusion set has a drop factor of 10 drops per milliliter. The nurse should anesthetic to minimize the gag reflex and thus facilitate the insertion of the
regulate the client’s IV to deliver how many drops per minute? bronchoscope. Giving the client food and drink after the procedure without
checking on the return of the gag reflex can cause the client to aspirate. The gag
a. 18 b. 21 c. 35 d. 40 reflex usually returns after two hours.

99. Mickey, a 6-year-old child with a congenital heart disorder is admitted with 16. Answer: (C) hyperkalemia.
congestive heart failure. Digoxin (lanoxin) 0.12 mg is ordered for the child. The Rationale: Hyperkalemia is a common complication of acute renal failure. It's life-
bottle of Lanoxin contains .05 mg of Lanoxin in 1 ml of solution. What amount threatening if immediate action isn't taken to reverse it. The administration of
should the nurse administer to the child? glucose and regular insulin, with sodium bicarbonate if necessary, can
temporarily prevent cardiac arrest by moving potassium into the cells and
a. 1.2 ml b. 2.4 ml c. 3.5 ml d. 4.2 ml temporarily reducing serum potassium levels. Hypernatremia, hypokalemia, and
hypercalcemia don't usually occur with acute renal failure and aren't treated with
100. Nurse Alexandra teaches a client about elastic stockings. Which of the glucose, insulin, or sodium bicarbonate.
following statements, if made by the client, indicates to the nurse that the
teaching was successful? 17. Answer: (A) This condition puts her at a higher risk for cervical cancer;
therefore, she should have a Papanicolaou (Pap) smear annually.
a. “I will wear the stockings until the physician tells me to remove them.” Rationale: Women with condylomata acuminata are at risk for cancer of the
b. “I should wear the stockings even when I am sleep.” cervix and vulva. Yearly Pap smears are very important for early detection.
c. “Every four hours I should remove the stockings for a half hour.” Because condylomata acuminata is a virus, there is no permanent cure. Because
condylomata acuminata can occur on the vulva, a condom won't protect sexual 29. Answer: (C) In long, even, outward, and downward strokes in the direction of
partners. HPV can be transmitted to other parts of the body, such as the mouth, hair growth
oropharynx, and larynx. Rationale: When applying a topical agent, the nurse should begin at the midline
and use long, even, outward, and downward strokes in the direction of hair
18. Answer: (A) The left kidney usually is slightly higher than the right one. growth. This application pattern reduces the risk of follicle irritation and skin
Rationale: The left kidney usually is slightly higher than the right one. An adrenal inflammation.
gland lies atop each kidney. The average kidney measures approximately 11 cm
(4-3/8") long, 5 to 5.8 cm (2" to 2¼") wide, and 2.5 cm (1") thick. The kidneys are 30. Answer: (A) Beta -adrenergic blockers
located retroperitoneally, in the posterior aspect of the abdomen, on either side of Rationale: Beta-adrenergic blockers work by blocking beta receptors in the
the vertebral column. They lie between the 12th thoracic and 3rd lumbar myocardium, reducing the response to catecholamines and sympathetic nerve
vertebrae. stimulation. They protect the myocardium, helping to reduce the risk of another
infraction by decreasing myocardial oxygen demand. Calcium channel blockers
19. Answer: (C) Blood urea nitrogen (BUN) 100 mg/dl and serum creatinine 6.5 reduce the workload of the heart by
mg/dl. decreasing the heart rate. Narcotics reduce myocardial oxygen demand, promote
Rationale: The normal BUN level ranges 8 to 23 mg/dl; the normal serum vasodilation, and decrease anxiety. Nitrates reduce myocardial oxygen
creatinine level ranges from 0.7 to 1.5 mg/dl. The test results in option C are consumption bt decreasing left ventricular end diastolic pressure (preload) and
abnormally elevated, reflecting CRF and the kidneys' decreased ability to remove systemic vascular resistance (afterload).
nonprotein nitrogen waste from the blood. CRF causes decreased pH and
increased hydrogen ions — not vice versa. CRF also increases serum levels of 31. Answer: (C) Raised 30 degrees
potassium, magnesium, and phosphorous, and decreases serum levels of Rationale: Jugular venous pressure is measured with a centimeter ruler to obtain
calcium. A uric acid analysis of 3.5 mg/dl falls within the normal range of 2.7 to the vertical distance between the sternal angle and the point of highest pulsation
7.7 mg/dl; PSP excretion of 75% also falls with the normal range of 60% to 75%. with the head of the bed inclined between 15 to 30 degrees. Increased pressure
can’t be seen when the client is supine or when the head of the bed is raised 10
20. Answer: (D) Alteration in the size, shape, and organization of differentiated degrees because the point that marks the pressure level is above the jaw
cells (therefore, not visible). In high Fowler’s position, the veins would be barely
Rationale: Dysplasia refers to an alteration in the size, shape, and organization of discernible above the clavicle.
differentiated cells. The presence of completely undifferentiated tumor cells that
don't resemble cells of the tissues of their origin is called anaplasia. An increase 32. Answer: (D) Inotropic agents
in the number of normal cells in a normal arrangement in a tissue or an organ is Rationale: Inotropic agents are administered to increase the force of the heart’s
called hyperplasia. Replacement of one type of fully differentiated cell by another contractions, thereby increasing ventricular contractility and ultimately increasing
in tissues where the second type normally isn't found is called metaplasia. cardiac output. Beta-adrenergic blockers and calcium channel blockers decrease
the heart rate and ultimately decreased the workload of the heart. Diuretics are
21. Answer: (D) Kaposi's sarcoma administered to decrease the overall vascular volume, also decreasing the
Rationale: Kaposi's sarcoma is the most common cancer associated with AIDS. workload of the heart.
Squamous cell carcinoma, multiple myeloma, and leukemia may occur in anyone
and aren't associated specifically with AIDS. 33. Answer: (B) Less than 30% of calories form fat
Rationale: A client with low serum HDL and high serum LDL levels should get
22. Answer: (C) To prevent cerebrospinal fluid (CSF) leakage less than 30% of daily calories from fat. The other modifications are appropriate
Rationale: The client receiving a subarachnoid block requires special positioning for this client.
to prevent CSF leakage and headache and to ensure proper anesthetic
distribution. Proper positioning doesn't help prevent confusion, seizures, or 34. Answer: (C) The emergency department nurse calls up the latest
cardiac arrhythmias. electrocardiogram results to check the client’s progress
Rationale: The emergency department nurse is no longer directly involved with
23. Answer: (A) Auscultate bowel sounds. the client’s care and thus has no legal right to information about his present
Rationale: If abdominal distention is accompanied by nausea, the nurse must first condition. Anyone directly involved in his care (such as the telemetry nurse and
auscultate bowel sounds. If bowel sounds are absent, the nurse should suspect the on-call physician) has the right to information about his condition. Because
gastric or small intestine dilation and these findings must be reported to the the client requested that the nurse update his wife on his condition, doing so
physician. Palpation should be avoided postoperatively with abdominal doesn’t breach confidentiality.
distention. If peristalsis is absent, changing positions and inserting a rectal tube
won't relieve the client's discomfort. 35. Answer: (B) Check endotracheal tube placement.
Rationale: ET tube placement should be confirmed as soon as the client arrives
24. Answer: (B) Lying on the left side with knees bent in the emergency department. Once the airways is secured, oxygenation and
Rationale: For a colonoscopy, the nurse initially should position the client on the ventilation should be confirmed using an end-tidal carbon dioxide monitor and
left side with knees bent. Placing the client on the right side with legs straight, pulse oximetry. Next, the nurse should make sure L.V. access is established. If
prone with the torso elevated, or bent over with hands touching the floor wouldn't the client experiences symptomatic bradycardia, atropine is administered as
allow proper visualization of the large intestine. ordered 0.5 to 1 mg every 3 to 5 minutes to a total of 3 mg. Then the nurse
should try to find the cause of the client’s arrest by obtaining an ABG sample.
25. Answer: (A) Blood supply to the stoma has been interrupted Amiodarone is indicated for ventricular tachycardia, ventricular fibrillation and
Rationale: An ileostomy stoma forms as the ileum is brought through the atrial flutter – not symptomatic bradycardia.
abdominal wall to the surface skin, creating an artificial opening for waste
elimination. The stoma should appear cherry red, indicating adequate arterial 36. Answer: (C) 95 mm Hg
perfusion. A dusky stoma suggests decreased perfusion, which may result from Rationale: Use the following formula to calculate MAP
interruption of the stoma's blood supply and may lead to tissue damage or MAP = systolic + 2 (diastolic)
necrosis. A dusky stoma isn't a normal finding. Adjusting the ostomy bag wouldn't 3
affect stoma color, which depends on blood supply to the area. An intestinal MAP=126 mm Hg + 2 (80 mm Hg)
obstruction also wouldn't change stoma color. 3
MAP=286 mm HG
26. Answer: (A) Applying knee splints 3
Rationale: Applying knee splints prevents leg contractures by holding the joints in MAP=95 mm Hg
a position of function. Elevating the foot of the bed can't prevent contractures
because this action doesn't hold the joints in a position of function. 37. Answer: (C) Electrocardiogram, complete blood count, testing for occult
Hyperextending a body part for an extended time is inappropriate because it can blood, comprehensive serum metabolic panel.
cause contractures. Performing shoulder range-of-motion exercises can prevent Rationale: An electrocardiogram evaluates the complaints of chest pain,
contractures in the shoulders, but not in the legs. laboratory tests determines anemia, and the stool test for occult blood
determines blood in the stool. Cardiac monitoring, oxygen, and creatine kinase
27. Answer: (B) Urine output of 20 ml/hour. and lactate dehydrogenase levels are appropriate for a cardiac primary problem.
Rationale: A urine output of less than 40 ml/hour in a client with burns indicates a A basic metabolic panel and alkaline phosphatase and aspartate
fluid volume deficit. This client's PaO2 value falls within the normal range (80 to aminotransferase levels assess liver function. Prothrombin time, partial
100 mm Hg). White pulmonary secretions also are normal. The client's rectal thromboplastin time, fibrinogen and fibrin split products are measured to verify
temperature isn't significantly elevated and probably results from the fluid volume bleeding dyscrasias, An electroencephalogram evaluates brain electrical activity.
deficit.
38. Answer: (D) Heparin-associated thrombosis and thrombocytopenia (HATT)
28. Answer: (A) Turn him frequently. Rationale: HATT may occur after CABG surgery due to heparin use during
Rationale: The most important intervention to prevent pressure ulcers is frequent surgery. Although DIC and ITP cause platelet aggregation and bleeding, neither
position changes, which relieve pressure on the skin and underlying tissues. If is common in a client after revascularization surgery. Pancytopenia is a reduction
pressure isn't relieved, capillaries become occluded, reducing circulation and in all blood cells.
oxygenation of the tissues and resulting in cell death and ulcer formation. During
passive ROM exercises, the nurse moves each joint through its range of 39. Answer: (B) Corticosteroids
movement, which improves joint mobility and circulation to the affected area but Rationale: Corticosteroid therapy can decrease antibody production and
doesn't prevent pressure ulcers. Adequate hydration is necessary to maintain phagocytosis of the antibody-coated platelets, retaining more functioning
healthy skin and ensure tissue repair. A footboard prevents plantar flexion and platelets. Methotrexate can cause thrombocytopenia. Vitamin K is used to treat
footdrop by maintaining the foot in a dorsiflexed position. an excessive anticoagulate state from warfarin overload, and ASA decreases
platelet aggregation.
first action of the nurse would be to discontinue the infusion right away to prevent
40. Answer: (D) Xenogeneic further edema and other complication.
Rationale: An xenogeneic transplant is between is between human and another
species. A syngeneic transplant is between identical twins, allogeneic transplant 54. Answer: (C) Chronic obstructive bronchitis
is between two humans, and autologous is a transplant from the same individual. Rationale: Clients with chronic obstructive bronchitis appear bloated; they have
large barrel chest and peripheral edema, cyanotic nail beds, and at times,
41. Answer: (B) circumoral cyanosis. Clients with ARDS are acutely short of breath and
Rationale: Tissue thromboplastin is released when damaged tissue comes in frequently need intubation for mechanical ventilation and large amount of
contact with clotting factors. Calcium is released to assist the conversion of oxygen. Clients with asthma don’t exhibit characteristics of chronic disease, and
factors X to Xa. Conversion of factors XII to XIIa and VIII to VIII a are part of the clients with emphysema appear pink and cachectic.
intrinsic pathway.
55. Answer: (D) Emphysema
42. Answer: (C) Essential thrombocytopenia Rationale: Because of the large amount of energy it takes to breathe, clients with
Rationale: Essential thrombocytopenia is linked to immunologic disorders, such emphysema are usually cachectic. They’re pink and usually breathe through
as SLE and human immunodeficiency vitus. The disorder known as von pursed lips, hence the term “puffer.” Clients with ARDS are usually acutely short
Willebrand’s disease is a type of hemophilia and isn’t linked to SLE. Moderate to of breath. Clients with asthma don’t have any particular characteristics, and
severe anemia is associated with SLE, not polycythermia. Dressler’s syndrome is clients with chronic obstructive bronchitis are bloated and cyanotic in
pericarditis that occurs after a myocardial infarction and isn’t linked to SLE. appearance.

43. Answer: (B) Night sweat 56. Answer: D 80 mm Hg


Rationale: In stage 1, symptoms include a single enlarged lymph node (usually), Rationale: A client about to go into respiratory arrest will have inefficient
unexplained fever, night sweats, malaise, and generalized pruritis. Although ventilation and will be retaining carbon dioxide. The value expected would be
splenomegaly may be present in some clients, night sweats are generally more around 80 mm Hg. All other values are lower than expected.
prevalent. Pericarditis isn’t associated with Hodgkin’s disease, nor is
hypothermia. Moreover, splenomegaly and pericarditis aren’t symptoms. 57. Answer: (C) Respiratory acidosis
Persistent hypothermia is associated with Hodgkin’s but isn’t an early sign of the Rationale: Because Paco2 is high at 80 mm Hg and the metabolic measure,
disease. HCO3- is normal, the client has respiratory acidosis. The pH is less than 7.35,
academic, which eliminates metabolic and respiratory alkalosis as possibilities. If
44. Answer: (D) Breath sounds the HCO3- was below 22 mEq/L the client would have metabolic acidosis.
Rationale: Pneumonia, both viral and fungal, is a common cause of death in
clients with neutropenia, so frequent assessment of respiratory rate and breath 58. Answer: (C) Respiratory failure
sounds is required. Although assessing blood pressure, bowel sounds, and heart Rationale: The client was reacting to the drug with respiratory signs of impending
sounds is important, it won’t help detect pneumonia. anaphylaxis, which could lead to eventually respiratory failure. Although the signs
are also related to an asthma attack or a pulmonary embolism, consider the new
45. Answer: (B) Muscle spasm drug first. Rheumatoid arthritis doesn’t manifest these signs.
Rationale: Back pain or paresthesia in the lower extremities may indicate
impending spinal cord compression from a spinal tumor. This should be 59. Answer: (D) Elevated serum aminotransferase
recognized and treated promptly as progression of the tumor may result in Rationale: Hepatic cell death causes release of liver enzymes alanine
paraplegia. The other options, which reflect parts of the nervous system, aren’t aminotransferase (ALT), aspartate aminotransferase (AST) and lactate
usually affected by MM. dehydrogenase (LDH) into the circulation. Liver cirrhosis is a chronic and
irreversible disease of the liver characterized by generalized inflammation and
46. Answer: (C)10 years fibrosis of the liver tissues.
Rationale: Epidermiologic studies show the average time from initial contact with
HIV to the development of AIDS is 10 years. 60. Answer: (A) Impaired clotting mechanism
Rationale: Cirrhosis of the liver results in decreased Vitamin K absorption and
47. Answer: (A) Low platelet count formation of clotting factors resulting in impaired clotting mechanism.
Rationale: In DIC, platelets and clotting factors are consumed, resulting in
microthrombi and excessive bleeding. As clots form, fibrinogen levels decrease 61. Answer: (B) Altered level of consciousness
and the prothrombin time increases. Fibrin degeneration products increase as Rationale: Changes in behavior and level of consciousness are the first sins of
fibrinolysis takes places. hepatic encephalopathy. Hepatic encephalopathy is caused by liver failure and
develops when the liver is unable to convert protein metabolic product ammonia
48. Answer: (D) Hodgkin’s disease to urea. This results in accumulation of ammonia and other toxic in the blood that
Rationale: Hodgkin’s disease typically causes fever night sweats, weight loss, damages the cells.
and lymph mode enlargement. Influenza doesn’t last for months. Clients with
sickle cell anemia manifest signs and symptoms of chronic anemia with pallor of 62. Answer: (C) “I’ll lower the dosage as ordered so the drug causes only 2 to 4
the mucous membrane, fatigue, and decreased tolerance for exercise; they don’t stools a day”.
show fever, night sweats, weight loss or lymph node enlargement. Leukemia Rationale: Lactulose is given to a patients with hepatic encephalopathy to reduce
doesn’t cause lymph node enlargement. absorption of ammonia in the intestines by binding with ammonia and promoting
more frequent bowel movements. If the patient experience diarrhea, it indicates
49. Answer: (C) A Rh-negative over dosage and the nurse must reduce the amount of medication given to the
Rationale: Human blood can sometimes contain an inherited D antigen. Persons patient. The stool will be mashy or soft. Lactulose is also very sweet and may
with the D antigen have Rh-positive blood type; those lacking the antigen have cause cramping and bloating.
Rh-negative blood. It’s important that a person with Rhnegative blood receives
Rh-negative blood. If Rh-positive blood is administered to an Rh-negative 63. Answer: (B) Severe lower back pain, decreased blood pressure, decreased
person, the recipient develops anti-Rh agglutinins, and sub sequent transfusions RBC count, increased WBC count.
with Rh-positive blood may cause serious reactions with clumping and hemolysis Rationale: Severe lower back pain indicates an aneurysm rupture, secondary to
of red blood cells. pressure being applied within the abdominal cavity. When ruptured occurs, the
pain is constant because it can’t be alleviated until the aneurysm is repaired.
50. Answer: (B) “I will call my doctor if Stacy has persistent vomiting and Blood pressure decreases due to the loss of blood. After the aneurysm ruptures,
diarrhea”. the vasculature is interrupted and blood volume is lost, so blood pressure
Rationale: Persistent (more than 24 hours) vomiting, anorexia, and diarrhea are wouldn’t increase. For the same reason, the RBC count is decreased – not
signs of toxicity and the patient should stop the medication and notify the health increased. The WBC count increases as cell migrate to the site of injury.
care provider. The other manifestations are expected side effects of
chemotherapy. 64. Answer: (D) Apply gloves and assess the groin site
Rationale: Observing standard precautions is the first priority when dealing with
51. Answer: (D) “This is only temporary; Stacy will re-grow new hair in 3-6 any blood fluid. Assessment of the groin site is the second priority. This
months, but may be different in texture”. establishes where the blood is coming from and determineshow much blood has
Rationale: This is the appropriate response. The nurse should help the mother been lost. The goal in this situation is to stop the bleeding. The nurse would call
how to cope with her own feelings regarding the child’s disease so as not to for help if it were warranted after the
affect the child negatively. When the hair grows back, it is still of the same color assessment of the situation. After determining the extent of the bleeding, vital
and texture. signs assessment is important. The nurse should never move the client, in case
a clot has formed. Moving can disturb the clot and cause rebleeding.
52. Answer: (B) Apply viscous Lidocaine to oral ulcers as needed.
Rationale: Stomatitis can cause pain and this can be relieved by applying topical 65. Answer: (D) Percutaneous transluminal coronary angioplasty (PTCA)
anesthetics such as lidocaine before mouth care. When the patient is already Rationale: PTCA can alleviate the blockage and restore blood flow and
comfortable, the nurse can proceed with providing the patient with oral rinses of oxygenation. An echocardiogram is a noninvasive diagnosis test. Nitroglycerin is
saline solution mixed with equal part of water or hydrogen peroxide mixed water an oral sublingual medication. Cardiac catheterization is a diagnostic tool – not a
in 1:3 concentrations to promote oral hygiene. Every 2-4 hours. treatment.

53. Answer: (C) Immediately discontinue the infusion 66. Answer: (B) Cardiogenic shock
Rationale: Edema or swelling at the IV site is a sign that the needle has been Rationale: Cardiogenic shock is shock related to ineffective pumping of the heart.
dislodged and the IV solution is leaking into the tissues causing the edema. The Anaphylactic shock results from an allergic reaction. Distributive shock results
patient feels pain as the nerves are irritated by pressure and the IV solution. The from changes in the intravascular volume distribution and is usually associated
with increased cardiac output. MI isn’t a shock state, though a severe MI can monitor blood glucose levels. A highcarbohydrate diet would exacerbate the
lead to shock. client's condition, particularly if fluid intake is low.

67. Answer: (C) Kidneys’ excretion of sodium and water 79. Answer: (D) Hyperparathyroidism
Rationale: The kidneys respond to rise in blood pressure by excreting sodium Rationale: Hyperparathyroidism is most common in older women and is
and excess water. This response ultimately affects sysmolic blood pressure by characterized by bone pain and weakness from excess parathyroid hormone
regulating blood volume. Sodium or water retention would only further increase (PTH). Clients also exhibit hypercaliuria-causing polyuria. While clients with
blood pressure. Sodium and water travel together across the membrane in the diabetes mellitus and diabetes insipidus also have polyuria, they don't have bone
kidneys; one can’t travel without the other. pain and increased sleeping. Hypoparathyroidism is characterized by urinary
frequency rather than polyuria.
68. Answer: (D) It inhibits reabsorption of sodium and water in the loop of Henle.
Rationale: Furosemide is a loop diuretic that inhibits sodium and water 80. Answer: (C) "I'll take two-thirds of the dose when I wake up and one-third in
reabsorption in the loop Henle, thereby causing a decrease in blood pressure. the late afternoon."
Vasodilators cause dilation of peripheral blood vessels, directly relaxing vascular Rationale: Hydrocortisone, a glucocorticoid, should be administered according to
smooth muscle and decreasing blood pressure. Adrenergic blockers decrease a schedule that closely reflects the body's own secretion of this hormone;
sympathetic cardioacceleration and decrease blood pressure. Angiotensin- therefore, two-thirds of the dose of hydrocortisone should be taken in the
converting enzyme inhibitors decrease blood pressure due to their action on morning and one-third in the late afternoon. This dosage schedule reduces
angiotensin. adverse effects.

69. Answer: (C) Pancytopenia, elevated antinuclear antibody (ANA) titer 81. Answer: (C) High corticotropin and high cortisol levels
Rationale: Laboratory findings for clients with SLE usually show pancytopenia, Rationale: A corticotropin-secreting pituitary tumor would cause high corticotropin
elevated ANA titer, and decreased serum complement levels. Clients may have and high cortisol levels. A high corticotropin level with a low cortisol level and a
elevated BUN and creatinine levels from nephritis, but the increase does not low corticotropin level with a low cortisol level would be associated with
indicate SLE. hypocortisolism. Low corticotropin and high cortisol levels would be seen if there
was a primary defect in the adrenal glands.
70. Answer: (C) Narcotics are avoided after a head injury because they may hide
a worsening condition. 82. Answer: (D) Performing capillary glucose testing every 4 hours
Rationale: Narcotics may mask changes in the level of consciousness that Rationale: The nurse should perform capillary glucose testing every 4 hours
indicate increased ICP and shouldn’t acetaminophen is strong enough ignores because excess cortisol may cause insulin resistance, placing the client at risk
the mother’s question and therefore isn’t appropriate. Aspirin is contraindicated in for hyperglycemia. Urine ketone testing isn't indicated because the client does
conditions that may have bleeding, such as trauma, and for children or young secrete insulin and, therefore, isn't at risk for ketosis. Urine specific gravity isn't
adults with viral illnesses due to the danger of Reye’s syndrome. Stronger indicated because although fluid balance can be compromised, it usually isn't
medications may not necessarily lead to vomiting but will sedate the client, dangerously imbalanced. Temperature regulation may be affected by excess
thereby masking changes in his level of consciousness. cortisol and isn't an accurate indicator of infection.

71. Answer: (A) Appropriate; lowering carbon dioxide (CO2) reduces intracranial 83. Answer: (C) onset to be at 2:30 p.m. and its peak to be at 4 p.m.
pressure (ICP) Rationale: Regular insulin, which is a short-acting insulin, has an onset of 15 to
Rationale: A normal Paco2 value is 35 to 45 mm Hg CO2 has vasodilating 30 minutes and a peak of 2 to 4 hours. Because the nurse gave the insulin at 2
properties; therefore, lowering Paco2 through hyperventilation will lower ICP p.m., the expected onset would be from 2:15 p.m. to 2:30 p.m. and the peak from
caused by dilated cerebral vessels. Oxygenation is evaluated through Pao2 and 4 p.m. to 6 p.m.
oxygen saturation. Alveolar hypoventilation would be reflected in an increased
Paco2. 84. Answer: (A) No increase in the thyroid-stimulating hormone (TSH) level after
30 minutes during the TSH stimulation test
72. Answer: (B) A 33-year-old client with a recent diagnosis of Guillain-Barre Rationale: In the TSH test, failure of the TSH level to rise after 30 minutes
syndrome confirms hyperthyroidism. A decreased TSH level indicates a pituitary deficiency
Rationale: Guillain-Barre syndrome is characterized by ascending paralysis and of this hormone. Below-normal levels of T3 and T4, as detected by
potential respiratory failure. The order of client assessment should follow client radioimmunoassay, signal hypothyroidism. A below-normal T4 level also occurs
priorities, with disorder of airways, breathing, and then circulation. There’s no in malnutrition and liver disease and may result from administration of phenytoin
information to suggest the postmyocardial infarction client has an arrhythmia or and certain other drugs.
other complication. There’s no evidence to suggest hemorrhage or perforation for
the remaining clients as a priority of care. 85. Answer: (B) "Rotate injection sites within the same anatomic region, not
among different regions."
73. Answer: (C) Decreases inflammation Rationale: The nurse should instruct the client to rotate injection sites within the
Rationale: Then action of colchicines is to decrease inflammation by reducing the same anatomic region. Rotating sites among different regions may cause
migration of leukocytes to synovial fluid. Colchicine doesn’t replace estrogen, excessive day-to-day variations in the blood glucose level; also, insulin
decrease infection, or decrease bone demineralization. absorption differs from one region to the next. Insulin should be injected only into
healthy tissue lacking large blood vessels, nerves, or scar tissue or other
74. Answer: (C) Osteoarthritis is the most common form of arthritis deviations. Injecting insulin into areas of hypertrophy may delay absorption. The
Rationale: Osteoarthritis is the most common form of arthritis and can be client shouldn't inject insulin into areas of lipodystrophy (such as hypertrophy or
extremely debilitating. It can afflict people of any age, although most are elderly. atrophy); to prevent lipodystrophy, the client should rotate injection sites
systematically. Exercise speeds drug absorption, so the client shouldn't inject
75. Answer: (C) Myxedema coma insulin into sites above muscles that will be exercised heavily.
Rationale: Myxedema coma, severe hypothyroidism, is a life-threatening
condition that may develop if thyroid replacement medication isn't taken. 86. Answer: (D) Below-normal serum potassium level
Exophthalmos, protrusion of the eyeballs, is seen with hyperthyroidism. Thyroid Rationale: A client with HHNS has an overall body deficit of potassium resulting
storm is life-threatening but is caused by severe hyperthyroidism. Tibial from diuresis, which occurs secondary to the hyperosmolar, hyperglycemic state
myxedema, peripheral mucinous edema involving the lower leg, is associated caused by the relative insulin deficiency. An elevated serum acetone level and
with hypothyroidism but isn't life-threatening. serum ketone bodies are characteristic of diabetic ketoacidosis. Metabolic
acidosis, not serum alkalosis, may occur in HHNS.
76. Answer: (B) An irregular apical pulse
Rationale: Because Cushing's syndrome causes aldosterone overproduction, 87. Answer: (D) Maintaining room temperature in the low-normal range
which increases urinary potassium loss, the disorder may lead to hypokalemia. Rationale: Graves' disease causes signs and symptoms of hypermetabolism,
Therefore, the nurse should immediately report signs and symptoms of such as heat intolerance, diaphoresis, excessive thirst and appetite, and weight
hypokalemia, such as an irregular apical pulse, to the physician. Edema is an loss. To reduce heat intolerance and diaphoresis, the nurse should keep the
expected finding because aldosterone overproduction causes sodium and fluid client's room temperature in the low-normal range. To replace fluids lost via
retention. Dry mucous membranes and frequent urination signal dehydration, diaphoresis, the nurse should encourage, not restrict, intake of oral fluids.
which isn't associated with Cushing's syndrome. Placing extra blankets on the bed of a client with heat intolerance would cause
discomfort. To provide needed energy and calories, the nurse should encourage
77. Answer: (D) Below-normal urine osmolality level, above-normal serum the client to eat high-carbohydrate foods.
osmolality level
Rationale: In diabetes insipidus, excessive polyuria causes dilute urine, resulting 88. Answer: (A) Fracture of the distal radius
in a below-normal urine osmolality level. At the same time, polyuria depletes the Rationale: Colles' fracture is a fracture of the distal radius, such as from a fall on
body of water, causing dehydration that leads to an above-normal serum an outstretched hand. It's most common in women. Colles' fracture doesn't refer
osmolality level. For the same reasons, diabetes insipidus doesn't cause above- to a fracture of the olecranon, humerus, or carpal scaphoid.
normal urine osmolality or below-normal serum osmolality levels.
89. Answer: (B) Calcium and phosphorous
78. Answer: (A) "I can avoid getting sick by not becoming dehydrated and by Rationale: In osteoporosis, bones lose calcium and phosphate salts, becoming
paying attention to my need to urinate, drink, or eat more than usual." porous, brittle, and abnormally vulnerable to fracture. Sodium and potassium
Rationale: Inadequate fluid intake during hyperglycemic episodes often leads to aren't involved in the development of osteoporosis.
HHNS. By recognizing the signs of hyperglycemia (polyuria, polydipsia, and
polyphagia) and increasing fluid intake, the client may prevent HHNS. Drinking a 90. Answer: (A) Adult respiratory distress syndrome (ARDS)
glass of nondiet soda would be appropriate for hypoglycemia. A client whose
diabetes is controlled with oral antidiabetic agents usually doesn't need to
Rationale: Severe hypoxia after smoke inhalation is typically related to ARDS. 5. What is Nurse John likely to note in a male client being admitted for alcohol
The other conditions listed aren’t typically associated with smoke inhalation and withdrawal?
severe hypoxia.
a. Perceptual disorders. b. Impending coma.
91. Answer: (D) Fat embolism c. Recent alcohol intake. d. Depression with mutism.
Rationale: Long bone fractures are correlated with fat emboli, whichcause
shortness of breath and hypoxia. It’s unlikely the client has developed asthma or 6. Aira has taken amitriptyline HCL (Elavil) for 3 days, but now complains that it
bronchitis without a previous history. He could develop atelectasis but it typically “doesn’t help” and refuses to take it. What should the nurse say or do?
doesn’t produce progressive hypoxia.
a. Withhold the drug.
92. Answer: (D) Spontaneous pneumothorax b. Record the client’s response.
Rationale: A spontaneous pneumothorax occurs when the client’s lung collapses, c. Encourage the client to tell the doctor.
causing an acute decreased in the amount of functional lung used in d. Suggest that it takes awhile before seeing the results.
oxygenation. The sudden collapse was the cause of his chest pain and shortness
of breath. An asthma attack would show wheezing breath sounds, and bronchitis 7. Dervid, an adolescent has a history of truancy from school, running away from
would have rhonchi. Pneumonia would have bronchial breath sounds over the home and “barrowing” other people’s things without their permission. The
area of consolidation. adolescent denies stealing, rationalizing instead that as long as no one was
using the items, it was all right to borrow them. It is important for the nurse to
93. Answer: (C) Pneumothorax understand the psychodynamically, this behavior may be largely attributed to a
Rationale: From the trauma the client experienced, it’s unlikely he has bronchitis, developmental defect related to the:
pneumonia, or TB; rhonchi with bronchitis, bronchial breath sounds with TB
would be heard. a. Id b. Ego c. Superego d. Oedipal complex

94. Answer: (C) Serous fluids fills the space and consolidates the region 8. In preparing a female client for electroconvulsive therapy (ECT), Nurse
Rationale: Serous fluid fills the space and eventually consolidates, preventing Michelle knows that succinylcoline (Anectine) will be administered for which
extensive mediastinal shift of the heart and remaining lung. Air can’t be left in the therapeutic effect?
space. There’s no gel that can be placed in the pleural space. The tissue from
the other lung can’t cross the mediastinum, although a temporary mediastinal a. Short-acting anesthesia
shift exits until the space is filled. b. Decreased oral and respiratory secretions.
c. Skeletal muscle paralysis.
95. Answer: (A) Alveolar damage in the infracted area d. Analgesia.
Rationale: The infracted area produces alveolar damage that can lead to the
production of bloody sputum, sometimes in massive amounts. Clot formation 9. Nurse Gina is aware that the dietary implications for a client in manic phase of
usually occurs in the legs. There’s a loss of lung parenchyma and subsequent bipolar disorder is:
scar tissue formation.
a. Serve the client a bowl of soup, buttered French bread, and apple slices.
96. Answer: (D) Respiratory alkalosis b. Increase calories, decrease fat, and decrease protein.
Rationale: A client with massive pulmonary embolism will have a large region c. Give the client pieces of cut-up steak, carrots, and an apple.
and blow off large amount of carbon dioxide, which crosses the unaffected d. Increase calories, carbohydrates, and protein.
alveolar-capillary membrane more readily than does oxygen and results in
respiratory alkalosis. 10.What parental behavior toward a child during an admission procedure should
cause Nurse Ron to suspect child abuse?
97. Answer: (A) Air leak
Rationale: Bubbling in the water seal chamber of a chest drainage system stems a. Flat affect b. Expressing guilt
from an air leak. In pneumothorax an air leak can occur as air is pulled from the c. Acting overly solicitous toward the child. d. Ignoring the child.
pleural space. Bubbling doesn’t normally occur with either adequate or
inadequate suction or any preexisting bubbling in the water seal chamber. 11.Nurse Lynnette notices that a female client with obsessive-compulsive
disorder washes her hands for long periods each day. How should the nurse
98. Answer: (B) 21 respond to this compulsive behavior?
Rationale: 3000 x 10 divided by 24 x 60.
a. By designating times during which the client can focus on the behavior.
99. Answer: (B) 2.4 ml b. By urging the client to reduce the frequency of the behavior as rapidly as
Rationale: .05 mg/ 1 ml = .12mg/ x ml, .05x = .12, x = 2.4 ml. possible.
c. By calling attention to or attempting to prevent the behavior.
100. Answer: (D) “I should put on the stockings before getting out of bed in the d. By discouraging the client from verbalizing anxieties.
morning.
Rationale: Promote venous return by applying external pressure on veins. 12.After seeking help at an outpatient mental health clinic, Ruby who was raped
NP5-MS/PSYCH while walking her dog is diagnosed with posttraumatic stress disorder (PTSD).
1. Mr. Marquez reports of losing his job, not being able to sleep at night, and Three months later, Ruby returns to the clinic, complaining of fear, loss of control,
feeling upset with his wife. Nurse John responds to the client, “You may want to and helpless feelings. Which nursing intervention is most appropriate for Ruby?
talk about your employment situation in group today.” The Nurse is using which
therapeutic technique? a. Recommending a high-protein, low-fat diet.
a. Observations b. Restating c. Exploring d. Focusing b. Giving sleep medication, as prescribed, to restore a normal sleepwake cycle.
c. Allowing the client time to heal.
2. Tony refuses his evening dose of Haloperidol (Haldol), then becomes d. Exploring the meaning of the traumatic event with the client.
extremely agitated in the dayroom while other clients are watching television. He
begins cursing and throwing furniture. Nurse Oliver first action is to: 13.Meryl, age 19, is highly dependent on her parents and fears leaving home to
go away to college. Shortly before the semester starts, she complains that her
a. Check the client’s medical record for an order for an as-needed I.M. dose of legs are paralyzed and is rushed to the emergency department. When physical
medication for agitation. examination rules out a physical cause for her paralysis, the physician admits her
b. Place the client in full leather restraints. to the psychiatric unit where she is diagnosed with conversion disorder. Meryl
c. Call the attending physician and report the behavior. asks the nurse, "Why has this happened to me?" What is the nurse's best
d. Remove all other clients from the dayroom. response?

3. Tina who is manic, but not yet on medication, comes to the drug treatment a. "You've developed this paralysis so you can stay with your parents. You must
center. The nurse would not let this client join the group session because: deal with this conflict if you want to walk again."
b. "It must be awful not to be able to move your legs. You may feel better if you
a. The client is disruptive. b. The client is harmful to self. realize the problem is psychological, not physical."
c. The client is harmful to others d. The client needs to be on medication first. c. "Your problem is real but there is no physical basis for it. We'll work on what is
going on in your life to find out why it's happened."
4. Dervid, an adolescent boy was admitted for substance abuse and d. "It isn't uncommon for someone with your personality to develop a conversion
hallucinations. The client’s mother asks Nurse Armando to talk with his husband disorder during times of stress."
when he arrives at the hospital. The mother says that she is afraid of what the
father might say to the boy. The most appropriate nursing intervention would be 14.Nurse Krina knows that the following drugs have been known to be effective
to: in treating obsessive-compulsive disorder (OCD):

a. Inform the mother that she and the father can work through this problem a. benztropine (Cogentin) and diphenhydramine (Benadryl).
themselves. b. chlordiazepoxide (Librium) and diazepam (Valium)
b. Refer the mother to the hospital social worker. c. fluvoxamine (Luvox) and clomipramine (Anafranil)
c. Agree to talk with the mother and the father together. d. divalproex (Depakote) and lithium (Lithobid)
d. Suggest that the father and son work things out.
15.Alfred was newly diagnosed with anxiety disorder. The physician prescribed d. Listening attentively with a neutral attitude and avoiding power struggles.
buspirone (BuSpar). The nurse is aware that the teaching instructions for newly
prescribed buspirone should include which of the following? 27.Ramon is admitted for detoxification after a cocaine overdose. The client tells
the nurse that he frequently uses cocaine but that he can control his use if he
a. A warning about the drugs delayed therapeutic effect, which is from 14 to 30 chooses. Which coping mechanism is he using?
days.
b. A warning about the incidence of neuroleptic malignant syndrome (NMS). a. Withdrawal b. Logical thinking c. Repression d. Denial
c. A reminder of the need to schedule blood work in 1 week to check blood levels
of the drug. 28.Richard is admitted with a diagnosis of schizotypal personality disorder.
d. A warning that immediate sedation can occur with a resultant drop in pulse. Which signs would this client exhibit during social situations?

16.Richard with agoraphobia has been symptom-free for 4 months. Classic signs a. Aggressive behavior b. Paranoid thoughts
and symptoms of phobias include: c. Emotional affect d. Independence needs

a. Insomnia and an inability to concentrate. 29. Nurse Mickey is caring for a client diagnosed with bulimia. The most
b. Severe anxiety and fear. appropriate initial goal for a client diagnosed with bulimia is to:
c. Depression and weight loss.
d. Withdrawal and failure to distinguish reality from fantasy. a. Avoid shopping for large amounts of food.
b. Control eating impulses.
17.Which medications have been found to help reduce or eliminate panic c. Identify anxiety-causing situations
attacks? d. Eat only three meals per day.

a. Antidepressants b. Anticholinergics c. Antipsychotics d. Mood stabilizers 30.Rudolf is admitted for an overdose of amphetamines. When assessing the
client, the nurse should expect to see:
18.A client seeks care because she feels depressed and has gained weight. To
treat her atypical depression, the physician prescribes tranylcypromine sulfate a. Tension and irritability b. Slow pulse c. Hypotension d. Constipation
(Parnate), 10 mg by mouth twice per day. When this drug is used to treat atypical
depression, what is its onset of action? 31.Nicolas is experiencing hallucinations tells the nurse, “The voices are telling
me I’m no good.” The client asks if the nurse hears the voices. The most
a. 1 to 2 days b. 3 to 5 days c. 6 to 8 days d. 10 to 14 days appropriate response by the nurse would be:

19. A 65 years old client is in the first stage of Alzheimer's disease. Nurse a. “It is the voice of your conscience, which only you can control.”
Patricia should plan to focus this client's care on: b. “No, I do not hear your voices, but I believe you can hear them”.
c. “The voices are coming from within you and only you can hear them.”
a. Offering nourishing finger foods to help maintain the client's nutritional status. d. “Oh, the voices are a symptom of your illness; don’t pay any attention to them.”
b. Providing emotional support and individual counseling.
c. Monitoring the client to prevent minor illnesses from turning into major 32.The nurse is aware that the side effect of electroconvulsive therapy that a
problems. client may experience:
d. Suggesting new activities for the client and family to do together.
a. Loss of appetite b. Postural hypotension
20.The nurse is assessing a client who has just been admitted to the emergency c. Confusion for a time after treatment d. Complete loss of memory for a time
department. Which signs would suggest an overdose of an antianxiety agent?
33.A dying male client gradually moves toward resolution of feelings regarding
a. Combativeness, sweating, and confusion impending death. Basing care on the theory of Kubler-Ross, Nurse Trish plans to
b. Agitation, hyperactivity, and grandiose ideation use nonverbal interventions when assessment reveals that the client is in the:
c. Emotional lability, euphoria, and impaired memory
d. Suspiciousness, dilated pupils, and increased blood pressure a. Anger stage b. Denial stage c. Bargaining stage d. Acceptance stage

21.The nurse is caring for a client diagnosed with antisocial personality disorder. 34.The outcome that is unrelated to a crisis state is:
The client has a history of fighting, cruelty to animals, and stealing. Which of the
following traits would the nurse be most likely to uncover during assessment? a. Learning more constructive coping skills
b. Decompensation to a lower level of functioning.
a. History of gainful employment c. Adaptation and a return to a prior level of functioning.
b. Frequent expression of guilt regarding antisocial behavior d. A higher level of anxiety continuing for more than 3 months.
c. Demonstrated ability to maintain close, stable relationships
d. A low tolerance for frustration 35.Miranda a psychiatric client is to be discharged with orders for haloperidol
(haldol) therapy. When developing a teaching plan for discharge, the nurse
22.Nurse Amy is providing care for a male client undergoing opiate withdrawal. should include cautioning the client against:
Opiate withdrawal causes severe physical discomfort and can be life-threatening.
To minimize these effects, opiate users are commonly detoxified with: a. Driving at night b. Staying in the sun
c. Ingesting wines and cheeses d. Taking medications containing aspirin
a. Barbiturates b. Amphetamines c. Methadone d. Benzodiazepines
36.Jen a nursing student is anxious about the upcoming board examination but is
23.Nurse Cristina is caring for a client who experiences false sensory able to study intently and does not become distracted by a roommate’s talking
perceptions with no basis in reality. These perceptions are known as: and loud music. The student’s ability to ignore distractions and to focus on
studying demonstrates:
a. Delusions b. Hallucinations c. Loose associations d. Neologisms
a. Mild-level anxiety b. Panic-level anxiety
24. Nurse Marco is developing a plan of care for a client with anorexia nervosa. c. Severe-level anxiety d. Moderate-level anxiety
Which action should the nurse include in the plan?
37.When assessing a premorbid personality characteristics of a client with a
a. Restricts visits with the family and friends until the client begins to eat. major depression, it would be unusual for the nurse to find that this client
b. Provide privacy during meals. demonstrated:
c. Set up a strict eating plan for the client.
d. Encourage the client to exercise, which will reduce her anxiety. a. Rigidity b. Stubbornness c. Diverse interest d. Over meticulousness

25.Tim is admitted with a diagnosis of delusions of grandeur. The nurse is aware 38.Nurse Krina recognizes that the suicidal risk for depressed client is greatest:
that this diagnosis reflects a belief that one is:
a. As their depression begins to improve
a. Highly important or famous. b. When their depression is most severe
b. Being persecuted c. Before nay type of treatment is started
c. Connected to events unrelated to oneself d. As they lose interest in the environment
d. Responsible for the evil in the world.
39.Nurse Kate would expect that a client with vascular dementis would
26.Nurse Jen is caring for a male client with manic depression. The plan of care experience:
for a client in a manic state would include:
a. Loss of remote memory related to anoxia
a. Offering a high-calorie meals and strongly encouraging the client to finish all b. Loss of abstract thinking related to emotional state
food. c. Inability to concentrate related to decreased stimuli
b. Insisting that the client remain active through the day so that he’ll sleep at d. Disturbance in recalling recent events related to cerebral hypoxia.
night.
c. Allowing the client to exhibit hyperactive, demanding, manipulative behavior 40.Josefina is to be discharged on a regimen of lithium carbonate. In the
without setting limits. teaching plan for discharge the nurse should include:
a. Advising the client to watch the diet carefully symptoms began at least 2 years ago. Based on this report, the nurse Tyfany
b. Suggesting that the client take the pills with milk suspects:
c. Reminding the client that a CBC must be done once a month.
d. Encouraging the client to have blood levels checked as ordered. a. Cyclothymic disorder. b. Atypical affective disorder.
c. Major depression. d. Dysthymic disorder.
41.The psychiatrist orders lithium carbonate 600 mg p.o t.i.d for a female client.
Nurse Katrina would be aware that the teaching about the side effects of this 53. After taking an overdose of phenobarbital (Barbita), Mario is admitted to the
drug were understood when the client state, “I will call my doctor immediately if I emergency department. Dr. Trinidad prescribes activated charcoal (Charcocaps)
notice any: to be administered by mouth immediately. Before administering the dose, the
nurse verifies the dosage ordered. What is the usual minimum dose of activated
a. Sensitivity to bright light or sun charcoal?
b. Fine hand tremors or slurred speech
c. Sexual dysfunction or breast enlargement a. 5 g mixed in 250 ml of water b. 15 g mixed in 500 ml of water
d. Inability to urinate or difficulty when urinating c. 30 g mixed in 250 ml of water d. 60 g mixed in 500 ml of water

42.Nurse Mylene recognizes that the most important factor necessary for the 54.What herbal medication for depression, widely used in Europe, is now being
establishment of trust in a critical care area is: prescribed in the United States?

a. Privacy b. Respect c. Empathy d. Presence a. Ginkgo biloba b. Echinacea c. St. John's wort d. Ephedra

43.When establishing an initial nurse-client relationship, Nurse Hazel should 55.Cely with manic episodes is taking lithium. Which electrolyte level should the
explore with the client the: nurse check before administering this medication?

a. Client’s perception of the presenting problem. a. Calcium b. Sodium c. Chloride d. Potassium


b. Occurrence of fantasies the client may experience.
c. Details of any ritualistic acts carried out by the client 56.Nurse Josefina is caring for a client who has been diagnosed with delirium.
d. Client’s feelings when external; controls are instituted. Which statement about delirium is true?

44.Tranylcypromine sulfate (Parnate) is prescribed for a depressed client who a. It's characterized by an acute onset and lasts about 1 month.
has not responded to the tricyclic antidepressants. After teaching the client about b. It's characterized by a slowly evolving onset and lasts about 1 week.
the medication, Nurse Marian evaluates that learning has occurred when the c. It's characterized by a slowly evolving onset and lasts about 1 month.
client states, “I will avoid: d. It's characterized by an acute onset and lasts hours to a number of days.

a. Citrus fruit, tuna, and yellow vegetables.” 57.Edward, a 66 year old client with slight memory impairment and poor
b. Chocolate milk, aged cheese, and yogurt’” concentration is diagnosed with primary degenerative dementia of the
c. Green leafy vegetables, chicken, and milk.” Alzheimer's type. Early signs of this dementia include subtle personality changes
d. Whole grains, red meats, and carbonated soda.” and withdrawal from social interactions. To assess for progression to the middle
stage of Alzheimer's disease, the nurse should observe the client for:
45.Nurse John is a aware that most crisis situations should resolve in about:
a. Occasional irritable outbursts. b. Impaired communication.
a. 1 to 2 weeks b. 4 to 6 weeks c. 4 to 6 months d. 6 to 12 months c. Lack of spontaneity. d. Inability to perform self-care activities.

46. Nurse Judy knows that statistics show that in adolescent suicide behavior: 58.Isabel with a diagnosis of depression is started on imipramine (Tofranil), 75
mg by mouth at bedtime. The nurse should tell the client that:
a. Females use more dramatic methods than males
b. Males account for more attempts than do females a. This medication may be habit forming and will be discontinued as soon as the
c. Females talk more about suicide before attempting it client feels better.
d. Males are more likely to use lethal methods than are females b. This medication has no serious adverse effects.
c. The client should avoid eating such foods as aged cheeses, yogurt, and
47. Dervid with paranoid schizophrenia repeatedly uses profanity during an chicken livers while taking the medication.
activity therapy session. Which response by the nurse would be most d. This medication may initially cause tiredness, which should become less
appropriate? bothersome over time.

a. "Your behavior won't be tolerated. Go to your room immediately." 59.Kathleen is admitted to the psychiatric clinic for treatment of anorexia
b. "You're just doing this to get back at me for making you come to therapy." nervosa. To promote the client's physical health, the nurse should plan to:
c. "Your cursing is interrupting the activity. Take time out in your room for 10
minutes." a. Severely restrict the client's physical activities.
d. "I'm disappointed in you. You can't control yourself even for a few minutes." b. Weigh the client daily, after the evening meal.
c. Monitor vital signs, serum electrolyte levels, and acid-base balance.
48.Nurse Maureen knows that the nonantipsychotic medication used to treat d. Instruct the client to keep an accurate record of food and fluid intake.
some clients with schizoaffective disorder is:
60.Celia with a history of polysubstance abuse is admitted to the facility. She
a. phenelzine (Nardil) b. chlordiazepoxide (Librium) complains of nausea and vomiting 24 hours after admission. The nurse assesses
c. lithium carbonate (Lithane) d. imipramine (Tofranil) the client and notes piloerection, pupillary dilation, and lacrimation. The nurse
suspects that the client is going through which of the following withdrawals?
49.Which information is most important for the nurse Trinity to include in a
teaching plan for a male schizophrenic client taking clozapine (Clozaril)? a. Alcohol withdrawal b. Cannibis withdrawal
c. Cocaine withdrawal d. Opioid withdrawal
a. Monthly blood tests will be necessary.
b. Report a sore throat or fever to the physician immediately. 61.Mr. Garcia, an attorney who throws books and furniture around the office after
c. Blood pressure must be monitored for hypertension. losing a case is referred to the psychiatric nurse in the law firm's employee
d. Stop the medication when symptoms subside. assistance program. Nurse Beatriz knows that the client's behavior most likely
represents the use of which defense mechanism?
50.Ricky with chronic schizophrenia takes neuroleptic medication is admitted to
the psychiatric unit. Nursing assessment reveals rigidity, fever, hypertension, and a. Regression b. Projection c. Reaction-formation d. Intellectualization
diaphoresis. These findings suggest which lifethreatening reaction:
62.Nurse Anne is caring for a client who has been treated long term with
a. Tardive dyskinesia. b. Dystonia. antipsychotic medication. During the assessment, Nurse Anne checks the client
c. Neuroleptic malignant syndrome. d. Akathisia. for tardive dyskinesia. If tardive dyskinesia is present, Nurse Anne would most
likely observe:

51.Which nursing intervention would be most appropriate if a male client develop a. Abnormal movements and involuntary movements of the mouth, tongue, and
orthostatic hypotension while taking amitriptyline (Elavil)? face.
b. Abnormal breathing through the nostrils accompanied by a “thrill.”
a. Consulting with the physician about substituting a different type of c. Severe headache, flushing, tremors, and ataxia.
antidepressant. d. Severe hypertension, migraine headache,
b. Advising the client to sit up for 1 minute before getting out of bed.
c. Instructing the client to double the dosage until the problem resolves. 63.Dennis has a lithium level of 2.4 mEq/L. The nurse immediately would assess
d. Informing the client that this adverse reaction should disappear within 1 week. the client for which of the following signs or symptoms?

52.Mr. Cruz visits the physician's office to seek treatment for depression, feelings a. Weakness b. Diarrhea c. Blurred vision d. Fecal incontinence
of hopelessness, poor appetite, insomnia, fatigue, low selfesteem, poor
concentration, and difficulty making decisions. The client states that these
64.Nurse Jannah is monitoring a male client who has been placed inrestraints a. Intellectualization b. Transference c. Triangulation d. Splitting
because of violent behavior. Nurse determines that it will be safe to remove the
restraints when: 78.An 83year-old male client is in extended care facility is anxious most of the
time and frequently complains of a number of vague symptoms that interfere with
a. The client verbalizes the reasons for the violent behavior. his ability to eat. These symptoms indicate which of the following disorders?
b. The client apologizes and tells the nurse that it will never happen again.
c. No acts of aggression have been observed within 1 hour after the release of a. Conversion disorder b. Hypochondriasis c. Severe anxiety d. Sublimation
two of the extremity restraints.
d. The administered medication has taken effect. 79. Charina, a college student who frequently visited the health center during the
past year with multiple vague complaints of GI symptoms before course
65.Nurse Irish is aware that Ritalin is the drug of choice for a child with ADHD. examinations. Although physical causes have been eliminated, the student
The side effects of the following may be noted by the nurse: continues to express her belief that she has a serious illness. These symptoms
are typically of which of the following disorders?
a. Increased attention span and concentration
b. Increase in appetite a. Conversion disorder b. Depersonalization
c. Sleepiness and lethargy c. Hypochondriasis d. Somatization disorder
d. Bradycardia and diarrhea
80. Nurse Daisy is aware that the following pharmacologic agents are
66.Kitty, a 9 year old child has very limited vocabulary and interaction skills. She sedativehypnotic medication is used to induce sleep for a client experiencing a
has an I.Q. of 45. She is diagnosed to have Mental retardation of this sleep disorder is:
classification:
a. Triazolam (Halcion) b. Paroxetine (Paxil)\
a. Profound b. Mild c. Moderate d. Severe c. Fluoxetine (Prozac) d. Risperidone (Risperdal)

67.The therapeutic approach in the care of Armand an autistic child include the 81. Aldo, with a somatoform pain disorder may obtain secondary gain. Which of
following EXCEPT: the following statement refers to a secondary gain?

a. Engage in diversionary activities when acting -out a. It brings some stability to the family
b. Provide an atmosphere of acceptance b. It decreases the preoccupation with the physical illness
c. Provide safety measures c. It enables the client to avoid some unpleasant activity
d. Rearrange the environment to activate the child d. It promotes emotional support or attention for the client

68.Jeremy is brought to the emergency room by friends who state that he took 82. Dervid is diagnosed with panic disorder with agoraphobia is talking with the
something an hour ago. He is actively hallucinating, agitated, with irritated nasal nurse in-charge about the progress made in treatment. Which of the following
septum. statements indicates a positive client response?

a. Heroin b. Cocaine c. LSD d. Marijuana a. “I went to the mall with my friends last Saturday”
b. “I’m hyperventilating only when I have a panic attack”
69.Nurse Pauline is aware that Dementia unlike delirium is characterized by: c. “Today I decided that I can stop taking my medication”
d. “Last night I decided to eat more than a bowl of cereal”
a. Slurred speech b. Insidious onset
c. Clouding of consciousness d. Sensory perceptual change 83. The effectiveness of monoamine oxidase (MAO) inhibitor drug therapy in
client with posttraumatic stress disorder can be demonstrated by which of the
70.A 35 year old female has intense fear of riding an elevator. She claims “ As if I following client self –reports?
will die inside.” The client is suffering from:
a. “I’m sleeping better and don’t have nightmares”
a. Agoraphobia b. Social phobia c. Claustrophobia d. Xenophobia b. “I’m not losing my temper as much”
c. “I’ve lost my craving for alcohol”
71.Nurse Myrna develops a counter-transference reaction. This is evidenced by: d. I’ve lost my phobia for water”

a. Revealing personal information to the client 84. Mark, with a diagnosis of generalized anxiety disorder wants to stop taking
b. Focusing on the feelings of the client. his lorazepam (Ativan). Which of the following important facts should nurse Betty
c. Confronting the client about discrepancies in verbal or non-verbal behavior discuss with the client about discontinuing the medication?
d. The client feels angry towards the nurse who resembles his mother.
a. Stopping the drug may cause depression
72.Tristan is on Lithium has suffered from diarrhea and vomiting. What should b. Stopping the drug increases cognitive abilities
the nurse in-charge do first: c. Stopping the drug decreases sleeping difficulties
d. Stopping the drug can cause withdrawal symptoms
a. Recognize this as a drug interaction
b. Give the client Cogentin 85. Jennifer, an adolescent who is depressed and reported by his parents as
c. Reassure the client that these are common side effects of lithium therapy having difficulty in school is brought to the community mental health center to be
d. Hold the next dose and obtain an order for a stat serum lithium level evaluated. Which of the following other health problems would the nurse
suspect?
73.Nurse Sarah ensures a therapeutic environment for all the client. Which of the
following best describes a therapeutic milieu? a. Anxiety disorder b. Behavioral difficulties
c. Cognitive impairment d. Labile moods
a. A therapy that rewards adaptive behavior
b. A cognitive approach to change behavior 86. Ricardo, an outpatient in psychiatric facility is diagnosed with dysthymic
c. A living, learning or working environment. disorder. Which of the following statement about dysthymic disorder is true?
d. A permissive and congenial environment
a. It involves a mood range from moderate depression to hypomania
74.Anthony is very hostile toward one of the staff for no apparent reason. He is b. It involves a single manic depression
manifesting: c. It’s a form of depression that occurs in the fall and winter
d. It’s a mood disorder similar to major depression but of mild to moderate
a. Splitting b. Transference c. Countertransference d. Resistance severity

75.Marielle, 17 years old was sexually attacked while on her way home from 87. The nurse is aware that the following ways in vascular dementia different
school. She is brought to the hospital by her mother. Rape is an example of from Alzheimer’s disease is:
which type of crisis:
a. Vascular dementia has more abrupt onset
a. Situational b. Adventitious c. Developmental d. Internal b. The duration of vascular dementia is usually brief
c. Personality change is common in vascular dementia
76. Nurse Greta is aware that the following is classified as an Axis I disorder by d. The inability to perform motor activities occurs in vascular dementia
the Diagnosis and Statistical Manual of Mental Disorders, Text Revision (DSM-
IV-TR) is: 88. Loretta, a newly admitted client was diagnosed with delirium and has history
of hypertension and anxiety. She had been taking digoxin, furosemide (Lasix),
a. Obesity b. Borderline personality disorder and diazepam (Valium) for anxiety. This client’s impairment may be related to
c. Major depression d. Hypertension which of the following conditions?

77.Katrina, a newly admitted is extremely hostile toward a staff member she has a. Infection b. Metabolic acidosis c. Drug intoxication d. Hepatic encephalopathy
just met, without apparent reason. According to Freudian theory, the nurse
should suspect that the client is experiencing which of the following phenomena?
89. Nurse Ron enters a client’s room, the client says, “They’re crawling on my RATIO 5
sheets! Get them off my bed!” Which of the following assessment is the most 1. Answer: (D) Focusing
accurate? Rationale: The nurse is using focusing by suggesting that the client discuss a
specific issue. The nurse didn’t restate the question, make observation, or ask
a. The client is experiencing aphasia further question (exploring).
b. The client is experiencing dysarthria
c. The client is experiencing a flight of ideas 2. Answer: (D) Remove all other clients from the dayroom.
d. The client is experiencing visual hallucination Rationale: The nurse’s first priority is to consider the safety of the clients in the
therapeutic setting. The other actions are appropriate responses after ensuring
90. Which of the following descriptions of a client’s experience and behavior can the safety of other clients.
be assessed as an illusion?
3. Answer: (A) The client is disruptive.
a. The client tries to hit the nurse when vital signs must be taken Rationale: Group activity provides too much stimulation, which the client will not
b. The client says, “I keep hearing a voice telling me to run away” be able to handle (harmful to self) and as a result will be disruptive to others.
c. The client becomes anxious whenever the nurse leaves the bedside
d. The client looks at the shadow on a wall and tells the nurse she sees 4. Answer: (C) Agree to talk with the mother and the father together.
frightening faces on the wall. Rationale: By agreeing to talk with both parents, the nurse can provide emotional
support and further assess and validate the family’s needs.
91. During conversation of Nurse John with a client, he observes that the client
shift from one topic to the next on a regular basis. Which of the following terms 5. Answer: (A) Perceptual disorders.
describes this disorder? Rationale: Frightening visual hallucinations are especially common in clients
experiencing alcohol withdrawal.
a. Flight of ideas b. Concrete thinking
c. Ideas of reference d. Loose association 6. Answer: (D) Suggest that it takes awhile before seeing the results.
Rationale: The client needs a specific response; that it takes 2 to 3 weeks (a
92. Francis tells the nurse that her coworkers are sabotaging the computer. delayed effect) until the therapeutic blood level is reached.
When the nurse asks questions, the client becomes argumentative. This
behavior shows personality traits associated with which of the following 7. Answer: (C) Superego
personality disorder? Rationale: This behavior shows a weak sense of moral consciousness.
According to Freudian theory, personality disorders stem from a weak superego.
a. Antisocial b. Histrionic c. Paranoid d. Schizotypal
8. Answer: (C) Skeletal muscle paralysis.
93. Which of the following interventions is important for a Cely experiencing with Rationale: Anectine is a depolarizing muscle relaxant causing paralysis. It is used
paranoid personality disorder taking olanzapine (Zyprexa)? to reduce the intensity of muscle contractions during the convulsive stage,
thereby reducing the risk of bone fractures or dislocation.
a. Explain effects of serotonin syndrome
b. Teach the client to watch for extrapyramidal adverse reaction 9. Answer: (D) Increase calories, carbohydrates, and protein.
c. Explain that the drug is less affective if the client smokes Rationale: This client increased protein for tissue building and increased calories
d. Discuss the need to report paradoxical effects such as euphoria to replace what is burned up (usually via carbohydrates).

94. Nurse Alexandra notices other clients on the unit avoiding a client diagnosed 10. Answer: (C) Acting overly solicitous toward the child.
with antisocial personality disorder. When discussing appropriate behavior in Rationale: This behavior is an example of reaction formation, a coping
group therapy, which of the following comments is expected about this client by mechanism.
his peers?
11. Answer: (A) By designating times during which the client can focus on the
a. Lack of honesty b. Belief in superstition behavior.
c. Show of temper tantrums d. Constant need for attention Rationale: The nurse should designate times during which the client can focus on
the compulsive behavior or obsessive thoughts. The nurse should urge the client
95. Tommy, with dependent personality disorder is working to increase his to reduce the frequency of the compulsive behavior gradually, not rapidly. She
selfesteem. Which of the following statements by the Tommy shows teaching shouldn't call attention to or try to prevent the behavior. Trying to prevent the
was successful? behavior may cause pain and terror in the client. The nurse should encourage
the client to verbalize anxieties to help distract attention from the compulsive
a. “I’m not going to look just at the negative things about myself” behavior.
b. “I’m most concerned about my level of competence and progress”
c. “I’m not as envious of the things other people have as I used to be” 12. Answer: (D) Exploring the meaning of the traumatic event with the client.
d. “I find I can’t stop myself from taking over things other should be doing” Rationale: The client with PTSD needs encouragement to examine and
understand the meaning of the traumatic event and consequent losses.
96. Norma, a 42-year-old client with a diagnosis of chronic undifferentiated Otherwise, symptoms may worsen and the client may become depressed or
schizophrenia lives in a rooming house that has a weekly nursing clinic. She engage in self-destructive behavior such as substance abuse. The client must
scratches while she tells the nurse she feels creatures eating away at her skin. explore the meaning of the event and won't heal without this, no matter how
Which of the following interventions should be done first? much time passes. Behavioral techniques, such as relaxation therapy, may help
decrease the client's anxiety and induce sleep. The physician may prescribe
a. Talk about his hallucinations and fears antianxiety agents or antidepressants cautiously to avoid dependence; sleep
b. Refer him for anticholinergic adverse reactions medication is rarely appropriate. A special diet isn't indicated unless the client
c. Assess for possible physical problems such as rash also has an eating disorder or a nutritional problem.
d. Call his physician to get his medication increased to control his psychosis
13. Answer: (C) "Your problem is real but there is no physical basis for it. We'll
97. Ivy, who is on the psychiatric unit is copying and imitating the movements of work on what is going on in your life to find out why it's happened."
her primary nurse. During recovery, she says, “I thought the nurse was my Rationale: The nurse must be honest with the client by telling her that the
mirror. I felt connected only when I saw my nurse.” This behavior is known by paralysis has no physiologic cause while also conveying empathy and
which of the following terms? acknowledging that her symptoms are real. The client will benefit from psychiatric
treatment, which will help her understand the underlying cause of her symptoms.
a. Modeling b. Echopraxia c. Ego-syntonicity d. Ritualism After the psychological conflict is resolved, her
symptoms will disappear. Saying that it must be awful not to be able to move her
98. Jun approaches the nurse and tells that he hears a voice telling him that he’s legs wouldn't answer the client's question; knowing that the cause is
evil and deserves to die. Which of the following terms describes the client’s psychological wouldn't necessarily make her feel better. Telling her that she has
perception? developed paralysis to avoid leaving her parents or that her personality caused
her disorder wouldn't help her understand and resolve the underlying conflict.
a. Delusion b. Disorganized speech c. Hallucination d. Idea of reference
14. Answer: (C) fluvoxamine (Luvox) and clomipramine (Anafranil)
99. Mike is admitted to a psychiatric unit with a diagnosis of undifferentiated Rationale: The antidepressants fluvoxamine and clomipramine have been
schizophrenia. Which of the following defense mechanisms is probably used by effective in the treatment of OCD. Librium and Valium may be helpful in treating
mike? anxiety related to OCD but aren't drugs of choice to treat the illness. The other
medications mentioned aren't effective in the treatment of OCD.
a. Projection b. Rationalization c. Regression d. Repression
15. Answer: (A) A warning about the drugs delayed therapeutic effect, which is
100. Rocky has started taking haloperidol (Haldol). Which of the following from 14 to 30 days.
instructions is most appropriate for Ricky before taking haloperidol? Rationale: The client should be informed that the drug's therapeutic effect might
not be reached for 14 to 30 days. The client must be instructed to continue taking
a. Should report feelings of restlessness or agitation at once the drug as directed. Blood level checks aren't necessary. NMS hasn't been
b. Use a sunscreen outdoors on a year-round basis reported with this drug, but tachycardia is frequently reported.
c. Be aware you’ll feel increased energy taking this drug
d. This drug will indirectly control essential hypertension 16. Answer: (B) Severe anxiety and fear.
Rationale: Phobias cause severe anxiety (such as a panic attack) that is out of and make responsible decisions, which would lead the client admitting the
proportion to the threat of the feared object or situation. Physical signs and problem and seeking help. Repression is suppressing past events from the
symptoms of phobias include profuse sweating, poor motor control, tachycardia, consciousness because of guilty association.
and elevated blood pressure. Insomnia, an inability to concentrate, and weight
loss are common in depression. Withdrawal and failure to distinguish reality from 28. Answer: (B) Paranoid thoughts
fantasy occur in schizophrenia. Rationale: Clients with schizotypal personality disorder experience excessive
social anxiety that can lead to paranoid thoughts. Aggressive behavior is
17. Answer: (A) Antidepressants uncommon, although these clients may experience agitation with anxiety. Their
Rationale: Tricyclic and monoamine oxidase (MAO) inhibitor antidepressants behavior is emotionally cold with a flattened affect, regardless of the situation.
have been found to be effective in treating clients with panic attacks. Why these These clients demonstrate a reduced capacity for close or dependent
drugs help control panic attacks isn't clearly understood. Anticholinergic agents, relationships.
which are smooth-muscle relaxants, relieve physical symptoms of anxiety but
don't relieve the anxiety itself. Antipsychotic drugs are inappropriate because 29. Answer: (C) Identify anxiety-causing situations
clients who experience panic attacks aren't psychotic. Mood stabilizers aren't Rationale: Bulimic behavior is generally a maladaptive coping response to stress
indicated because panic attacks are rarely associated with mood changes. and underlying issues. The client must identify anxiety-causing situations that
stimulate the bulimic behavior and then learn new ways of
18. Answer: (B) 3 to 5 days coping with the anxiety.
Rationale: Monoamine oxidase inhibitors, such as tranylcypromine, have an
onset of action of approximately 3 to 5 days. A full clinical response may be 30. Answer: (A) Tension and irritability
delayed for 3 to 4 weeks. The therapeutic effects may continue for 1 to 2 weeks Rationale: An amphetamine is a nervous system stimulant that is subject to
after discontinuation. abuse because of its ability to produce wakefulness and euphoria. An overdose
increases tension and irritability. Options B and C are incorrect because
19. Answer: (B) Providing emotional support and individual counseling. amphetamines stimulate norepinephrine, which increase the heart rate and blood
Rationale: Clients in the first stage of Alzheimer's disease are aware that flow. Diarrhea is a common adverse effect so option D in is incorrect.
something is happening to them and may become overwhelmed and frightened.
Therefore, nursing care typically focuses on providing emotional support and 31. Answer: (B) “No, I do not hear your voices, but I believe you can hear them”.
individual counseling. The other options are appropriate during the second stage Rationale: The nurse, demonstrating knowledge and understanding, accepts the
of Alzheimer's disease, when the client’s perceptions even though they are hallucinatory.
client needs continuous monitoring to prevent minor illnesses from progressing
into major problems and when maintaining adequate nutrition may become a 32. Answer: (C) Confusion for a time after treatment
challenge. During this stage, offering nourishing finger foods helps clients to feed Rationale: The electrical energy passing through the cerebral cortex during ECT
themselves and maintain adequate nutrition. results in a temporary state of confusion after treatment.

20. Answer: (C) Emotional lability, euphoria, and impaired memory 33. Answer: (D) Acceptance stage
Rationale: Signs of antianxiety agent overdose include emotional lability, Rationale: Communication and intervention during this stage are mainly
euphoria, and impaired memory. Phencyclidine overdose can cause nonverbal, as when the client gestures to hold the nurse’s hand.
combativeness, sweating, and confusion. Amphetamine overdose can result in
agitation, hyperactivity, and grandiose ideation. Hallucinogen overdose can 34. Answer: (D) A higher level of anxiety continuing for more than 3 months.
produce suspiciousness, dilated pupils, and increased blood pressure. Rationale: This is not an expected outcome of a crisis because by definition a
crisis would be resolved in 6 weeks.
21. Answer: (D) A low tolerance for frustration
Rationale: Clients with an antisocial personality disorder exhibit a low tolerance 35. Answer: (B) Staying in the sun
for frustration, emotional immaturity, and a lack of impulse control. They Rationale: Haldol causes photosensitivity. Severe sunburn can occur on
commonly have a history of unemployment, miss work repeatedly, and quit work exposure to the sun.
without other plans for employment. They don't feel guilt about their behavior and
commonly perceive themselves as victims. They also display a lack of 36. Answer: (D) Moderate-level anxiety
responsibility for the outcome of their actions. Because of a lack of trust in others, Rationale: A moderately anxious person can ignore peripheral events and
clients with antisocial personality disorder commonly have difficulty developing focuses on central concerns.
stable, close relationships.
37. Answer: (C) Diverse interest
22. Answer: (C) Methadone Rationale: Before onset of depression, these clients usually have very narrow,
Rationale: Methadone is used to detoxify opiate users because it binds with limited interest.
opioid receptors at many sites in the central nervous system but doesn’t have the
same deterious effects as other opiates, such as cocaine, heroin, and morphine. 38. Answer: (A) As their depression begins to improve
Barbiturates, amphetamines, and benzodiazepines are highly addictive and Rationale: At this point the client may have enough energy to plan and execute
would require detoxification treatment. an attempt.

23. Answer: (B) Hallucinations 39. Answer: (D) Disturbance in recalling recent events related to cerebral
Rationale: Hallucinations are visual, auditory, gustatory, tactile, or olfactory hypoxia.
perceptions that have no basis in reality. Delusions are false beliefs, rather than Rationale: Cell damage seems to interfere with registering input stimuli, which
perceptions, that the client accepts as real. Loose associations are rapid shifts affects the ability to register and recall recent events; vascular dementia is
among unrelated ideas. Neologisms are bizarre words that have meaning only to related to multiple vascular lesions of the cerebral cortex and subcortical
the client. structure.

24. Answer: (C) Set up a strict eating plan for the client. 40. Answer: (D) Encouraging the client to have blood levels checked as ordered.
Rationale: Establishing a consistent eating plan and monitoring the client’s Rationale: Blood levels must be checked monthly or bimonthly when the client is
weight are very important in this disorder. The family and friends should be on maintenance therapy because there is only a small range between
included in the client’s care. The client should be monitored during meals-not therapeutic and toxic levels.
given privacy. Exercise must be limited and supervised.
41. Answer: (B) Fine hand tremors or slurred speech
25. Answer: (A) Highly important or famous. Rationale: These are common side effects of lithium carbonate.
Rationale: A delusion of grandeur is a false belief that one is highly important or
famous. A delusion of persecution is a false belief that one is being persecuted. 42. Answer: (D) Presence
A delusion of reference is a false belief that one is connected to events unrelated Rationale: The constant presence of a nurse provides emotional support
to oneself or a belief that one is responsible for the evil in the world. because the client knows that someone is attentive and available in case of an
emergency.
26. Answer: (D) Listening attentively with a neutral attitude and avoiding power
struggles. 43. Answer: (A) Client’s perception of the presenting problem.
Rationale: The nurse should listen to the client’s requests, express willingness to Rationale: The nurse can be most therapeutic by starting where the client is,
seriously consider the request, and respond later. The nurse should encourage because it is the client’s concept of the problem that serves as the starting point
the client to take short daytime naps because he expends so much energy. The of the relationship.
nurse shouldn’t try to restrain the client when he feels the need to move around
as long as his activity isn’t harmful. High calorie finger foods should be offered to 44. Answer: (B) Chocolate milk, aged cheese, and yogurt’”
supplement the client’s diet, if he can’t remain seated long enough to eat a Rationale: These high-tyramine foods, when ingested in the presence of an MAO
complete meal. The nurse shouldn’t be forced to stay seated at the table to finish inhibitor, cause a severe hypertensive response.
a meal. The nurse should set limits in a calm, clear, and self-confident tone of
voice. 45. Answer: (B) 4 to 6 weeks
Rationale: Crisis is self-limiting and lasts from 4 to 6 weeks.
27. Answer: (D) Denial
Rationale: Denial is unconscious defense mechanism in which emotional conflict 46. Answer: (D) Males are more likely to use lethal methods than are females
and anxiety is avoided by refusing to acknowledge feelings, desires, impulses, or Rationale: This finding is supported by research; females account for 90% of
external facts that are consciously intolerable. Withdrawal is a common response suicide attempts but males are three times more successful because of methods
to stress, characterized by apathy. Logical thinking is the ability to think rationally used.
irritable outbursts and lack of spontaneity, the client is usually cooperative and
47. Answer: (C) "Your cursing is interrupting the activity. Take time out in your exhibits socially appropriate behavior. Signs of
room for 10 minutes." advancement to the middle stage of Alzheimer's disease include exacerbated
Rationale: The nurse should set limits on client behavior to ensure a comfortable cognitive impairment with obvious personality changes and impaired
environment for all clients. The nurse should accept hostile or quarrelsome client communication, such as inappropriate conversation, actions, and responses.
outbursts within limits without becoming personally offended, as in option A. During the late stage, the client can't perform self-care activities and may
Option B is incorrect because it implies that the client's actions reflect feelings become mute.
toward the staff instead of the client's own misery. Judgmental remarks, such as
option D, may decrease the client's self-esteem. 58. Answer: (D) This medication may initially cause tiredness, which should
become less bothersome over time.
48. Answer: (C) lithium carbonate (Lithane) Rationale: Sedation is a common early adverse effect of imipramine, a tricyclic
Rationale: Lithium carbonate, an antimania drug, is used to treat clients with antidepressant, and usually decreases as tolerance develops. Antidepressants
cyclical schizoaffective disorder, a psychotic disorder once classified under aren't habit forming and don't cause physical or psychological dependence.
schizophrenia that causes affective symptoms, including maniclike activity. However, after a long course of high-dose therapy, the dosage should be
Lithium helps control the affective component of this disorder. Phenelzine is a decreased gradually to avoid mild withdrawal symptoms. Serious adverse
monoamine oxidase inhibitor prescribed for clients who don't respond to other effects, although rare, include myocardial infarction, heart failure, and
antidepressant drugs such as imipramine. Chlordiazepoxide, an antianxiety tachycardia. Dietary restrictions, such as avoiding aged cheeses, yogurt, and
agent, generally is contraindicated in psychotic clients. Imipramine, primarily chicken livers, are necessary for a client taking a monoamine oxidase inhibitor,
considered an antidepressant agent, is also used to treat clients with not a tricyclic antidepressant.
agoraphobia and that undergoing cocaine detoxification.
59. Answer: (C) Monitor vital signs, serum electrolyte levels, and acid-base
49. Answer: (B) Report a sore throat or fever to the physician immediately. balance.
Rationale: A sore throat and fever are indications of an infection caused by Rationale: An anorexic client who requires hospitalization is in poor physical
agranulocytosis, a potentially life-threatening complication of clozapine. Because condition from starvation and may die as a result of arrhythmias, hypothermia,
of the risk of agranulocytosis, white blood cell (WBC) counts are malnutrition, infection, or cardiac abnormalities secondary to electrolyte
necessary weekly, not monthly. If the WBC count drops below 3,000/μl, the imbalances. Therefore, monitoring the client's vital signs, serum electrolyte level,
medication must be stopped. Hypotension may occur in clients taking this and acid base balance is crucial. Option A may worsen anxiety. Option B is
medication. Warn the client to stand up slowly to avoid dizziness from orthostatic incorrect because a weight obtained after breakfast is more accurate than one
hypotension. The medication should be continued, even when symptoms have obtained after the evening meal. Option D would reward the client with attention
been controlled. If the medication must be stopped, it should be slowly tapered for not eating and reinforce the control issues that are central to the underlying
over 1 to 2 weeks and only under the supervision of a physician. psychological problem; also, the client may record food and fluid intake
inaccurately.
50. Answer: (C) Neuroleptic malignant syndrome.
Rationale: The client's signs and symptoms suggest neuroleptic malignant 60. Answer: (D) Opioid withdrawal
syndrome, a life-threatening reaction to neuroleptic medication that requires Rationale: The symptoms listed are specific to opioid withdrawal. Alcohol
immediate treatment. Tardive dyskinesia causes involuntary movements of the withdrawal would show elevated vital signs. There is no real withdrawal from
tongue, mouth, facial muscles, and arm and leg muscles. Dystonia is cannibis. Symptoms of cocaine withdrawal include depression, anxiety, and
characterized by cramps and rigidity of the tongue, agitation.
face, neck, and back muscles. Akathisia causes restlessness, anxiety, and
jitteriness. 61. Answer: (A) Regression
Rationale: An adult who throws temper tantrums, such as this one, is displaying
51. Answer: (B) Advising the client to sit up for 1 minute before getting out of bed. regressive behavior, or behavior that is appropriate at a younger age. In
Rationale: To minimize the effects of amitriptyline-induced orthostatic projection, the client blames someone or something other than the source. In
hypotension, the nurse should advise the client to sit up for 1 minute before reaction formation, the client acts in opposition to his feelings. In
getting out of bed. Orthostatic hypotension commonly occurs with tricyclic intellectualization, the client overuses rational explanations or
antidepressant therapy. In these cases, the dosage may be reduced or the abstract thinking to decrease the significance of a feeling or event.
physician may prescribe nortriptyline, another tricyclic
antidepressant. Orthostatic hypotension disappears only when the drug is 62. Answer: (A) Abnormal movements and involuntary movements of the mouth,
discontinued. tongue, and face.
Rationale: Tardive dyskinesia is a severe reaction associated with long term use
52. Answer: (D) Dysthymic disorder. of antipsychotic medication. The clinical manifestations include abnormal
Rationale: Dysthymic disorder is marked by feelings of depression lasting at least movements (dyskinesia) and involuntary movements of the mouth, tongue (fly
2 years, accompanied by at least two of the following symptoms: sleep catcher tongue), and face.
disturbance, appetite disturbance, low energy or fatigue, low selfesteem, poor
concentration, difficulty making decisions, and hopelessness. These symptoms 63. Answer: (C) Blurred vision
may be relatively continuous or Rationale: At lithium levels of 2 to 2.5 mEq/L the client will experienced blurred
separated by intervening periods of normal mood that last a few days to a few vision, muscle twitching, severe hypotension, and persistent nausea and
weeks. Cyclothymic disorder is a chronic mood disturbance of at least 2 years' vomiting. With levels between 1.5 and 2 mEq/L the client experiencing vomiting,
duration marked by numerous periods of depression and hypomania. Atypical diarrhea, muscle weakness, ataxia, dizziness, slurred speech, and confusion. At
affective disorder is characterized by manic signs and symptoms. Major lithium levels of 2.5 to 3 mEq/L or higher, urinary and fecal incontinence occurs,
depression is a recurring, persistent sadness or loss of interest or pleasure in as well as seizures, cardiac dysrythmias, peripheral vascular collapse, and
almost all activities, with signs and symptoms recurring for at least 2 weeks. death.

53. Answer: (C) 30 g mixed in 250 ml of water 64. Answer: (C) No acts of aggression have been observed within 1 hour after
Rationale: The usual adult dosage of activated charcoal is 5 to 10 times the the release of two of the extremity restraints.
estimated weight of the drug or chemical ingested, or a minimum dose of 30 g, Rationale: The best indicator that the behavior is controlled, if the client exhibits
mixed in 250 ml of water. Doses less than this will be ineffective; doses greater no signs of aggression after partial release of restraints. Options A, B, and D do
than this can increase the risk of adverse reactions, although toxicity doesn't not ensure that the client has controlled the behavior.
occur with activated charcoal, even at the maximum dose.
65. Answer: (A) increased attention span and concentration
54. Answer: (C) St. John's wort Rationale: The medication has a paradoxic effect that decrease hyperactivity and
Rationale: St. John's wort has been found to have serotonin-elevating properties, impulsivity among children with ADHD. B, C, D. Side effects of Ritalin include
similar to prescription antidepressants. Ginkgo biloba is prescribed to enhance anorexia, insomnia, diarrhea and irritability.
mental acuity. Echinacea has immune-stimulating properties. Ephedra is a
naturally occurring stimulant that is similar to ephedrine. 66. Answer: (C) Moderate
Rationale: The child with moderate mental retardation has an I.Q. of 35- 50
55. Answer: (B) Sodium Profound Mental retardation has an I.Q. of below 20; Mild mental retardation 50-
Rationale: Lithium is chemically similar to sodium. If sodium levels are reduced, 70 and Severe mental retardation has an I.Q. of 20-35.
such as from sweating or diuresis, lithium will be reabsorbed by the kidneys,
increasing the risk of toxicity. Clients taking lithium shouldn't restrict their intake 67. Answer: (D) Rearrange the environment to activate the child
of sodium and should drink adequate amounts of fluid each day. The other Rationale: The child with autistic disorder does not want change. Maintaining a
electrolytes are important for normal body functions but sodium is most important consistent environment is therapeutic. A. Angry outburst can be re-channeling
to the absorption of lithium. through safe activities. B. Acceptance enhances a trusting relationship. C.
Ensure safety from self-destructive behaviors like head banging and hair pulling.
56. Answer: (D) It's characterized by an acute onset and lasts hours to a number
of days 68. Answer: (B) cocaine
Rationale: Delirium has an acute onset and typically can last from several hours Rationale: The manifestations indicate intoxication with cocaine, a CNS
to several days. stimulant. A. Intoxication with heroine is manifested by euphoria then impairment
in judgment, attention and the presence of papillary constriction. C. Intoxication
57. Answer: (B) Impaired communication. with hallucinogen like LSD is manifested by grandiosity, hallucinations,
Rationale: Initially, memory impairment may be the only cognitive deficit in a synesthesia and increase in vital signs D. Intoxication with Marijuana, a
client with Alzheimer's disease. During the early stage of this disease, subtle cannabinoid is manifested by sensation of slowed time, conjunctival redness,
personality changes may also be present. However, other than occasional social withdrawal, impaired judgment and hallucinations.
some unpleasant activity. A dysfunctional family may disregard the real issue,
69. Answer: (B) insidious onset although some conflict is relieved. Somatoform pain disorder is a preoccupation
Rationale: Dementia has a gradual onset and progressive deterioration. It causes with pain in the absence of physical disease.
pronounced memory and cognitive disturbances. A,C and D are all
characteristics of delirium. 82. Answer: (A) “I went to the mall with my friends last Saturday”
Rationale: Clients with panic disorder tent to be socially withdrawn. Going to the
70. Answer: (C) Claustrophobia mall is a sign of working on avoidance behaviors. Hyperventilating is a key
Rationale: Claustrophobia is fear of closed space. A. Agoraphobia is fear of open symptom of panic disorder. Teaching breathing control is a major intervention for
space or being a situation where escape is difficult. B. Social phobia is fear of clients with panic disorder. The client taking medications for panic disorder; such
performing in the presence of others in a way that will be humiliating or as tricylic antidepressants and benzodiazepines, must be weaned off these
embarrassing. D. Xenophobia is fear of strangers. drugs. Most clients with panic disorder with agoraphobia don’t have nutritional
problems.
71. Answer: (A) Revealing personal information to the client
Rationale: Counter-transference is an emotional reaction of the nurse on the 83. Answer: (A) “I’m sleeping better and don’t have nightmares”
client based on her unconscious needs and conflicts. B and C. These are Rationale:MAO inhibitors are used to treat sleep problems, nightmares, and
therapeutic approaches. D. This is transference reaction where a client has an intrusive daytime thoughts in individual with posttraumatic stress disorder. MAO
emotional reaction towards the nurse based on her past. inhibitors aren’t used to help control flashbacks or phobias or to decrease the
craving for alcohol.
72. Answer: (D) Hold the next dose and obtain an order for a stat serum lithium
level 84. Answer: (D) Stopping the drug can cause withdrawal symptoms
Rationale: Diarrhea and vomiting are manifestations of Lithium toxicity. The next Rationale: Stopping antianxiety drugs such as benzodiazepines can cause the
dose of lithium should be withheld and test is done to validate the observation. A. client to have withdrawal symptoms. Stopping a benzodiazepine doesn’t tend to
The manifestations are not due to drug interaction. B. Cogentin is used to cause depression, increase cognitive abilities, or decrease sleeping difficulties.
manage the extra pyramidal symptom side effects of antipsychotics. C. The
common side effects of Lithium are fine hand tremors, nausea, polyuria and 85. Answer: (B) Behavioral difficulties
polydipsia. Rationale: Adolescents tend to demonstrate severe irritability and behavioral
problems rather than simply a depressed mood. Anxiety disorder is more
73. Answer: (C) A living, learning or working environment. commonly associated with small children rather than with adolescents. Cognitive
Rationale: A therapeutic milieu refers to a broad conceptual approach in which all impairment is typically associated with delirium or dementia. Labile mood is more
aspects of the environment are channeled to provide a therapeutic environment characteristic of a client with cognitive impairment or bipolar disorder.
for the client. The six environmental elements include structure, safety, norms;
limit setting, balance and unit modification. A. Behavioral approach in psychiatric 86. Answer: (D) It’s a mood disorder similar to major depression but of mild to
care is based on the premise that behavior can be learned or unlearned through moderate severity
the use of reward and punishment. B. Cognitive approach to change behavior is Rationale: Dysthymic disorder is a mood disorder similar to major depression but
done by correcting distorted perceptions and irrational beliefs to correct it remains mild to moderate in severity. Cyclothymic disorder is a mood disorder
maladaptive behaviors. D. This is not congruent with therapeutic milieu. characterized by a mood range from moderate depression to hypomania. Bipolar
I disorder is characterized by a single manic episode with no past major
74. Answer: (B) Transference depressive episodes. Seasonalaffective disorder is a form of depression
Rationale: Transference is a positive or negative feeling associated with a occurring in the fall and winter.
significant person in the client’s past that are unconsciously assigned to another
A. Splitting is a defense mechanism commonly seen in a client with personality 87. Answer: (A) Vascular dementia has more abrupt onset
disorder in which the world is perceived as all good or all bad C. Countert- Rationale: Vascular dementia differs from Alzheimer’s disease in that it has a
transference is a phenomenon where the nurse shifts feelings assigned to more abrupt onset and runs a highly variable course. Personally change is
someone in her past to the patient D. Resistance is the client’s refusal to submit common in Alzheimer’s disease. The duration of delirium is usually brief. The
himself to the care of the nurse inability to carry out motor activities is common in Alzheimer’s disease.

75. Answer: (B) Adventitious 88. Answer: (C) Drug intoxication


Rationale: Adventitious crisis is a crisis involving a traumatic event. It is not part Rationale: This client was taking several medications that have a propensity for
of everyday life. A. Situational crisis is from an external source that upset ones producing delirium; digoxin (a digitalis glycoxide), furosemide (a thiazide diuretic),
psychological equilibrium C and D. Are the same. They are transitional or and diazepam (a benzodiazepine). Sufficient supporting data don’t exist to
developmental periods in life suspect the other options as causes.

76. Answer: (C) Major depression 89. Answer: (D) The client is experiencing visual hallucination
Rationale: The DSM-IV-TR classifies major depression as an Axis I disorder. Rationale: The presence of a sensory stimulus correlates with the definition of a
Borderline personality disorder as an Axis II; obesity and hypertension, Axis III. hallucination, which is a false sensory perception. Aphasia refers to a
communication problem. Dysarthria is difficulty in speech production. Flight of
77. Answer: (B) Transference ideas is rapid shifting from one topic to another.
Rationale: Transference is the unconscious assignment of negative or positive
feelings evoked by a significant person in the client’s past to another person. 90. Answer: (D) The client looks at the shadow on a wall and tells the nurse she
Intellectualization is a defense mechanism in which the client avoids dealing with sees frightening faces on the wall.
emotions by focusing on facts. Triangulation refers to conflicts involving three Rationale: Minor memory problems are distinguished from dementia by their
family members. Splitting is a defense minor severity and their lack of significant interference with the client’s social or
mechanism commonly seen in clients with personality disorder in which the world occupational lifestyle. Other options would be included in the history data but
is perceived as all good or all bad. don’t directly correlate with the client’s lifestyle.

78. Answer: (B) Hypochondriasis 91. Answer: (D) Loose association


Rationale: Complains of vague physical symptoms that have no apparent Rationale: Loose associations are conversations that constantly shift in topic.
medical causes are characteristic of clients with hypochondriasis. In many cases, Concrete thinking implies highly definitive thought processes. Flight of ideas is
the GI system is affected. Conversion disorders are characterized by one or characterized by conversation that’s disorganized from the onset. Loose
more neurologic symptoms. The client’s symptoms don’t suggest severe anxiety. associations don’t necessarily start in a cogently, then becomes loose.
A client experiencing sublimation channels
maladaptive feelings or impulses into socially acceptable behavior 92. Answer: (C) Paranoid
Rationale: Because of their suspiciousness, paranoid personalities ascribe
79. Answer: (C) Hypochondriasis malevolent activities to others and tent to be defensive, becoming quarrelsome
Rationale: Hypochodriasis in this case is shown by the client’s belief that she has and argumentative. Clients with antisocial personality disorder can also be
a serious illness, although pathologic causes have been eliminated. The antagonistic and argumentative but are less suspicious than paranoid
disturbance usually lasts at lease 6 with identifiable life stressor such as, in this personalities. Clients with histrionic personality disorder are dramatic, not
case, course examinations. Conversion disorders are characterized by one or suspicious and argumentative. Clients with schizoid personality disorder are
more neurologic symptoms. Depersonalization refers to persistent recurrent usually detached from other and tend to have eccentric behavior.
episodes of feeling detached from one’s self or body. Somatoform disorders
generally have a chronic course with few remissions. 93. Answer: (C) Explain that the drug is less affective if the client smokes
Rationale: Olanzapine (Zyprexa) is less effective for clients who smoke
80. Answer: (A) Triazolam (Halcion) cigarettes. Serotonin syndrome occurs with clients who take a combination of
Rationale: Triazolam is one of a group of sedative hypnotic medication that can antidepressant medications. Olanzapine doesn’t cause euphoria, and
be used for a limited time because of the risk of dependence. Paroxetine is a extrapyramidal adverse reactions aren’t a problem. However, the client should be
scrotonin-specific reutake inhibitor used for treatment of depression panic aware of adverse effects such as tardive dyskinesia.
disorder, and obsessive-compulsive disorder. Fluoxetine is a scrotonin-specific
reuptake inhibitor used for depressive disorders and obsessive-compulsive 94. Answer: (A) Lack of honesty
disorders. Risperidome is indicated for psychotic disorders. Rationale: Clients with antisocial personality disorder tent to engage in acts of
dishonesty, shown by lying. Clients with schizotypal personality disorder tend to
81. Answer: (D) It promotes emotional support or attention for the client be superstitious. Clients with histrionic personality disorders tend to overreact to
Rationale: Secondary gain refers to the benefits of the illness that allow the client frustrations and disappointments, have temper tantrums, and seek attention.
to receive emotional support or attention. Primary gain enables the client to avoid
95. Answer: (A) “I’m not going to look just at the negative things about myself” b. A marathon runner complains of severe leg cramps and nausea. Tachycardia,
Rationale: As the clients makes progress on improving self-esteem, selfblame diaphoresis, pallor, and weakness are observed.
and negative self evaluation will decrease. Clients with dependent personality c. A previously healthy homemaker reports broken air conditioner for days.
disorder tend to feel fragile and inadequate and would be extremely unlikely to Tachypnea, hypotension, fatigue, and profuse diaphoresis are observed.
discuss their level of competence and progress. These clients focus on self and d. A homeless person, poor historian, presents with altered mental status, poor
aren’t envious or jealous. Individuals with dependent personality disorders don’t muscle coordination, and hot, dry, ashen skin. Duration of exposure is unknown.
take over situations because they see themselves as inept and inadequate.
7. You respond to a call for help from the ED waiting room. There is an elderly
96. Answer: (C) Assess for possible physical problems such as rash patient lying on the floor. List the order for the actions that you must perform.
Rationale: Clients with schizophrenia generally have poor visceral recognition
because they live so fully in their fantasy world. They need to have as in-depth a. Perform the chin lift or jaw thrust maneuver.
assessment of physical complaints that may spill over into their delusional b. Establish unresponsiveness.
symptoms. Talking with the client won’t provide as assessment of his itching, and c. Initiate cardiopulmonary resuscitation (CPR).
itching isn’t as adverse reaction of antipsychotic drugs, calling the physician to d. Call for help and activate the code team.
get the client’s medication increased doesn’t address his physical complaints. e. Instruct a nursing assistant to get the crash cart.

97. Answer: (B) Echopraxia 8. The emergency medical service (EMS) has transported a patient with severe
Rationale: Echopraxia is the copying of another’s behaviors and is the result of chest pain. As the patient is being transferred to the emergency stretcher, you
the loss of ego boundaries. Modeling is the conscious copying of someone’s note unresponsiveness, cessation of breathing, and no palpable pulse. Which
behaviors. Ego-syntonicity refers to behaviors that correspond with the task is appropriate to delegate to the nursing assistant?
individual’s sense of self. Ritualism behaviors are repetitive and compulsive.
a. Chest compressions b. Bag-valve mask ventilation
98. Answer: (C) Hallucination c. Assisting with oral intubation d. Placing the defibrillator pads
Rationale: Hallucinations are sensory experiences that are misrepresentations of
reality or have no basis in reality. Delusions are beliefs not based in reality. 9. An anxious 24-year-old college student complains of tingling sensations,
Disorganized speech is characterized by jumping from one topic to the next or palpitations, and chest tightness. Deep, rapid breathing and carpal spasms are
using unrelated words. An idea of reference is a belief that an unrelated situation noted. What priority nursing action should you take?
holds special meaning for the client.
a. Notify the physician immediately.
99. Answer: (C) Regression b. Administer supplemental oxygen.
Rationale: Regression, a return to earlier behavior to reduce anxiety, is the basic c. Have the student breathe into a paper bag.
defense mechanism in schizophrenia. Projection is a defense mechanism in d. Obtain an order for an anxiolytic medication.
which one blames others and attempts to justify actions; it’s used primarily by
people with paranoid schizophrenia and delusional disorder. Rationalization is a 10.An experienced traveling nurse has been assigned to work in the ED;
defense mechanism used to justify one’s however, this is the nurse’s first week on the job. Which area of the ED is the
action. Repression is the basic defense mechanism in the neuroses; it’s an most
involuntary exclusion of painful thoughts, feelings, or experiences from appropriate assignment for the nurse?
awareness.
a. Trauma team
100.Answer: (A) Should report feelings of restlessness or agitation at once b. Triage
Rationale: Agitation and restlessness are adverse effect of haloperidol and can c. Ambulatory or fats track clinic
be treated with antocholinergic drugs. Haloperidol isn’t likely to cause d. Pediatric medicine team
photosensitivity or control essential hypertension. Although the client may
experience increased concentration and activity, these effects are due to a 11.A tearful parent brings a child to the ED for taking an unknown amount of
decreased in symptoms, not the drug itself. children’s chewable vitamins at an unknown time. The child is currently alert and
EMERGENCY NURSING asymptomatic. What information should be immediately reported to the
1. You are the charge nurse in an emergency department (ED) and must assign physician?
two staff members to cover the triage area. Which team is the most appropriate
for this assignment? a. The ingested children’s chewable vitamins contain iron.
b. The child has been treated several times for ingestion of toxic substances.
a. An advanced practice nurse and an experienced LPN/LVN c. The child has been treated several times for accidental injuries.
b. An experienced LPN/LVN and an inexperienced RN d. The child was nauseated and vomited once at home.
c. An experienced RN and an inexperienced RN
d. An experienced RN and a nursing assistant 12.In caring for a victim of sexual assault, which task is most appropriate for an
LPN/LVN?
2. You are working in the triage area of an ED, and four patients approach the
triage desk at the same time. List the order in which you will assess these a. Assess immediate emotional state and physical injuries
patients. b. Collect hair samples, saliva swabs, and scrapings beneath fingernails.
c. Provide emotional support and supportive communication.
a. An ambulatory, dazed 25-year-old male with a bandaged head wound d. Ensure that the “chain of custody” is maintained.
b. An irritable infant with a fever, petechiae, and nuchal rigidity
c. A 35-year-old jogger with a twisted ankle, having pedal pulse and no deformity 13.You are caring for a victim of frostbite to the feet. Place the following
d. A 50-year-old female with moderate abdominal pain and occasional vomiting interventions in the correct order.
_____, _____, _____, _____
a. Apply a loose, sterile, bulky dressing.
3. In conducting a primary survey on a trauma patient, which of the following is b. Give pain medication.
considered one of the priority elements of the primary survey? c. Remove the victim from the cold environment.
d. Immerse the feet in warm water 100o F to 105o F (40.6o C to 46.1o C)
a. Complete set of vital signs
b. Palpation and auscultation of the abdomen 14.A patient sustains an amputation of the first and second digits in a chainsaw
c. Brief neurologic assessment accident. Which task should be delegated to the LPN/LVN?
d. Initiation of pulse oximetry
a. Gently cleanse the amputated digits with Betadine solution.
4. A 56-year-old patient presents in triage with left-sided chest pain, diaphoresis, b. Place the amputated digits directly into ice slurry.
and dizziness. This patient should be prioritized into which category? c. Wrap the amputated digits in sterile gauze moistened with saline.
d. Store the amputated digits in a solution of sterile normal saline.
a. High urgent b. Urgent c. Non-urgent d. Emergent
15.A 36-year-old patient with a history of seizures and medication compliance of
5. The physician has ordered cooling measures for a child with fever who is likely phenytoin (Dilantin) and carbamazepine (Tegretol) is brought to the ED by the
to be discharged when the temperature comes down. Which of the following MS personnel for repetitive seizure activity that started 45 minutes prior to arrival.
would be appropriate to delegate to the nursing assistant? You anticipate that the physician will order which drug for status epilepticus?

a. Assist the child to remove outer clothing. a. PO phenytoin and carbamazepine b. IV lorazepam (Ativan)
b. Advise the parent to use acetaminophen instead of aspirin. c. IV carbamazepam d. IV magnesium sulfate
c. Explain the need for cool fluids.
d. Prepare and administer a tepid bath. 16.You are preparing a child for IV conscious sedation prior to repair of a facial
laceration. What information should you immediately report to the physician?
6. It is the summer season, and patients with signs and symptoms of heat-related
illness present in the ED. Which patient needs attention first? a. The parent is unsure about the child’s tetanus immunization status.
b. The child is upset and pulls out the IV.
a. An elderly person complains of dizziness and syncope after standing in the c. The parent declines the IV conscious sedation.
sun for several hours to view a parade d. The parent wants information about the IV conscious sedation.
17.An intoxicated patient presents with slurred speech, mild confusion, and 27.In a multiple-trauma victim, which assessment finding signals the most
uncooperative behavior. The patient is a poor historian but admits to “drinking a serious and life-threatening condition?
few on the weekend.” What is the priority nursing action for this patient?
a. A deviated trachea b. Gross deformity in a lower extremity
a. Obtain an order for a blood alcohol level. c. Decreased bowel sounds d. Hematuria
b. Contact the family to obtain additional history and baseline information.
c. Administer naloxone (Narcan) 2 - 4 mg as ordered.
d. Administer IV fluid support with supplemental thiamine as ordered. 28.A patient in a one-car rollover presents with multiple injuries. Prioritize the
interventions that must be initiated for this patient.
18.When an unexpected death occurs in the ED, which of the following tasks is
most appropriate to delegate to the nursing assistant? a. Secure/start two large-bore IVs with normal saline
b. Use the chin lift or jaw thrust method to open the airway.
a. Escort the family to a place of privacy. c. Assess for spontaneous respirations
b. Go with the organ donor specialist to talk to the family. d. Give supplemental oxygen per mask.
c. Assist with postmortem care. e. Obtain a full set of vital signs.
d. Assist the family to collect belongings. f. Remove patient’s clothing.
g. Insert a Foley catheter if not contraindicated.
19.Following emergency endotracheal intubation, you must verify tube placement
and secure the tube. List in order the steps that are required to perform this 29.In the work setting, what is your primary responsibility in preparing for disaster
function? management that includes natural disasters or bioterrorism incidents?

a. Obtain an order for a chest x-ray to document tube placement. a. Knowledge of the agency’s emergency response plan
b. Secure the tube in place. b. Awareness of the signs and symptoms for potential agnets of bioterrorism
c. Auscultate the chest during assisted ventilation. c. Knowledge of how and what to report to the CDC
d. Confirm that the breath sounds are equal and bilateral. d. Ethical decision-making about exposing self to potentially lethal substances

20.A teenager arrives by private car. He is alert and ambulatory, but this shirt and 30.You are giving discharge instructions to a woman who has been treated for
pants are covered with blood. He and his hysterical friends are yelling and trying contusions and bruises sustained during an episode of domestic violence. What
to explain that that they were goofing around and he got poked in the abdomen is your priority intervention for this patient?
with a stick. Which of the following comments should be given first
consideration? a. Transportation arrangements to a safe house
b. Referral to a counselor
a. “There was a lot of blood and we used three bandages.” c. Advise about contacting the police
b. “He pulled the stick out, just now, because it was hurting him.” d. Follow-up appointment for injuries
c. “The stick was really dirty and covered with mud.” RATIO ER
d. “He’s a diabetic, so he needs attention right away.” 1. ANSWER C – Triage requires at least one experienced RN. Pairing an
experienced RN with inexperienced RN provides opportunities for mentoring.
21.A prisoner, with a known history of alcohol abuse, has been in police custody Advanced practice nurses are qualified to perform triage; however, their services
for 48 hours. Initially, anxiety, sweating, and tremors were noted. Now, are usually required in other areas of the ED. An LPN/LVN is not qualified to
disorientation, hallucination, and hyper-reactivity are observed. The medical perform the initial patient assessment or decision making. Pairing an experienced
diagnosis is delirium tremens. What is the priority nursing diagnosis? RN with a nursing assistant is the second best option, because the assistant can
obtain vital signs and assist in transporting.
a. Risk for Injury related to seizures
b. Risk for Other-Directed Violence related to hallucinations 2. ANSWER B, A, D, C – An irritable infant with fever and petechiae should be
c. Risk for Situational Low Self-esteem related to police custody further assessed for other meningeal signs. The patient with the head wound
d. Risk for Nutritional Deficit related to chronic alcohol abuse needs additional history and assessment for intracranial pressure. The patient
with moderate abdominal pain is uncomfortable, but not unstable at this point.
22.You are assigned to telephone triage. A patient who was stung by a common For the ankle injury, medical evaluation can be delayed 24 – 48 hours if
honey bee calls for advice, reports pain and localized swelling, but denies any necessary.
respiratory distress or other systemic signs of anaphylaxis. What is the action
that you should direct the caller to perform? 3. ANSWER C – A brief neurologic assessment to determine level of
consciousness and pupil reaction is part of the primary survey. Vital signs,
a. Call 911. b. Remove the stinger by scraping. assessment of the abdomen, and initiation of pulse oximetry are considered part
c. Apply a cool compress. d. Take an oral antihistamine. of the secondary survey.

23.In relation to submersion injuries, which task is most appropriate to delegate 4. ANSWER D – Chest pain is considered an emergent priority, which is defined
to an LPN/LVN? as potentially life-threatening. Patients with urgent priority need treatment within
2 hours of triage (e.g. kidney stones). Non-urgent conditions can wait for hours or
a. Talk to a community group about water safety issues. even days. (High urgent is not commonly used; however, in 5-tier triage systems,
b. Stabilize the cervical spine for an unconscious drowning victim. High urgent patients fall between emergent and urgent in terms of the time
c. Remove wet clothing and cover the victim with a warm blanket. lapsing prior to treatment).
d. Monitor an asymptomatic near-drowning victim.
5. ANSWER A – The nursing assistant can assist with the removal of the outer
24.You are assessing a patient who has sustained a cat bite to the left hand. The clothing, which allows the heat to dissipate from the child’s skin. Advising and
cat is up-to-date immunizations. The date of the patient’s last tetanus shot is explaining are teaching functions that are the responsibility of the RN. Tepid
unknown. Which of the following is the priority nursing diagnosis? baths are not usually performed because of potential for rebound and shivering.

a. Risk for Infection related to organisms specific to cat bites 6. ANSWER D – The homeless person has symptoms of heat stroke, a medical
b. Impaired Skin Integrity related to puncture wounds emergency, which increases risk for brain damage. Elderly patients are at risk for
c. Ineffective Health Maintenance related to immunization status heat syncope and should be educated to rest in cool area and avoid future
d. Risk for Impaired Mobility related to potential tendon damage similar situations. The runner is having heat crams, which can be managed with
rest and fluids. The housewife is experiencing heat exhaustion, and management
25.These patients present to the ED complaining of acute abdominal pain. includes fluids (IV or parenteral) and cooling measures. The prognosis for
Prioritize them in order of severity. recovery is good.

a. A 35-year-old male complaining of severe, intermittent cramps with three 7. ANSWER B, D, A, C, E – Establish unresponsiveness first. (The patient may
episodes of watery diarrhea, 2 hours after eating have fallen and sustained a minor injury.) If the patient is unresponsive, get help
b. A 11-year-old boy with a low-grade fever, left lower quadrant tenderness, and have someone initiate the code. Performing the chin lift or jaw thrust
nausea, and anorexia for the past 2 days maneuver opens the airway. The nurse is then responsible for starting CPR.
c. A 40-year-old female with moderate left upper quadrant pain, vomiting small CPR should not be interrupted until the patient recovers or it is determined that
amounts of yellow bile, and worsening symptoms over the past week heroic efforts have been exhausted. A crash cart should be at the site when the
d. A 56-year-old male with a pulsating abdominal mass and sudden onset of code team arrives; however, basic CPR can be effectively performed until the
pressure-like pain in the abdomen and flank within the past hour team arrives.

26.The nursing manager decides to form a committee to address the issue of 8. ANSWER A – Nursing assistants are trained in basic cardiac life support and
violence against ED personnel. Which combination of employees is best suited to can perform chest compressions. The use of the bag-valve mask requires
fulfill this assignment? practice and usually a respiratory therapist will perform this function. The nurse
or the respiratory therapist should provide PRN assistance during intubation. The
a. ED physicians and charge nurses defibrillator pads are clearly marked; however, placement should be done by the
b. Experienced RNs and experienced paramedics RN or physician because of the potential for skin damage and electrical arcing.
c. RNs, LPN/LVNs, and nursing assistants
d. At least one representative from each group of ED personnel
9. ANSWER C – The patient is hyperventilating secondary to anxiety, and 26. ANSWER D – At least one representative from each group should be
breathing into a paper bag will allow rebreathing of carbon dioxide. Also, included because all employees are potential targets fro violence in the ED.
encouraging slow breathing will help. Other treatments such as oxygen and
medication may be needed if other causes are identified. 27. ANSWER A – A deviated trachea is a symptoms of tension pneumothorax.
All of the other symptoms need to be addressed, but are of lesser priority.
10. ANSWER C – The fast track clinic will deal with relatively stable patients.
Triage, trauma, and pediatric medicine should be staffed with experienced 28. ANSWER C, B, D, A, E, F, G – For a multiple trauma victim, many
nurses who know the hospital routines and policies and can rapidly locate interventions will occur simultaneously as team members assist in the
equipment. resuscitation. Methods to open the airway such as the chin lift or jaw thrust can
be used simultaneously while assessing for spontaneous respirations. However,
11. ANSWER A – Iron is a toxic substance that can lead to massive hemorrhage, airway and oxygenation are priority. Starting IVs for fluid resuscitation is part of
coma, shock, and hepatic failure. Deferoxame is an antidote that can be used for supporting circulation. (EMS will usually establish at least one IV in the field.)
severe cases of iron poisoning. Other information needs additional investigation, Nursing assistants can be directed to take vitals and remove clothing. Foley
but will not change the immediate diagnostic testing or treatment plan. catheter is necessary to closely monitor output.

12. ANSWER C – The LPN/LVN is able to listen and provide emotional support 29. ANSWER A – In preparing for disasters, the RN should be aware of the
for her patients. The other tasks are the responsibility of an RN or, if available, a emergency response plan. The plan gives guidance that includes roles of team
SANE (sexual assault nurse examiner) who has received training to assess, members, responsibilities, and mechanisms of reporting. Signs and symptoms of
collect and safeguard evidence, and care for these victims. many agents will mimic common complaints, such as flu-like symptoms.
Discussions with colleagues and supervisors may help the individual nurse to
13. ANSWER C, B, D, A – The victim should be removed from the cold sort through ethical dilemmas related to potential danger to self.
environment first, and then the rewarming process can be initiated. It will be
painful, so give pain medication prior to immersing the feet in warmed water. 30. ANSWER A – Safety is a priority for this patient, and she should not return to
a place where violence could reoccur. The other options are important for the
14. ANSWER C – The only correct intervention is C. the digits should be gently long term management of this care.
cleansed with normal saline, wrapped in sterile gauze moistened with saline, and
placed in a plastic bag or container. The container is then placed on ice.

15. ANSWER B – IV Lorazepam (Ativan) is the drug of choice for status


epilepticus. Tegretol is used in the management of generalized tonic-clonic,
absence or mixed type seizures, but it does not come in an IV form. PO (per os)
medications are inappropriate for this emergency situation. Magnesium sulfate is
given to control seizures in toxemia of pregnancy.

16. ANSWER C – Parent refusal is an absolute contraindication; therefore, the


physician must be notified. Tetanus status can be addressed later. The RN can
restart the IV and provide information about conscious sedation; if the parent still
notsatisfied, the physician can give more information.

17. ANSWER D – The patient presents with symptoms of alcohol abuse and
there is a risk for Wernicke’s syndrome, which is caused by a thiamine
deficiency. Multiples drug abuse is not uncommon; however, there is nothing in
the question that suggests an opiate overdose that requires naloxone. Additional
information or the results of the blood alcohol level are part of the total treatment
plan but should not delay the immediate treatment.

18. ANSWER C – Postmortem care requires some turning, cleaning, lifting, etc.,
and the nursing assistant is able to assist with these duties. The RN should take
responsibility for the other tasks to help the family begin the grieving process. In
cases of questionable death, belongings may be retained for evidence, so the
chain of custody would have to be maintained.

19. ANSWER C, D, B, A – Auscultating and confirming equal bilateral breath


sounds should be performed in rapid succession. If the sounds are not equal or if
the sounds are heard over the mid-epigastric area, tube placement must be
corrected immediately. Securing the tube is appropriate while waiting for the x-
ray study.

20. ANSWER B – An impaled object may be providing a tamponade effect, and


removal can precipitate sudden hemodynamic decompensation. Additional
history including a more definitive description of the blood loss, depth of
penetration, and medical history should be obtained. Other information, such as
the dirt on the stick or history of diabetes, is important in the overall treatment
plan, but can be addressed later.

21. ANSWER A – The patient demonstrates neurologic hyperactivity and is on


the verge of a seizure. Patient safety is the priority. The patient needs
chlordiazepoxide (Librium) to decrease neurologic irritability and phenytoin
(Dilantin) for seizures. Thiamine and haloperidol (Haldol) will also be ordered to
address the other problems. The other diagnoses are pertinent but not as
immediate.

22. ANSWER B – The stinger will continue to release venom into the skin, so
prompt removal of the stinger is advised. Cool compresses and antihistamines
can follow. The caller should be further advised about symptoms that require 911
assistance.

23. ANSWER D – The asymptomatic patient is currently stable but should be


observed for delayed pulmonary edema, cerebral edema, or pneumonia.
Teaching and care of critical patients is an RN responsibility. Removing clothing
can be delegated to a nursing assistant.

24. ANSWER A – Cat’s mouths contain a virulent organism, Pasteurella


multocida, that can lead to septic arthritis or bacteremia. There is also a risk for
tendon damage due to deep puncture wounds. These wounds are usually not
sutured. A tetanus shot can be given before discharge.

25. ANSWER D, B, C, A – The patient with a pulsating mass has an abdominal


aneurysm that may rupture and he may decompensate suddenly. The 11-year-
old boy needs evaluation to rule out appendicitis. The woman needs evaluation
for gallbladder problems that appear to be worsening. The 35-year-old man has
food poisoning, which is usually self-limiting.

Anda mungkin juga menyukai